You are on page 1of 195

Test One

1. With regards to haemoglobinopathies, which of the following is FALSE?


a)alpha thalassaemia are caused by deletions in the alpha globin gene
b)beta thalassaemia is not clinically apparent until 6 months of age
c)HbS forms as a result of a point mutation in codon 7 of the beta globin gene
d)sickle cell disease is the commonest genetic disorder in the UK
e)they are blood cell disorders which cause haemolytic anaemia
2. Caused by deletion of all four alpha globin genes
a)beta thalassaemia major
b)Hb Barts hydrops fetails
c)Diamond-Blackfan anaemia
d)alpha thalassaemia trait
e)hereditary spherocytosis
3. Inheritance for HbS from one parent and a normal beta globin gene from another
a)Sickle cell disease
b)Sickle beta thalassaemia
c)Sickle Cyprositis
d)Sickle trait
e)Sickle cell anaemia
4. Which of the following is a type of red cell aplasia (cause of anaemia)?
a)Glucose-6-phosphate dehydrogenase deficiency
b)Haemolytic disease of the newborn
c)Folic acid deficiency
d)Hereditary spherocytosis
e)Diamond Blackfan anaemia
5. With regards to herediatary spherocytosis, which of the following is FALSE?
a)usually has autosomal dominant inheritance
b)caused by mutations in genes for proteins such as spectrin, ankrin or band 3
c)red blood cells are destroyed in the spleen
d)occurs in 1 in 5000 births in caucasians
e)aplastic crises are common
6. Children from which area are most likely to develop beta thalassaemia
a)United Kingdom
b)Mediterranean
c)Africa
d)North America
e)Gulf States
7. Deletion of one or two alpha globin genes
a)hereditary spherocytosis
b)Hb Barts hydrops fetails
c)Diamond-Blackfan anaemia
d)alpha thalassaemia major
e)alpha thalassaemia trait
8. Which of the following is a red cell enzyme disorder (cause of anaemia)?
a)Folic acid deficiency
b)Diamond Blackfan anaemia
c)Glucose-6-phosphate dehydrogenase deficiency
d)Haemolytic disease of the newborn
e)Hereditary spherocytosis
9. Rare autosomal recessive disorder characterised by bone marrow failure together with signs of
pancreatic exocrine failure and skeletalk abnormalities
a)Fanconi anaemia
b)Diamond-Blackfan anaemia
c)hereditary spherocytosis
d)G6PD deficiency
e)Schwachman-Diamond syndrome
10. Anaemia in an infant aged 1-12 months can be defines as
a)< 14g/dL
b)< 10 g/dL
c)< 8 g/dL
d)< 6 g/dL
e)< 12 g/dL
11. Which of the following is a cause of ineffective erythroporesis (cause of anaemia)?
a)Folic acid deficiency
b)Hereditary spherocytosis
c)Haemolytic disease of the newborn
d)Diamond Blackfan anaemia
e)Glucose-6-phosphate dehydrogenase deficiency
12. Involved in management of haemophilia A
a)Recombinant FV
b)Recombinant FIX
c)Recombinant FVIII
d)Recombinant FIV
e)Recombinant FVII
13. Usually associated with parvovirus B19 infection in those with hereditary spherocytosis
a)mild to moderate splenomegaly
b)aplastic crisis
c)gallstones
d)jaundice
e)anaemia
14. Anaemia in a child aged 1-12 years can be defined as
a)< 11 g/dL
b)< 5 g/dL
c)< 7 g/dL
d)< 15g/dL
e)< 13 g/dL
15. Which of the following is NOT a main cause of anaemia in premature babies?
a)infection with parvovirus B19
b)Frequent blood sampling in hospital
c)Iron and folic acid deficiency (after 2-3 months)
d)Inadequate erythropoetin production
e)Reduced red cell lifespan
16. Characterised by FIX deficiency
a)Diamond-Blackfan anaemia
b)G6PD deficiency
c)haemophilia B
d)haemophilia A
e)Fanconi anaemia
17. The first globin chain produced in humans
a)Gamma-globin
b)Delta-globin
c)Beta-globin
d)Alpha-globin
e)Epsilon-globin

18. The only cure for sickle cell disease


a)Hydroxyurea
b)Spenectomy
c)Exchange transfusion
d)Progressive desensitization
e)Bone marrow transplant
19. The diagnostic clue is that there is anaemia, the reticulocyte count is low and the bilirubin is normal
a)congenital red cell aplasia
b)alpha thalassaemia major
c)haemolytic disease of the newborn
d)hereditary spherocytosis
e)G6PD deficiency
20. A condition resulting from overstimulation of the blood- clotting mechanisms in response to disease
or injury. Results in generalized blood coagulation and excessive consumption of coagulation factors.
a)Disseminated Intravascular coagulation
b)Haemophilia A
c)Haemophilia B
d)von Wilebrand's disease
e)Immune thrombocytopaenia
21. The diagnostic clue is a positive direct anti-globulin (Coombs) test
a)congenital red cell aplasia
b)hereditary spherocytosis
c)haemolytic disease of the newborn
d)G6PD deficiency
e)alpha thalassaemia major
22. Autosomal recessive aplastic anaemia and an increased predisposition to malignancy. Also causes
mental retardation, poor growth, skeletal abnormalities, and kidneys of an unusual shape or in an
unusual position. Usually diagnosed between five and ten years.
a)Fanconi anaemia
b)hereditary spherocytosis
c)Diamond-Blackfan anaemia
d)G6PD deficiency
e)Schwachman-Diamond syndrome
23. Which of the following globin chains is not normaly found in adult blood?
a)beta 2
b)they all are
c)gamma 2
d)delta 2
e)alpha 2
24. With regards to G6PD deficiency, which of the following is FALSE?
a)affects the pentose phosphate pathway
b)females with this condition seldom survive
c)associated with neonatal jaundice
d)acute haemolysis can be precipitated by broad beans
e)X-linked
25. Anaemia due to antibodies against blood group antigens
a)hereditary spherocytosis
b)congenital red cell aplasia
c)G6PD deficiency
d)haemolytic disease of the newborn
e)alpha thalassaemia major

26. The most important diagnostic feature for beta thalassaemia trait
a)anaemia
b)raised HbF
c)reduced MCH
d)reduced MCV
e)raised HbA2
27. The process of production of blood cells and platelets
a)haemotological
b)haemopoesis
c)haemolysis
d)haemothrombosis
e)haemotolysis
28. The commonest cause of thrombocytopaenia in childhood
a)Disseminated Intravascular coagulation
b)Haemophilia A
c)Immune thrombocytopaenia
d)von Wilebrand's disease
e)Haemophilia B
29. Patients are homozygous for HbS, they have no HbA
a)Sickle Cyprositis
b)Sickle cell disease
c)Sickle trait
d)Sickle cell anaemia
e)Sickle beta thalassaemia
30. No longer used to investigate platelet disorders as it is unreliable
a)D-dimers
b)prothrombin time
c)activated partial thromboplastin time
d)full blood count and blood film
e)bleeding time
31. The commonest red cell enzymopathy
a)G6PD deficiency
b)beta thalassaemia
c)Hereditary spherocytosis
d)Diamond Blackfan anaemia
e)alpha thalassaemia
32. The severiy of haemophilia wuith a Factor VIII:C ratio of < 1% is
a)borderline
b)severe
c)moderate
d)not applicable
e)mild
33. Which of the following is an immune response cause of anaemia?
a)Diamond Blackfan anaemia
b)Hereditary spherocytosis
c)Glucose-6-phosphate dehydrogenase deficiency
d)Folic acid deficiency
e)Haemolytic disease of the newborn
34. Which of the following should be avioded in patients with haemophilia
A) Intramuscular injections
b)NSAIDs
C) Aspirin
d) B&C
e)A, B & C
35. What is the name for the neonatal screening test for sickle cell disease?
a)Guthrie
b)Coombs
c)Fields
d)BCG
e)Tuberin
36. A term used ot describe the eating of non-food materials
a)pica
b)kira
c)picu
d)rika
e)pingu
37. Severe thrombocytopaenia is a platelet count
a)> 60 x 10^9 / L
b)<150 x 10^9 / L
c)< 100 x 10^9 / L
d)< 20 x 10^9 / L
e)< 50 x 10^9 / L
38. Affected children inherit HbS from one parent and a beta thalassaemia trait form the other
a)Sickle trait
b)Sickle cell disease
c)Sickle Cyprositis
d)Sickle cell anaemia
e)Sickle beta thalassaemia
39. Where would you be least to find thalassaemia in the indigenous population?
a)Spain
b)Japan
c)India
d)Sub saharan Africa
e)Iraq
40. Which of the following is NOT suggestive of haemolytic anaemia?
a)Raised reticulocyte count
b)Increased erythryopoesis in bone marrow
c)Increased urinary urobilinogen
d)Unconjugated bilirubinaemia
e)Absent red cell precursosrs in bone marrow
41. Which of the following complications of sickle cell disease is least common in children?
a)leg ulcers
b)cognitive problems
c)short stature
d)psychosocial problems
e)delayed puberty
42. In the neonate, levels of all clotting factors except factor ___ and fibrinogen are lower in term infants
at birth
a)V
b)II
c)VII
d)X
e)III

43. Which of the following is NOT characteristic of von Willebrand's disease?


a)PT Normal
b)APTT raised or normal
c)vWF Ag lowered
d)RiCoF (activity) raised
e)Ristocetin-induced platelet aggregation abnormal
44. An inherited disorder of the blood that is characterized by episodes of spontaneous bleeding
a)Folic acid deficiency
b)Diamond Blackfan anaemia
c)Von Willebrands disease
d)Glucose-6-phosphate dehydrogenase deficiency
e)Hereditary spherocytosis
45. Affected children inherit HbS from one parent and HbC from another, they have no HbA
a)Sickle cell disease
b)Sickle trait
c)Sickle Cyprositis
d)Sickle beta thalassaemia
e)Sickle cell anaemia
46. A persistent and usually painful erection of the penis that may be a clinical manifestation of sickle
cell disease
a)Hyperoophism
b)Cavernosis
c)Priapism
d)Nepotosis
e)Corpoenis
47. Which of the following is a red cell membrane disorder (cause of anaemia)?
a)Folic acid deficiency
b)Hereditary spherocytosis
c)Diamond Blackfan anaemia
d)Glucose-6-phosphate dehydrogenase deficiency
e)Haemolytic disease of the newborn
48. Which of the following is NOT part of the management of sickle cell disease?
a)avoiding exposue to cold
b)Intense exercise regieme
c)once daily folic acid
d)twice daily penicillin throughout childhood
e)full immunization
49. Which of the following is NOT a clinical feature of beta thalassaemia major?
a)hepatosplenomegaly
b)failure to thrive
c)maxillary undergrowth
d)skull bossing
e)severe anaemia and jaundice form 3-6 months of age
50. The most common inherited form of aplastic anaemia
a)G6PD deficiency
b)Fanconi anaemia
c)Schwachman-Diamond syndrome
d)Diamond-Blackfan anaemia
e)hereditary spherocytosis
51. May allow mild haemophilia A to be managed without the use of Blood products
a)Fibrinogen
b)Warfarin
c)ITP
d)Vitamin K
e)Desmopressin
52. Iron absorption is decreated by consumption of
a)Oily fish
b)fresh fruit
c)Tea
d)vegetables
e)Vitamin C
53. In a healthy term infant the average blood volume is
a)10ml/kg
b)40ml/kg
c)160ml/kg
d)80ml/kg
e)20ml/kg
54. Hb concentration at birth is normally
a)> 10 g/dL
b)> 8 g/dL
c)> 6 g/dL
d)> 12 g/dL
e)> 14g/dL
55. Also known as bone marrow failure
a)aplastic anaemia
b)alpha thalassaemia major
c)haemolytic disease of the newborn
d)hereditary spherocytosis
e)G6PD deficiency
56. Characterised by FVIII deficiency
a)haemophilia A
b)haemophilia B
c)Fanconi anaemia
d)Diamond-Blackfan anaemia
e)G6PD deficiency
57. Iron is almost entirely absorbed in the
a)Jejunum
b)Ileum
c)Duodenum
d)Colon
e)Stomach
58. Which of the following is NOT suggestive of red cell aplasia?
a)Normal bilirubin
b)Increased erythryopoesis in boine marrow
c)Low reticulocyte count and low Hb
d)Absent red cell precursors in bone marrow
e)Negative Coombs test
59. A 1 year old infant requires an iron intake of about
a)15 mg/day
b)10 mg/day
c)6 mg/day
d)8 mg/day
e)12 mg/day

Key test one


1. C
2. B
3. D
4. E
5. E
6. B
7. E
8. C
9. E
10. B
11. A
12. C
13. B
14. A
15. A
16. C
17. E
18. E
19. A
20. A
21. C
22. A
23. C
24. B
25. D
26. E
27. B
28. C
29. D
30. E
31. A
32. B
33. E
34. E
35. A
36. A
37. D
38. E
39. D
40. E
41. A
42. C
43. D
44. C
45. A
46. C
47. B
48. B
49. C
50. B
51. E
52. C
53. D
54. E
55. A
56. A
57. C
58. B
59. D
Test 2
1. Management includes prostaglandin infusion and balloon atrial septostomy
a)Tetralogy of Fallot
b) Hypoplastic left heart syndrome
c) Transposition of the great vessels
d) PDA
e) Coarctation of the Aorta
2. Clinical features may include: active precordium, soft pansystolic or no murmur, loud pulmonary
second souns, tachypnoea, tachycardia and hepatomegaly
a)large ASD
b) mitral stenosis
c) patent ductus arteriosus
d) large VSD
e) aortic stenosis
3. Physical signs include: An ejection systolic murmur and ejection click best heard at the upper left
sternal edge, soft or absent P2, right ventricualr hypertrophy.
a)Adult type coarctation of the aorta
b) Tricuspid regurgitation
c) Mitral stenosis
d) Pulmonary stenosis
e) Aortic stenosis

4. Clinical features include: systemic hypertension in right arm, ejection systolic murmur at upper
sternal edge, radio- femoral delay.
a)Adult type coarctation of the aorta
b)Pulmonary stenosis
c)Tricuspid regurgitation
d)Aortic stenosis
e)Mitral stenosis
5. May be associated with a sudden loss of consciousness during exercise, sress or emotion, usually in
late childhood. may be misdiagnosed as epilepsy.
a)Long QT syndrome
b)Tetralogy of Fallott
c)complete heart block
d)Transposition of the great arteries
e)supraventricular tachycardia
6. Which of the following is most likely to be associated with William's syndrome?
a)aortic valve stenosis or coarctation of the aorta
b)ASD, VSD, or tetralogy of Fallot
c)peripheral pulmonary stenosis or PDA
d)complete heart block
e)supravalvulvar aortic stenosis or peripheral pulmonary artery stenosis
7. In this condition, only the left ventricle is effective, the right being small and non-functional
a)Hypoplastic left heart syndrome
b)Tricuspid atresia
c)Transposition of the great vessels
d)Coarctation of the Aorta
e)Tetralogy of Fallot
8. Which of the following is a LATE sign of infective endocarditis?
a)Fever
b)Anaemia
c)Raised ESR
d)Clubbing
e)Splinter haemorrhages
9. Seldom used diagnostically. Reserved for haemodynamic measurements and intervention
a)Doppler ultrasound
b)ECG
c)echocardiography
d)Chest X-Ray
e)Cardiac catheterisation
10. The "figure 3 sign" is suggestive of
a)Adult type coarctation of the aorta
b)Aortic stenosis
c)Pulmonary stenosis
d)Tricuspid regurgitation
e)Mitral stenosis
11. Consists of right ventricular hypertrophy, ventricular septal defect, abnormal position of the aorta,
and pulmonary valve stenosis.
a)Transposition of the great vessels
b)Duct dependant co-arctation
c)Supravulvular pulmonary stenosis
d)Tetralogy of Fallot
e)ASVSD complex
12. Which of the following is NOT a physical sign of aortic stenosis
a)Small volume, slow rising pulse
b)Apical ejection click
c)Ejection systolic murmur radiating to the neck
d)Carotid thrill
e)Absent aortic second sound
13. What percentage of infants requiring heart surgery in the first six months of life are diagnosed
antenatally?
a)50%
b)70%
c)10%
d)20%
e)30%
14. With regards to the foetal/newborn circulation, which of the following in FALSE?
a)The ductus arteriosus usually closes around 1-2 days of age
b)Left atrial pressure is low in the foetus
c)With the first breaths, resistance to pulmonary blood flow falls
d)The foramen ovale closes as right sided pressure of the heart increases
e)The right atrium receives blood from the placenta in the foetus
15. Central cyanosis can only be recognised if the concentration of reduced haemoglobin the blood
exceeds
a)1g/dl
b)5g/dl
c)0.5g/dl
d)15g/dl
e)50g/dl
16. The classic finding on X-Ray is a narrow upper mediastinum with and 'egg on the side' appearance
of the cardiac shadow
a)Hypoplastic left heart syndrome
b)Coarctation of the Aorta
c)Transposition of the great vessels
d)PDA
e)Tetralogy of Fallot
17. Management includes diuretics, captopril, extra calories,
and surgery at 3-6 months
a)secundum ASD
b)PDA (term)
c)PDA (preterm)
d)partial AVSD
e)large VSD
18. A defect of the atrioventricular septum
a)situs solitus
b)secundum ASD
c)discordant VAC
d)concordant AVC
e)primum ASD
19. Signs include clubbing in older children, loud harsh ejection systolic murmur at the left sternal
edge from day 1 of life, usually with a single second heart sound. Chest X Ray and ECG may be
normal.
a)Tetralogy of Fallot
b)Coarctation of the Aorta
c)Transposition of the great vessels
d)Hypoplastic left heart syndrome
e)PDA
20. Characterised by episodes of tachycardia and ECG signs of ventricular pre-excitation between
attacks. Partly results from abnormal atrio-ventricular conduction along a pathway termed the
bundle of Kent.
a)Woodhouse-Sakati syndrome
b)Weill-Marchesani syndrome
c)Wilson-Mikity syndrome
d)Wolff-Parkinson-White syndrome
e)Wolf-Hirschhorn syndrome
21. Treatment for bacterial endocarditis is usually
a)nitrous oxide and intravenous magnesium sulphate
b)diuretics and ACE inhibitors
c)high dose ciproflaxin with flecainide
d)oral phosphodiesterase inhibitors
e)high dose penicillin with aminoglycoside
22. Which of the following is NOT associated with increased risk of infective endocarditis?
a)VSD
b)PDA
c)secundum ASD
d)coarctation of the aorta
e)AVSD
23. Used for maintenance of ductal patency
a)intravenous NSAIDs
b)intravenous factor VIII
c)intravenous factor IX
d)intravenous prostaglandin
e)intravenous warfarin
24. Suggestive of right ventricular hypertrophy
a)Inverted P wave
b)inverted T wave in V6
c)Angluar P wave
d)Superior QRS axis
e)Upright T wave in V1
25. Capillary refill is used to ascertain
a). Cyanosis
b). Anaemia
C. Dehydration
D. Peripheral perfusion
e) C&D
26. Treatment of cardiomyopathy is symptomatic with
a)nitrous oxide and intravenous magnesium sulphate
b)high dose penicillin with aminoglycoside
c)high dose ciproflaxin with flecainide
d)oral phosphodiesterase inhibitors
e)diuretics and ACE inhibitors
27. Occurs when blood flow through an original left-to-right cardiac shunt becomes reversed or
bidirectional. It is a result of reactive pulmonary hypertension causing pulmonary pressure to exceed
systemic pressure. Deoxygenated blood is mixed with systemic blood producing cyanosis.
a)Edwards syndrome
b)Eisenmenger's syndrome
c)Eales syndrome
d)Fabry's syndrome
e)Ehrlichiosis syndrome
28. Has been associated with the use of cisapride and erythromycin (rare)
a)Long QT syndrome
b)supraventricular tachycardia
c)Transposition of the great arteries
d)Tetralogy of Fallott
e)complete heart block
29. The most common form of long term damage to the heart from rheumatic fever
a)Pulmonary stenosis
b)Mitral stenosis
c)Rhabdomyoma
d)Aortic stenosis
e)Tricuspid stenosis
30. The most common causative organism for bacterial endocarditis
a)Streptococcus pneumoniae
b)Staphlococcus aureus
c)Streptococcus pyogenes
d)Streptococcus agalactiae
e)Streptococcus viridians
31. A superior QRS axis woiuld be suggestive of
a)Tetralogy of Fallott
b)Patent Ductus Arteriosus
c)Partial AVSD
d)Secundum ASD
e)Transposition of the great vessels
32. Can confirm cyanotic congenital heart disease if echocardiography is not available
a)hyperoxia (C02 washout) test
b)hypercardia (platelet washout) test
c)hypo-oxia (nitrogen washout) text
d)hypo-oxia (C02 washout) test
e)hyperoxia (nitrogen washout) text
33. Which of the following may be used in maintenance therapy for supraventricular tachycardia?
a) flecainide
b) sotalol
c) propanalol
d) digoxin
e) all of these
34. Which of the following is most likely to be associated with systemic lupus erythematosus?
a)ASD, VSD, or tetralogy of Fallot
b)supravalvulvar aortic stenosis or peripheral pulmonary artery stenosis
c)peripheral pulmonary stenosis or PDA
d)complete heart block
e)aortic valve stenosis or coarctation of the aorta
35. Physical signs may include: thrill at lower sternal edge, loud pansystolic murmur at lower sternal
edge, quiet pulmonary scond sound
a)small ASD
b)aortic stenosis
c)patent ductus arteriosus
d)mitral stenosis
e)small VSD
36. The management of this condition consists of a difficult neonatal operation caled the Norwood
procedure.
a)Tetralogy of Fallot
b)Coarctation of the Aorta
c)Transposition of the great vessels
d)PDA
e)Hypoplastic left heart syndrome
37. Which of the following is most likely to be associated with maternal rubella infection?
a)ASD, VSD, or tetralogy of Fallot
b)supravalvulvar aortic stenosis or peripheral pulmonary artery stenosis
c)aortic valve stenosis or coarctation of the aorta
d)complete heart block
e)peripheral pulmonary stenosis or PDA
38. Which of the following manifestations of rheumatic fever is the least common?
a)Sydenham's chorea
b)Subcutaneous nodules
c)Erythema marginatum
d)Pancarditis
e)Polyarthritis
39. The treatment of choice to restore sinus rythm in supraventricular tachycardia
a)intravous propanalol
b)intravenous adrenaline
c)intravenous alpha adrenoceptor agonist
d)intravenous protaglandin
e)intravenous adenosine
40. One feature of this condition is a defect in the middle of the heart with a single five leaflet valve
between the artia and the ventricles which stretches accross the entrie atrioventricular junction
a)Coarctation of the Aorta
b)Transposition of the great vessels
c)Hypoplastic left heart syndrome
d)cAVSD
e)Tetralogy of Fallot
41. Management includes a Blalock-Taussing shunt in children who are severely cyanosed and
pulmonary artery banding if breathless
a)Transposition of the great vessels
b)Tricuspid atresia
c)Coarctation of the Aorta
d)Hypoplastic left heart syndrome
e)Tetralogy of Fallot
42. The most common childhood arrythmia
a)ectopic atrial tachycardia
b)complete heart block
c)Long QT syndrome
d)supraventricular tachycardia
e)atrial fibrillation
43. Deep S wave in V2 and tall R wave in V6, along with a downgoing T wave wouldbe suggestive of
a)Adult type coarctation of the aorta
b)Pulmonary stenosis
c)Aortic stenosis
d)Tricuspid regurgitation
e)Mitral stenosis
44. VSDs account for ____% of all cases of congenital heart disease
a)30
b)20
c)5
d)50
e)10
45. Susceptible individuals are prone to group A Beta haemolytic streptococcal infection. After a latent
interval of 2- 6 weeks following a pharyngeal infection, polyarthritis, mild fever and malaise develop.
a)Kawasaki's disease
b)Pulmonary hypertension
c)Grave's disease
d)Rheumatic fever
e)Tuberculosis
46. Neonatal infants may be treated by surgical placement of an artificial tube between the subclavian
artery and the pulmonary artery. Hypercyanotic spells require treatment with pain relief, intravenous
propranolol, volume administration, bicarbonate, and artificial ventilation
a)PDA
b)Tetralogy of Fallot
c)Coarctation of the Aorta
d)Transposition of the great vessels
e)Hypoplastic left heart syndrome
47. Used to restore sinus rythm in supraventricular tachycardia if adenosine fails
a)Intravous propanalol
b)Electrical cardioversion with synchronised DC shock (0.5- 2J/kg body weight)
c)Circulatory and respiratory support
d)Carotid sinus massage
e)Blalock-Taussing manoeuvre
48. A defect in the centre of the atrial septum involving the foramen ovale
a)primum ASD
b)situs solitus
c)concordant AVC
d)secundum ASD
e)discordant VAC
49. Management includes coil or device closure by cardiac catheter
a)large VSD
b)partial AVSD
c)PDA (preterm)
d)PDA (term)
e)Mitral stenosis
50. Which of the following is most likely to be associated with Down's syndrome?
a)complete heart block
b)peripheral pulmonary stenosis or PDA
c)aortic arch anomalies or tetralogy of fallot
d)supravalvulvar aortic stenosis or peripheral pulmonary artery stenosis
e)atrioventricular septal defect, VSD
51. An ACE inhibitor used in the treatment of essential hypertension, either alone or with a thiazide. It
is also used as an adjunct in the treatment of congestive heart failure.
a)Catapres
b)Caduet
c)Cataflam
d)Captopril
e)Cabergoline
52. The commonest cause of death from congenital heart disease in the first week of life. There is
disturbed intrauterine development of the whole left heart, possibly due to premature closure of the
foramen ovale. The condition is characterised by a small left atrium, mitral valve, left ventricle and
aortic root are small.
a)ASD VSD complex
b)Hypoplastic left heart syndrome
c)Persistent ductus arteriosus
d)Tetralogy of fallot
e)Transposition of the great vessels
53. Which of the following is NOT a characteristic of an innocent murmur?
a)Soft blowing systolic murmur, usually from the right side
b)Localised ot the left sternal edge
c)No diastolic component
d)No added sounds
e)Radiates to the axilla
54. Management includes fluid restriction, indomethacin or ibuprofen, or surgical ligation
a)PDA (preterm)
b)PDA (term)
c)partial AVSD
d)secundum ASD
e)large VSD
55. Which of the following is FALSE?
a)Ventricular septal defects are the most common congenital heart defect
b)1-2% of live births have some cardiovascular abnormality
c)Atrial septal defect accounts for 7% of congental heart defects
d)Persistent ductus arteriosus accounts for 20% of congenital heart defects
e)8 per 1000 live births have significant cardiac malformations
56. Suggestive of left ventricular strain
a)Superior QRS axis
b)inverted T wave in V6
c)Angluar P wave
d)Inverted P wave
e)Upright T wave in V1
57. The most common cause of cyanotic congenital heart disease
a)Tetralogy of Fallot
b)PDA
c)Coarctation of the Aorta
d)Hypoplastic left heart syndrome
e)Transposition of the great vessels
58. May act on the pulmonary vasculature on the cyclic GMP pathway
a). Oral phosphodiesterase inhibitors
b). intravenous magnesium sulphate
c). inhalend nitrous oxide
d). Intravenous prostacyclin
e)A, B or C
59. A rare condition usually related to the presence of anti-Ro or anti-La antibodies in the maternal
serum
a)Long QT syndrome
b)complete heart block
c)ectopic atrial tachycardia
d)atrial fibrillation
e)supraventricular tachycardia
60. Which of the following is most likely to be associated with Chromosome 22q11.2 deletion
a)supravalvulvar aortic stenosis or peripheral pulmonary artery stenosis
b)complete heart block
c)atrioventricular septal defect, VSD
d)aortic arch anomalies or tetralogy of fallot+
e)peripheral pulmonary stenosis or PDA
61. Most children present with a continuous murmur from beneath the left clavicle
a)Eisenmenger's syndrome
b)PDA
c)ASD
d)VSD
e)PFA
62. Which of the following is most likely to be associated with Turner's syndrome?
a)aortic valve stenosis or coarctation of the aorta
b)supravalvulvar aortic stenosis or peripheral pulmonary artery stenosis
c)peripheral pulmonary stenosis or PDA
d)complete heart block
e)ASD, VSD, or tetralogy of Fallot
63. Characterised by a fixed and widely split second heart sound, and an ejection systolic or
pansystolic murmur.
a)PDA
b)ASD
c)TOF
d)VSD
e)PFO
64. May act on the cycluic AMP pathway
a). inhalend nitrous oxide
b). Intravenous prostacyclin
c). inhaled iloprost
d)B or C
e)A, B or C
65. Which of the following is most likely to be associated with fetal alcohol syndrome?
a)aortic valve stenosis or coarctation of the aorta
b)supravalvulvar aortic stenosis or peripheral pulmonary artery stenosis
c)complete heart block
d)peripheral pulmonary stenosis or PDA
e)ASD, VSD, or tetralogy of Fallot
66. 1 in ___ children in England and Wales attend A&E every year
a) 6
b) 4
c) 10
d) 8
e) 2
67. What percentage of the neonate is comprised of water?
a) 70
b) 55
c) 80
d) 45
e) 60
68. Which of the following types on intervention study is thought to be the most valid?
a) Case controlled study
b) Case report
c) Cohort study
d) Individual RCT
e) Systematic review of RCTs
69. The guideline that a doctor can give contraceptives to a girl under 16 without parental consent,
providing it is in her best interests and she can not be persuaded to tell her parents
a) Lawton guidelines
b) Roberts guidelines
c) Peterson guidelines
d) Gillick guidelines
e) Fraser guidelines
70. An ethical justification for some trials that there is no good reason to suspect that one treatment
will be better than the other
a) therapeutic autonomy
b) double blinding
c) therapeutic equipoise
d) experimenter expectancy
e) therapeutic equidistance
71. Regarding medications for children, which of the following is FALSE?
a) In neonates, drug biotransformation is increased
b) Are usually presecribed per kg of body weight
c) Renal excretion is reduced in neonates
d) Oral formulations should be given as liquids if possible
e) IM drugs should be avoided
72. Another way of saying 'do no harm'
a) utility
b) autonomy
c) non-maleficence
d) rights
e) beneficence
73. Which of the following is the most common reason for paediatric medical admission to a DGH?
a) Gastroenterology
b) Infection
c) Neurology
d) Respiratory
e) Trauma
74. What is the most common reason for a child to be seen by their GP?
a)Skin problems
b)Asthma
c)Ear problems
d)Respiratory infection
e)Diarrhoea/vomiting
75. What is the UK stillbirth rate per 1000?
a)3
b)5
c)6
d)7
e)8
76. Defined as deaths in the first 7 days per 1000 live births
a)Stillbirth rate
b)Infant mortality rate
c)Early neonatal mortality rate
d)Perinatal mortality rate
e)Under 5 mortality rate
77. Children under the age of 16 comprise ___ of the population in the UK
a)10
b)50
c)30
d)20
e)40
78. Was is the least common cause of childhood death?
a)Cerebrovascular incidents
b)Chronic respiratory disease
c)Homicide/probably homicide
d)Epilepsy
e)Influenza / pneumonia
79. What is the most common cause of death in childhood?
a)Accidents
b)Meningitis
c)Cerebral palsy
d)Malignant neoplasms
e)Congenital malformations
80. What is the definition of stillbirth? A child born dead after the __th week of pregnancy
a)18
b)20
c)26
d)24
e)22
81. Defined as stillbirths + 1st week deaths per 1000 live births
a)Infant mortality rate
b)Perinatal mortality rate
c)Stillbirth rate
d)Early neonatal mortality rate
e)Under 5 mortality rate

Key test two

1. C
2. D
3. D
4. A
5. A
6. E
7. B
8. D
9. E
10. A
11. D
12. E
13. B
14. D
15. B
16. C
17. E
18. E
19. A
20. D
21. E
22. C
23. D
24. E
25. E
26. E
27. B
28. A
29. B
30. E
31. C
32. E
33. D
34. D
35. E
36. E
37. E
38. B
39. E
40. D
41. B
42. D
43. C
44. A
45. D
46. B
47. B
48. D
49. D
50. E
51. D
52. B
53. E
54. A
55. D
56. B
57. A
58. E
59. B
60. E
61. B
62. A
63. B
64. D
65. E
66. B
67. C
68. E
69. E
70. C
71. A
72. C
73. D
74. D
75. B
76. C
77. D
78. A
79. D
80. D
81. B
Test 3
1. What is the biggest cause of death amongst 15-19 year olds?
a) CNS related
b) injury and poisoning
c) unknown causes
d) heart disease
e) cancer
2. The average age for first sexual intercourse is
a)16
b)18
c)14
d)17
e)15
3. Which of the following is MOST common during adolescence?
a) skin conditions
b) respiratory conditions
c) significant mental health problems
d)diabetes
e) musculoskeletal conditions
4. The prevalence of mental health problems is estimated to be about
a) 37%
b) 1%
c) 6%
d) 11%
e) 25%
5. Which of the following is the most common cause of chronic urticaria?
a) IgE-mediated
b) Pharmacological
c) Infection
d) Physical agents
e) Idiopathic
6. Presents with delayed separartion of the umbilical cord, delayed wound healing, chronic skin ulcers
and deep seated infections.
a) Major immunoglobulin deficiencies
b) Defects in leucocyte function
c) Defects of bacterial phagocytic function
d) Opsonisation defects
e) Severe combined immunodeficiency
7. Abnormal response in chronic granulomatous disease
a) Nitroblue tetrazolium test
b) Full blood count
c) Chromosomal fragility test
d) total haemolytic compliment
e) tests of chemotaxis
8. Patients cannot make a normal response of the Epstein barr virus infection and either succumb to
the infection or develop secondary malignancy
a) Duncan's syndrome
b) Chronic granulomatous disease
c) Di George syndrome
d) Ataxia telangiectasia
e) Wiskott-Aldrich syndrome
9. An immune defect in Which of the following is least likely to be associated with increased
susceptability to fungal infactions?
a) Combined cellular and humoral
b) Opsonisation
c) Neutrophils
d) T lymphocytes
e) B lymphocytes
10. A defect in DNA repair
a) Chronic granulomatous disease
b) Ataxia telangiectasia
c) Wiskott-Aldrich syndrome
d) Di George syndrome
e) Duncan's syndrome
11. The progression from eczema or food allergy in infancy to asthma or allergic rhinitis in later
childhood is known as the
a) progressive hypothesis
b) autoimmune presupposition
c) allergic march
d) hygiene hypothesis
e) wedge theory
12. A reaction initiated by specific immunological mechanisms. Can be IgE mediated or non IgE
mediated.
a) Allergy
b) Hypersensitivity
c) Anaphylaxis
d) Intolerance
e) Atopy
13. Extra folds under the eyes associated with allergic disease
a) Morgan-Stanley folds
b) Marlow-Backs folds
c) Morgan-Dennie folds
d) Marlow-Hinds folds
e) Marlow-Deanshaw folds
14. Immunodeficiency associated with maldevelopment of the 5th branchialarch causing heart
malformation, palatal defects, absence of the thymus and hypocalcaemia
a) Ataxia telangiectasia
b) Duncan's syndrome
c) Wiskott-Aldrich syndrome
d) Di George syndrome
e) Chronic granulomatous disease
15. Triad of immunodeficiency, thrombocytopaenia and eczema
a) Duncan's syndrome
b) Ataxia telangiectasia
c) Chronic granulomatous disease
d) Wiskott-Aldrich syndrome
e) Di George syndrome
16. A severe, life threatening, generalised or systemic hypersensitivity reaction
a) Intolerance
b) Anaphylaxis
c) Atopy
d) Allergy
e) Hypersensitivity
17. Peanut allergy affects 1 in ___ children (according to Lissauer, 2007)
a) 300
b) 500
c) 1000
d) 70
e) 120
18. Objectively reproducible symptoms or signs following a defined stimulus at a dose tolerated by
normal persons
a) Allergy
b) Anaphylaxis
c) Atopy
d) Hypersensitivity
e) Intolerance
19. Usually due to exposure to the house dust mite or a pet
a) eczema
b) anaphylaxis
c) asthma
d) urticaria
e) perennial allergic rhinitis
20. A personal and/or familial tendency, usually in childhood or adolescence, to become sensitised and
produce IgE antibodies in response to ordinary exposure to antigens.
a) Atopy
b) Intolerance
c) Allergy
d) Hypersensitivity
e) Anaphylaxis
21. Cause susceptibility to bacterial infections, esp. Neisseria meningitidis. May also lead to
autoimmune diseases such as SLE
a) Severe combined immunodeficiency
b) Opsonisation defects
c) Defects of bacterial phagocytic function
d) Major immunoglobulin deficiencies
e) Defects in leucocyte function
22. An inherited disorder where phagoctyic cells fail to produce superoxide after ingestion of micro-
organisms due to a lesion in a membrane associated NADPH oxidase.
a) Chronic granulomatous disease
b) Di George syndrome
c) Duncan's syndrome
d) Wiskott-Aldrich syndrome
e) Ataxia telangiectasia
23. Test for leucocyte adhesion deficiency
a) tests of chemotaxis
b) Nitroblue tetrazolium test
c) Chromosomal fragility test
d) Surface adhesion molecules (CD18, CD11b)
e) total haemolytic compliment
24. Defects which lead to this include purine metabolism abnormalities, abnormal lymphocyte
maturation and IL-2 receptor gamma chain deficiency
a) Defects of bacterial phagocytic function
b) Major immunoglobulin deficiencies
c) Opsonisation defects
d) Severe combined immunodeficiency
e) Defects in leucocyte function
25. Test for ataxia telangiectasia
a) tests of chemotaxis
b) Chromosomal fragility test
c) Nitroblue tetrazolium test
d) Full blood count
e) total haemolytic compliment
26. A heterogenous group of inherited disorders characterised by profoundly defective cellular and
humoral immunity. Presents in the first 6 months with unusual and severe failure to thrive.
a) Opsonisation defects
b) Defects of bacterial phagocytic function
c) Major immunoglobulin deficiencies
d) Defects in leucocyte function
e) Severe combined immunodeficiency
27. Used to examine the morpholoy of cells
a) Full blood count
b) Nitroblue tetrazolium test
c) tests of chemotaxis
d) total haemolytic compliment
e) Blood film
28. Usually non allergic in origin
a) Rhinitis
b) Eczema
c) Chronic urticaria
d) Food hypersensitivity
e) Asthma
29. The most common type is caused by abnormalities in the Bruton tyrosine kinase gene
a) Severe combined immunodeficiency
b) Defects in leucocyte function
c) Major immunoglobulin deficiencies
d) Opsonisation defects
e) Defects of bacterial phagocytic function
30. What percentage of 10 year olds are not dry at night?
a) 3%
b) 6%
c) 1%
d) 24%
e) 12%
31. Breakdown in the appreciation of reality and a lack of insight that anything is wrong
a) mania
b) oppositional defiant disorder
c) depression
d) psychosis
e) conduct disorder
32. In general, the further the pain is from the umbilicus, the more likely it is to be caused by organic
pathology
a) Able's rule
b) Apple's rule
c) Appleby's rule
d) Abelson's rule
e) Apley's rule
33. Serious antisocial behaviour that infringes on the rights of others and represents a handicap to
general functioning may suggest
a) depression
b) oppositional defiant disorder
c) psychosis
d) conduct disorder
e) mania
34. Peak age of onset of anorexia nervosa is ___ years
a) 13
b) 17
c) 16
d) 14
e) 15
35. A psychotic disorder where no specific medical cause is identified and there is generally no major
disturbance of mood other than blunting or flattening of affect
a) conduct disorder
b) schizophrenia
c) bipolar affective disorder
d) oppositional defiant disorder
e) delerium
36. What percentage of 5 year olds are not dry at night?
a) 36%
b) 12%
c) 3%
d) 6%
e) 24%
37. Faecal soiling is abnormal after the age of ___ years
a) 5
b) 3
c) 7
d) 4
e) 6
38. Angry defiant behaviour to authority figures
a) depression
b) conduct disorder
c) oppositional defiant disorder
d) mania
e) psychosis
39. An autosomal disorder or connective tissue associated with tall stature, long thin digits,
hyperextensible joints, a high arched palate, dislocation of the lenses of the eyes and severe myopia
a) Thanatophoric dysplasia
b) Achondroplasia
c) Arthrogryposis
d) Marfan's syndrome
e) Osteogenesis imperfecta
40. Below is a list of conditions which may present with musculoskeletal clinical features. Which is the
most common (in children)?
a)septic arthritis
b)kawasaki's disease
c)osteomyelitis
d)reactive arthritis
e)juvenile idiopathic arthritis
41. The normal age range for presentation of out-toeing is
a)1-6 months
b)1-3 years
c)12-24 months
d)6-12 months
e)2-7 years
42. Most often affects adolescent females, causing pain when the patella is tightly opposed to the
femoral condyles, eg walking up stairs
a) Osteochondritis dissecans
b) Osgood-Schlatter disease
c) Subluxation and dislocation of the patella
d) Slipped upper femoral epophysis
e) Chondromalacia patellae
43. Stress fracture of the pars interarticularis of the vertebra, typically lower lumbar
a) Scoliosis
b) Scheuermann's disease
c) Torticollis
d) Vertebral osteomyelitis
e) Spondylolysis
44. A complex abnormality where the entire foot is inverted and supinated and the forefoot is
adducted. The heel is rotated inwards in plantarflexion. The affected foot is shorter and the calf
muscles thinner than normal. The posiion of the foot is fixed and can not be corrected completely.
a) Osgood-Schlatter disease
b) talipes equinovarus
c) pes cavus
e) positional talipes
f) talipes calcaneovalgus
45. When bilateral, this is due to lateral rotation of the hips and resolves spontaneously
a) genu varum
b) out-toeing
c) toe walking
d) pes planus
e) in-toeing
46. Differential diagnoses ot consider would be Hypermobility or congenital tarsal fusion
a) Flat feet (pes planus)
b) Knock knees (genu valgum)
c) In-toeing
d)Toe walking
e) Bow legs (genu varum)
47. The most common form of arthritis in childhood
a) juveline idiopathic arthritis
b) septic arthritis
c) osteomyelitis
d) psoriatic arthritis
e) reactive arthritis
48. Caused by infection of the metaphysis of long bones. Most common sites are are distal femur and
proximal tibia. Usually due to haematogenous spread of a pathogen.
a) Juvenile idiopathic arthritis
b) Septic arthritis
c) Arthrogryposis
d) Osteomyelitis
e) Juvenile dermatomyositis
49. In this autosomal dominant disordre there is absence of part or all of the clavicles and delay in
closure of the anterior fontanelle
a) Achondroplasia
b) Arthrogryposis
c) Cleidocranial dystosis
d) Thanatophoric dysplasia
e) Osteopetrosis
50. In this rare disorder, the bones are dense but brittle. Presents with failure to thrive, recurrent
infection, hypocalcaemia and thrombocytopaenia
a) Cleidocranial dystosis
b) Osteopetrosis
c) Arthrogryposis
d) Achondroplasia
e) Thanatophoric dysplasia
51. Management is surgical, usually with pin fixation in situ. Severe cases may require subsequent
corrective realignment osteotomy
a) Juvenile idiopathic arthritis
b) Perthes disease
c) Transient synovitis
d) Slipped upper femoral epophysis
e) Septic arthritis
52. Laboratory abnormalities include HLA B27+. Localised inflammation at insertions of tendons or
ligaments into bone
a) Enthesitis related arthritis
b) Polyarthritis
c) Psoriatic arthritis
d) Oligoarthritis
e) Systemic arthritis
53. Differential diagnoses ot consider would be tibial torsion or femoral anteversion
a) Bow legs (genu varum)
b) Flat feet (pes planus)
c) Knock knees (genu valgum)
d) Toe walking
e) In-toeing
54. A heterogenous group of congenital disorders in which there is stiffness and contracture of joints.
May be an association with oligohydramnios.
a) Osteopetrosis
b) Arthrogryposis
c) Cleidocranial dystosis
d) Achondroplasia
e) Thanatophoric dysplasia
55. The foot is dorsiflexed and everted. It usually results from intrauterine moulding and self corrects.
a) talipes calcaneovalgus
b) pes cavus
c) talipes equinovarus
d) Osgood-Schlatter disease
e) positional talipes
56. Can result from Ehlers-Danlos syndrome. Some will develop a prominence of the navicular boine
on the medial aspect of the foot.
a) developmental dysplasia of the hip
b) Perthes disease
c) genu valgum
d) pes planus
e) genu varum
57. Usually begins insidiously with malaise, progressive weakness and facial rash with erythema over
the bridge of the nose and malar areas and a violaceous discolouration of the eyelids
a) Henoch-Schonlein purpura
b) Systemic lupus erythematosus
c) Juvenile dermatomyositis
d) Arthrogryposis
e) Achondroplasia
58. The most common vasculitis of childhood
a) Henoch-Schonlein purpura
b) Achondroplasia
c) Arthrogryposis
d) Juvenile dermatomyositis
e) Systemic lupus erythematosus
59). An overuse syndrome commonly occuring in physically active males around puberty, resulting in
a partial avulsion fracture through the ossification centre of the tibial tuberosity
a) Subluxation and dislocation of the patella
b) Osgood-Schlatter disease
c) Slipped upper femoral epophysis
d) Osteochondritis dissecans
e) Chondromalacia patellae
60. There is a high arched foot. In older children it may be associated with Friedrich's ataxia and type
I hereditary motor sensory neuropathy.
a) pes cavus
b) positional talipes
c) talipes equinovarus
d) Osgood-Schlatter disease
e) talipes calcaneovalgus
61. Often follows or is accompanied by a viral infection. Presentation is with a sudden onset of pain in
the hip or a limp. The child is afebrile or a has a mild fever.
a) Perthes disease
b) Septic arthritis
c) Juvenile idiopathic arthritis
d) Cerebral palsy
e) Transient synovitis
62. Usually due to flatness of the medical longitudinal arch and the presence of a fat pad
a) genu varum
b) genu valgum
c) Perthes disease
d) pes planus
e) developmental dysplasia of the hip
63. The most common cause of acute hip pain in children
a) Cerebral palsy
b) Perthes disease
c) Juvenile idiopathic arthritis
d) Septic arthritis
e) Transient synovitis
64. A benign tumor affecting adolescents, especially boys, usually involving the femur or tibia
a) Osteogenic sarcoma
b) Ewing's tumour
c) Arthrogryposis
d) Osteomyelitis
e) Osteoid osteoma
65. One cause of this is Rickets, another is Blount's disease
a) genu varum
b) pes planus
c) Perthes disease
d) genu valgum
e) developmental dysplasia of the hip
66. The most common cause in infants is a sternomastoid tumour
a) Vertebral osteomyelitis
b) Scheuermann's disease
c) Scoliosis
d) Spondylolysis
e) Torticollis
67. In this condition the feet are wide apart when standing with the knees held together
a) Perthes disease
b) genu valgum
c) developmental dysplasia of the hip
d) pes planus
e) genu varum
68. Pain is caused by separation of bone and cartilage from the medial femoral condyle following
avascular necrosis
a) Osgood-Schlatter disease
b) Subluxation and dislocation of the patella
c) Osteochondritis dissecans
d) Chondromalacia patellae
e) Slipped upper femoral epophysis
69. Associated with "W" sitting
a) Perthe's disease
b) metatarsus varus
c) persistent anteversion of the femoral neck
d) transient synovitis
e) medial tibial torsion
70. Bowing of the tibiae causing the knees to be wide apart while standing with the feet together
a) developmental dysplasia of the hip
b) pes planus
c) Perthes disease
d) genu varum
e) genu valgum
71. The true birth prevalence of DDH is about ___ per 1000 live births
a) 0.1
b) 3.0
c) 0.5
d) 1.5
e) 5.5
72. Lateral curvature in the frontal plane of the spine
a) Scheuermann's disease
b) Spondylolysis
c) Scoliosis
d) Vertebral osteomyelitis
e) Torticollis
73. Seen in children between 2 and 7 years, resulting in an intermalleolar distance at the ankles of up
to 5cm
a) pes planus
b) genu valgum
c) genu varum
d) developmental dysplasia of the hip
e) Perthes disease
74. Clinical features are short stature from marked shortening of the limbs, a large head, frontal
bossing and depression of the nasal bridge
a) Henoch-Schonlein purpura
b) Achondroplasia
c) Arthrogryposis
d) Juvenile dermatomyositis
e) Systemic lupus erythematosus
75. Results in stillbirth. Inheritance is sporadic. Large head, extremely short limbs and small chest.
a) Thanatophoric dysplasia
b) Cleidocranial dystosis
c) Achondroplasia
d) Osteopetrosis
e) Arthrogryposis
76. Used to treat developmental dysplasia. Holds the hip abducted, allowing the hip joint to develop
normally
a) Rosberg splint
b) Craig splint
c) Effeick's splint
d) Pavlik harness
e) Marlow harness
77. Differential diagnoses to consider would be Rickets, osteogenesis imperfecta, Blount's disease
a) knees (genu valgum)
b) Bow legs (genu varum)
c) Flat feet (pes planus)
d) Out-toeing
e) In-toeing
78. May be due to mild cerebral palsy, isolated tightness of the achilles tendon, or Duchenne's
muscular dystrophy
a) in-toeing
b) genu varum
c) toe walking
d) pes planus
e) out-toeing
79. Associated with fever, pain at rest and minimal or no movement of the affected joint.
a) Septic arthritis
b) Perthes disease
c) Transient synovitis
d) Cerebral palsy
e)Juvenile idiopathic arthritis
80. Associated with a blue appearance to the sclera
a) Osteogenesis imperfecta
b) Cleidocranial dystosis
c) Achondroplasia
d) Thanatophoric dysplasia
e) Arthrogryposis
81. Due to ischaemia of the femoral epiphysis, resulting in avascular necrosis, followed by
revascularisation and reossification over 18-36 months
a) Transient synovitis
b) Perthes disease
c) Cerebral palsy
d) Juvenile idiopathic arthritis
e) Septic arthritis
82. Differential diagnoses to consider would be hypermobility, Ehlers-Danlos or Marfan's syndrome
a) In-toeing
b) Out-toeing
c) Knock knees (genu valgum)
d) Bow legs (genu varum)
e) Flat feet (pes planus)
83. Differential diagnoses to consider would be spastic diplegia or muscular dystrophy
a) Knock knees (genu valgum)
b) In-toeing
c) Bow legs (genu varum)
d) Flat feet (pes planus)
e) Toe walking
84. Checking if the hip can be dislocated posteriorly out of the acetabulum in a newborn
a) Gower's test
b) Barlow's manoeuvre
c) Collinger's test
d) Ortolani's manoeuvre
e) Enthsitis
85. Presents with fever, a painful immobile limb, swelling and extreme tenderness. Blood cultures
usually positive. Parenteral antibiotics must be given immediately.
a) Juvenile idiopathic arthritis
b) Osteomyelitis
c) Juvenile dermatomyositis
d) Arthrogryposis
e) Reactive arthritis
86. Three main causes are: metatarsus varus, medial tibial torsion, persistent anteversion of the
femoral neck
a) genu varum
b) out-toeing
c) toe walking
d) pes planus
e) in-toeing
87. An osteochondritis of the thoracic vertebrae in adolescents resulting in a fixed kyphosis
a) Spondylolysis
b) Torticollis
c) Vertebral osteomyelitis
d) Scheuermann's disease
e) Scoliosis

Key test 3
1. B
2. A
3. B
4. D
5. E
6. B
7. A
8. A
9. E
10. B
11. C
12. A
13. C
14. D
15. D
16. B
17. D
18. D
19. E
20. A
21. B
22. A
23. D
24. D
25. B
26. E
27. E
28. C
29. C
30. A
31. D
32. E
33. D
34. D
35. B
36. D
37. D
38. C
39. D
40. D
41. D
42. E
43. E
44. B
45. B
46. A
47. E
48. D
49. C
50. B
51. D
52. A
53. E
54. B
55. A
56. D
57. C
58. A
59. B
60. A
61. E
62. D
63. E
64. E
65. A
66. E
67. B
68. C
69. C
70. D
71. D
72. C
73. B
74. B
75. A
76. D
77. B
78. C
79. A
80. A
81. B
82. B
83. E
84. B
85. B
86. E
87. D

Test 4
1. Babbles a lot, speaks a few words and understands simple commands by
a)9 months
b) 18 months
c) 12 months
d) 2 years
e) 3 years
2. Co-operates with dressing, e.g. holding up arms, and waves bye bye by
a)3 years
b) 18 months
c) 2 years
d) 9 months
e) 12 months
3. Can sit briefly on a firm, flat surface, often using arms for support
A. 7 months
B. 4 months
C. 12-15 months
D. 9 months
E. 10 months
4. Speaks in sentences. Gives full name. Eats with spoon and fork by
A. 12 months
B. 18 months
C. 4 years
D. 2 years
E. 3 years
5. Speech: consonants (goo, gah) - gurgling by
A. 2 years
B. 9 months
C. 1 year
D. 6 months
E. 3 months
6. Can sit briefly on a firm, flat surface, often using arms for support
A. 9 months
B. 4 months
C. 12-15 months
D. 10 months
E. 7 months
7. Speech: sentences used to describe past and present happenings
A. 3 years
B. 6 months
C. 9 months
D. 2 years
E. 1 year
8. Can feed self efficiently with spoon and fork
A. 3 years
B. 2 years
C. 9 months
D. 1 year
E. 4 years
9. Speech: varied and tuneful babling
A. 6 months
B. 9 months
C. 1 year
D. 3 months
E. 2 years
10. Can copy a circle by
A. 2.5 years
B. 2 years
C. 4 years
D. 3 years
E. 5 years
11. Uses many words, sound labels, occasionally two words together by
A. 2 years
B. 3 years
C. 9 months
D. 12 months
E. 18 months
12. Speech: words joined to convey ideas - eg ‘dada gone’ - child follows simple instructions
A. 6 months
B. 3 months
C. 2 years
D. 1 year
E. 9 months
13. Observes with a convergent gaze a dangling toy or bright object held 20-30cm from his/her face and
moves his/her head and neck in order to follow it by...
A. 6 weeks
B. 4 weeks
C. 10 week
D. 2 weeks
E. 8 weeks
14. Can pull him/herself up to stand and begins to cruise (walk holding on)
A. 10 months
B. 9 months
C. 12-15 months
D. 4 months
E. 7 months
15. Control of posture and movement is acquired…
A. From the hands inwards
B. From the feet upwards
C. From the head downwards
D. From the trunk outwards
16. Similing is seen at ___ in response to mothers face
A. 12-16 weeks
B. 4-8 weeks
C. 8-10 weeks
D. 10-12 weeks
E. 2-3 weeks
17. There is a clumsy palmar grasp - objects are approached with the ulnar border of the hand by
A. 6 months
B. 2 years
C. 18 months
D. 9 months
E. 12 months
18. The optimal age at which to test an infant’s hearing is
A. 5 months
B. 7 months
C. 9 months
D. 3 months
E. 12 months
19. Up to age ___ most babies will be handled happily by anyone
A. 9 months
B. 12 months
C. 3 months
D. 16 months
E. 6 months
20. Can build a tower of 9 wooden cubes by
A. 5 years
B. 3 years
C. 2 years
D. 2.5 years
E. 4 years
21. Speech: single word labels used for familiar objects and people
A. 6 months
B. 2 years
C. 3 years
D. 9 months
E. 1 year
22. Drinks form a cup using two hands by
A. 12 months
B. 3 years
C. 18 months
D. 9 months
E. 2 years
23. Objects are approached with the index finger and picked up precisely between the ends of the thumb
and finger in a pincer grasp by
A. 9 months
B. 2 years
C. 12 months
D. 18 months
E. 6 months
24. Fluent speech with few infantile substitutions by
A. 5 years
B. 2 years
C. 6 years
D. 4 years
E. 3 years
25. Can sit without arm support and can turn without falling
A. 9 months
B. 10 months
C. 4 months
D. 12-15 months
E. 7 months
26. There is a scissor grasp between the sides of the thumb and index finger and objects are approached
with the radial border of the hand by
A. 18 months
B. 9 months
C. 12 months
D. 2 years
E. 6 months
27. Can skip, hop and stand on one foot with arms folded
A. 6 years
B. 5 years
C. 4 years
D. 2 years
E. 3 years
28. Joins words together in simple phrases. Dry by day. Play imitates adult activities by
A. 3 years
B. 12 months
C. 9 months
D. 2 years
E. 18 months
29. Speech: open vowel sounds (ooh, eeh ) & cooing by
A. 6 months
B. 9 months
C. 3 months
D. 2 years
E. 1 year
30. Can walk unaided
A. 4 months
B. 9 months
C. 12-15 months
D. 7 months
E. 10 months
31. Which of the following is NOT one of the four main areas of development?
A. Gross motor
B. Language
C. Personal/Social
D. Fine motor
E. Sexual
32. Developmental testing ___ is less reliable than at any other time
A. 1 - 2 years
B. 6-10 months
C. 14-18 months
D. 10-14 months
E. Before 6 months
33. Can build a tower of 3 wooden cubes by
A. 2 years
B. 3 years
C. 4 years
D. 2.5 years
E. 1.5 years
34. Can copy a straight line by
A. 2 years
B. 3 years
C. 1.5 years
D. 2.5 years
E. 4 years
35. Can copy a cross by
A. 5 years
B. 2 years
C. 3 years
D. 2.5 years
E. 4 years
36. Can kick a ball, draw a circle, ride a trike, knows three body parts and three colours, and speaks in at
least 3 word sentences by age
A. 2 years
B. 2.5 years
C. 3 years
D. 4 years
E. 1.5 years
37. Can build a tower of 6 wooden cubes by
A. 1.5 years
B. 2.5 years
C. 2 years
D. 4 years
E. 3 years
38. Can copy a triangle by
A. 2 years
B. 2.5 years
C. 3 years
D. 4 years
E. 5 years

Key test 4

1. c
2. e
3. a
4. e
5. d
6. e
7. a
8. a
9. b
10. d
11. e
12. c
13. e
14. a
15. d
16. b
17. a
18. b
19. a
20. b
21. e
22. c
23. c
24. a
25. a
26. b
27. b
28. c
29. c
30. c
31. e
32. e
33. e
34. a
35. e
36. c
37. c
38. e

Test 5
1. Misalignment of the visual axes

a. Hypermetropia
b.Strabismus
c. Myopia
d.Retinoblastoma
e. Amblyopia

2. Characterized by sustained muscle contractions, repetitive twisting movements, and abnormal posturing of
the trunk, neck, face, or arms and legs

a. Hypertonia
b. Dystonia
c. Atonia
d. Athetosis
e. Chorea

3. What percentage of children with cerebral palsy also have visual impairment?

a. 30%
b. 10%
c. 60%
d. 40%
e. 20%
4. There is fluctuating tone, leading tot frequent involuntary

movement (generally of all four limbs) especially evident with movement or stress.

a. Ataxic hypotonic cerebral palsy


b. Dyskinetic cerebral palsy
c. Dystaxic cerebral palsy
d. Spastic cerebral palsy
e. Atonic cerebral palsy

5. Repetitive involuntary, slow, sinuous, writhing movements, which are especially severe in the hands. There
are also elements of postural disturbance.

a. Chorea
b. Atonia
c. Dystonia
d. Athetosis
e. Hypertonia

6. Irregular, sudden and brief non repetitive movements

a. Dystonia
b. Atonia
c. Chorea
d. Hypertonia
e. Athetosis

7. Pulls to standing - limit age?

a. 13 months
b. 6 months
c. 9 months
d. 3 months
e. 18 months

8. Characterised by three features; micrognathia, glossoptosis and cleft palate

a. Freidreich’s syndrome
b. Noonan’s syndrome
c. Pierre-Robin sequence
d. Asperger’s syndrome
e. ADD

9. Severe visual impairment affects 1 in _______ live births in the UK

a. 3000
b. 500
c. 2000
d. 1000
e. 5000

10. Hand dominance is acquired around

a. 8-10 months
b. 3-4 years
c. 6-8 months
d. 12-14 months
e. 1-2 years
11. What percentage of children with cerebral palsy also have learning difficulties?

a. 20%
b. 30%
c. 10%
d. 60%
e. 40%

12. An acquired deficit in the comprehension or production of language whether spoken or written.

a. Dysarthria
b. Dysphasia
c. Dyspraxia
d. Aphasia
e. Dysfluency

13. The most common refractive error in young children and should be corrected early to avoid irreversible
damage to vision

a. Myopia
b. Amblyopia
c. Strabismus
d. Retinal dystrophy
e. Hypermetropia

14. Also known as short sightedness

a. Strabismus
b. Myopia
c. Retinal dystrophy
d. Amblyopia
e. Hypermetropia

15. A norm-based practical tool including two scales - Comprehension and Expressive. Contain 62 items
focusing on the structural aspects of language and how they contribute to its acquisition and use, and examine
disorders that can occur in either area.

a. Raymond Developmental Language Scales


b. Raynauld Developmental Language Scales
c. Rainman Developmental Language Scales
d. Reynell Developmental Language Scales
e. Ronald Developmental Language Scales

16. General features of this type of cerebral palsy are brisk tendon reflexes and extensor plantar responses.

a. Spastic cerebral palsy


b. Ataxic hypotonic cerebral palsy
c. Dyskinetic cerebral palsy
d. Atonic cerebral palsy
e. Dystaxic cerebral palsy

17. What percentage of cases of cerebral palsy are thought to be due to hypoxic-ischaemic injury at birth?

a. 30%
b. 40%
c. 10%
d. 20%
e. 60%
18. A mild learning difficulty is associated with an IQ of

a. 70-90
b. < 20
c. 50-70
d. 20-50
e. 90-110

19. The prevalence of autistic spectrum disorders is _________ live births

a. 10-12/1000
b. 3-6/1000
c. 1-2/1000
d. 6-10/1000
e. 2-4/10000

20. This form of cerebral palsy is often associated with seizures, microcephaly and moderate or severe
intellectual impairment

a. Diplegia
b. Quadriplegia
c. Hemiplegia
d. Biplegia
e. Triplegia

21. Children with Down’s syndrome, cleft palate and atopy are particularly prone to

a. Hearing loss form middle ear disease


b. Meningitis
c. Sensorineural hearing loss.
d. B&C
e. All of these

22. Independent standing or walking - limit age?

a. 9 months
b. 18 months
c. 13 months
d. 3 months
e. 6 months

23. Loss or abnormality of physiological function or anatomical structure

a. Impairment
b. Delay
c. Handicap
d. Disorder
e. Disability

24. Less than ___ of children with autism are able to function independently as adults

a. 50%
b. 25%
c. 10%
d. 20%
e. 15%

25. Sits without support, arms free to reach and grasp - limit age?
a. 13 months
b. 18 months
c. 3 months
d. 6 months
e. 9 months

26. Slow acquisition of all skills or of one particular field or area of skill, particularly in relation to
developmental problems in the 0-5 year age group

a. Impairment
b. Delay
c. Disorder
d. Disability
e. Handicap

27. What percentage of children with cerebral palsy also have epilepsy?

a. 10%
b. 40%
c. 20%
d. 30%
e. 60%

28. Which of the following are possible post natal causes of cerebral palsy?

a. Meningitis
b. Symptomatic hypoglycaemia
c. Head trauma
d. All of these
e. Encephalitis

29. What percentage of children with cerebral palsy also have squints?

a. 40%
b. 30%
c. 20%
d. 60%
e. 10%

30. Treatment involves correction of any refractive error with glasses, together with patching of the good eye
for specific periods of the day

a. Myopia
b. Amblyopia
c. Retinal dystrophy
d. Hypermetropia
e. Strabismus

31. About ____% of severe visual impairment is genetic

a. 20
b. 90
c. 40
d. 5
e. 50

32. A disadvantage from a disability which limits or prevents fulfilment of a normal role
a. Delay
b. Handicap
c. Impairment
d. Disorder
e. Disability

33. Cerebral palsy affects around ______ live births

a. 2 per 100
b. 2 per 3000
c. 2 per 500
d. 2 per 1000
e. 2 per 5000

34. Said to be the 2nd most common heritable cause of mental retardation. Caused by a dominant X-linked
gene with a penetrance of only 50% in females. The gene which is most commonly responsible is FMR-1

a. Downs syndrome
b. Reiter’s syndrome
c. Fragile X syndrome
d. Turners syndrome
e. Noonan’s syndrome

35. Pushes up on arms, holds head up - limit age?

a. 18 months
b. 13 months
c. 3 months
d. 9 months
e. 6 months

36. Measures the air pressure within the middle ear and the compliance of the tympanic membrane

a. Otoacoustic emission
b. Auditory response cradle test
c. Impedance audiometry
d. Speech discrimination test
e. Auditory brainstem evoked potential

37. The legs are affected to a much greater degree than the arms, so that hand function may appear to be
relatively normal.

a. Quadriplegia
b.Biplegia
c. Triplegia
d.Diplegia
e. Hemiplegia

38. A disorder of movement and posture which appears before age three and is due to non-progressive damage
to the brain

a. Dyspraxia
b.Cerebral palsy
c. Epilepsy
d.Congenital myopathy
e. Spina bifida

39. A potentially permanent loss of visual acuity in an eye that has not yet received a clear image.
a. Amblyopia
b.Hypermetropia
c. Strabismus
d.Retinal dystrophy
e. Myopia

40. A severe learning difficulty is associated with an IQ of

a. 50-70
b.< 20
c. 70-90
d.90-110
e. 20-50

41. Maldevelopment of a skill

a. Delay
b.Disability
c. Handicap
d.Disorder
e. Impairment

42. Sits with head support, holds head up, rounded back - limit age?

a. 9 months
b.18 months
c. 6 months
d.13 months

e. 3 months

43. A moderate learning difficulty is associated with an IQ of

a. 20-50
b.50-70
c. < 20
d.90-110
e. 70-90

44. Difficulty in performing a series of complex actions

a. Dysfluency
b.Dysphasia
c. Aphasia
d.Dysarthria
e. Dyspraxia

45. Any type of speech which is marked with repetitions, prolongation, and hesitations; an interruption in the
flow of speech sounds

a. Dysphasia
b.Aphasia
c. Dysarthria
d.Dyspraxia
e. Dysfluency

46. Sometimes known as a lazy eye

a. Myopia
b.Strabismus
c. Retinal dystrophy
d.Hypermetropia
e. Amblyopia

47. A rare type of strabismus due to paralysis of motor nerves

a. Paplexic
b.Paralytic
c. Commisural
d.Ambylopia
e. Concomitant

48. Affected children often present at 4-12 months of age with fisting of the affected hand, a flexed arm, a
pronated forearm, asymmetric reaching or hand function.

a. Triplegia
b.Quadriplegia
c. Diplegia
d.Hemiplegia
e. Biplegia

49. In the cover test, when a squint is present and the fixing eye is covered, the squinting eye..

a. Displays nystagmus
b.Moves inwards
c. Does not move
d.Moves outwards
e. Moves to take up fixation

50. What percentage of cases of cerebral palsy are thought to be post natal in origin?

a. 20%
b.60%
c. 30%
d.10%
e. 40%

51. What percentage of children with cerebral palsy also have hearing impairment?

a. 60%
b.10%
c. 30%
d.40%
e. 20%

52. Signs are relatively symmetrical. There is early trunk and limb hypotonia, poor balance and delayed motor
movement. Intention tremor and incordinate movements may be evident later.

a. Spastic cerebral palsy


b.Atonic cerebral palsy
c. Ataxic hypotonic cerebral palsy
d.Dyskinetic cerebral palsy
e. Dystaxic cerebral palsy

53. Any restriction or lack of ability due to an impairment

a. Delay
b.Disorder
c. Handicap
d.Impairment
e. Disability

54. What percentage of cases of cerebral palsy are spastic?

a. 10
b.40
c. 70
d.90
e. 20

55. Also known as long sightedness

a. Amblyopia
b.Strabismus
c. Myopia
d.Hypermetropia
e. Retinal dystrophy

56. A complete loss of language comprehension or production

a. Dysfluency
b.Aphasia
c. Dyspraxia
d.Dysphasia
e. Dysarthria

57. A disorder of motor planning and/or execution with no significant findings on standard neurological
examination

a. Dyspraxia
b.Dyscalculia
c. Dysgraphia
d.Dyskinesia
e. Dyslexia

58. The most common cause of motor impairment in children

a. Epilepsy
b.Dyspraxia
c. Spina bifida
d.Congenital myopathy
e. Cerebral palsy

59. Affected children often present with floppiness, poor trunk control and delayed motor development in
infancy, with abnormal movements sometimes not appearing before the age of 1.

a. Spastic cerebral palsy


b.Ataxic hypotonic cerebral palsy
c. Dyskinetic cerebral palsy
d.Dystaxic cerebral palsy
e. Atonic cerebral palsy

60. A profound learning difficulty is associated with an IQ of

a. 20-50
b.90-110
c. 70-90
d.< 20
e. 50-70

61. Refers to a child with the social impairments of an autistic spectrum disorder but at the milder end, and
near normal speech development.

a. Pierre-Robin sequence
b.Asperger’s syndrome
c. Noonan’s syndrome
d.Freidreich’s syndrome
e. ADD

62. A difficulty in articulation - there is no disorder of the content of speech. The most common cause is
alcohol intoxication

a. Dysphasia
b.Dysarthria
c. Dyspraxia
d.Dysfluency
e. Aphasia
Key test 5

1.B
2.B
3.E
4.B
5.D
6.C
7.A
8.C
9.D
10. E
11. D
12. B
13. E
14. B
15. D
16. A
17. C
18. A
19. B
20. B
21. A
22. B
23. A
24. A
25. E
26. B
27. B
28. D
29. B
30. B
31. E
32. B
33. D
34. C
35. C
36. C
37. D
38. B
39. A
40. E
41. D
42. C
43. B
44. E
45. E
46. E
47. B
48. D
49. E
50. D
51. E
52. C
53. E
54. C
55. D
56. B
57. A
58. E
59. C
60. D
61. B
62. B

Test 6
1. A small hard mass of faeces, found particularly in the vermiform appendix: a cause of inflammation.

a. faecolith

b. faecollum

c. faecostoid

d. faecoloid
e. faecolus

2. Which of the following has NOT been implicated as a viral cause of gastroenteritis?

a. adenovirus

b. astrovirus

c. coronavirus

d. e.coli

e. calcivirus

3. Specific zinc malabsorption is a feature of this

a. Gastroenteritis

b. Intermittent interspersion

c. Irritable bowel syndrome

d. Enterocytosis

e. Acrodermatitis enteropathica

4. Associated with absent bile acid and vitamin B12 absorption

a. Loss of terminal ileal function

b. Exocrine pancreatic dysfunction

c. Lymphatic leakage or obstruction

d. Cholestatic liver disease or biliary atresia

e. Short bowel syndrome

5. Which of the following is NOT suggestive of pyloric stenosis

a. constant hunger even after vomiting

b. visible gastric peristalsis

c. weight loss or poor weight gain

d. hyperchloraemic alkalosis with low plasma potassium

e. bile stained vomiting

6. Children classically present in the fist few years of life with failure to thrive following the introduction of
cereals

a. Gastroenteritis

b. Acrodermatitis enteropathica

c. Coeliac disease
d. Irritable bowel syndrome

e. Chron's disease

7. The ratio of diagnosed pyloric stenosis is; males:females

a. 8:1

b. 2:1

c. 1:4

d. 1:2

e. 4:1

8. Diagnosis is made by suction rectal biopsy

a. Ulcerative Colitis

b. Coeliac disease

c. Hirschsprung's disease

d. Crohn's disease

e. Cystic fibrosis

9. Affects any part of the intestinal tract from the mouth to the anus

a. IBS

b. Cystic fibrosis

c. Ulcerative Colitis

d. Coeliac disease

e. Crohn's disease

10. May lead to transient hyperglycaemia, and multiple, small cerebral haemmorhages and convulsions

a. mild dehydration

b. asymptomatic dehydration

c. hyponatraemic dehydration

d. isonatraemic dehydration

e. hypernatraemic dehydration

11. Most commonly treated using air insufflation

a. Appendicitis

b. Faecoliths

c. Intussusception

d. Volvulus
e. Mesenteric adenitis

12. The incidence of coelic in children with type 1 diabetes is around

a. 5%

b. 0.5%

c. 0.1%

d. 10%

e. 1%

13. Vomiting at the end of paroxysmal coughing would be suggestive of

a. lower intestinal obstruction

b. pertussis

c. oesophagitis or peptic ulceration

d. small bowel obstruction

e. pyloric stenosis

14. Acute appendicitis is uncommon in children under the age of ___ years

a. 5

b. 7

c. 3

d. 11

e. 9

15. One third of patients require colectomy during the course of the disease

a. Cystic fibrosis

b. Crohn's disease

c. IBS

d. Coeliac

e. Ulcerative Colitis

16. Azathioprine may maintain remission. Infliximab may be needed as a third line agent.

a. Crohn's disease

b. Cystic fibrosis

c. Coeliac disease

d. Gastroenteritis

e. IBS
17. Projective vomiting in the first few weeks of life would prompt investigations for

a. oesophagitis or peptic ulceration

b. small bowel obstruction

c. pyloric stenosis

d. pertussis

e. lower intestinal obstruction

18. An operation where the fundus of the stomach is wrapped around the intra-abdominal oesophagus

a. Nestargotomy

b. Carobelotomy

c. Obfcusation

d. Fundoplication

e. Oesophagation

19. When treating hypernatraemic dehydration, the reduction in plasma sodium should not exceed

a. 50 mmol/L/24 hr

b. 100 mmol/L/24 hr

c. 1.0 mmol/L/24 hr

d. 5.0 mmol/L/24 hr

e. 10.0 mmol/L/24 hr

20. The most common 'surgical cause' of abdominal pain

a. Meckel's diverticulum

b. intussusception

c. acute appendicitis

d. peritonitis

e. inguinal hernia

21. The histological hallmark is the presence of non- caseating epithelioid cell granulomata

a. IBS

b. Crohn's disease

c. Coeliac disease

d. Cystic fibrosis

e. Ulcerative Colitis
22. Rare congenital malformations of the gastrointestinal tract where there is formation of a second tract of
bowel, either as a tube running parallel to the normal bowel, or as a series of cysts. They tend to form in the
small intestine.

a. Duplications+

b. Volvulus

c. Malrotations

d. Intussusceptions

e. Sandifers

23. General irritability, abnormal stools (foul smelling), abdominal distention and buttock wasting are the
usual symptoms.

a. Irritable bowel syndrome

b. Coeliac disease

c. Gastroenteritis

d. Acrodermatitis enteropathica

e. Chron's disease

24. Initial management of pyloric stenosis

a. barium meal

b. correct fluid and electrolyte disturbance

c. Pyloromyotomy

d. Laparoscopy

e. Test feeding

25. Bile stained vomit would prompt investigations for

a. pyloric stenosis

b. lower intestinal obstruction

c. small bowel obstruction

d. pertussis

e. oesophagitis or peptic ulceration

26. In Northern Europe the incidence of Crohns disease is about

a. 2 per 100,000

b. 1 per 100,000

c. 16 per 100,000

d. 4 per 100,000
e. 8 per 100,000

27. Occurs when the connection between the intestine and the umbilical cord doesn't completely close off
during fetal development, resulting in a small outpouching of the small intestine.

a. Appendictum

b. Vitellointestium

c. Faecolith

d. Peyer's pouch

e. Meckel's diverticulum

28. Intussception usually occurs between _____ of age

a. 4 months and 4 years

b. 1 week and 1 month

c. 2 weeks and 2 months

d. 10 months and 6 years

e. 2 months and 2 years

29. Maintenance intravenous fluid requirement for the weights above 20kg _____ ml/kg/24hr

a. 50

b. 10

c. 100

d. 75

e. 20

30. An enteropathy in which the gliadin fraction of gluten provokes a damaging immunological response in
the proximal small intestinal mucosa.

a. Irritable bowel syndrome

b. Acrodermatitis enteropathica

c. Gastroenteritis

d. Coeliac disease

e. Chron's disease

31. Associated with a shift of water from extra to intracellular compartments

a. hypernatraemic dehydration

b. isonatraemic dehydration

c. asymptomatic dehydration
d. mild dehydration

e. hyponatraemic dehydration

32. Associated with a shift of water into the extracellular space

a. asymptomatic dehydration

b. isonatraemic dehydration

c. hyponatraemic dehydration

d. mild dehydration

e. hypernatraemic dehydration

33. Which of the following does teething cause?

a. vomiting

b. convulsions

c. diarrhoea

d. high fever

e. crying

34. When might reflux be particularly problematic?

a. All of these

b. Following surgery for diaphragmatic hernia

c. Following surgery for oesophageal atresia

d. In children with cerebral plasy

e. In preterm infants who develop bronchopulmonary dysplasia

35. Fluid deficit in ml can be calculated as

a. % dehydration x 5 x weight in kg

b. % dehydration x 100 x weight in kg

c. % dehydration x 2 x weight in kg

d. (% dehydration / 10) x weight in kg

e. % dehydration x 10 x weight in kg

36. The commonest cause of gastroenteritis in developed countries

a. e-coli

b. salmonella

c. campylobacter jejuni

d. rotavirus
e. adenovirus

37. Bile salts no longer enter duodenum in the bile. Leads to defective solubilisation of the products of
trigylceride hydrolysis.

a. Lymphatic leakage or obstruction

b. Short bowel syndrome

c. Cholestatic liver disease or biliary atresia

d. Loss of terminal ileal function

e. Exocrine pancreatic dysfunction

38. Helpful in both diagnosis and checking response to insufflation

a. Barium meal

b. Abdominal X ray

c. CT

d. Ultrasound

e. MRI

39. Maintenance intravenous fluid requirement for the second 10kg of body weight _____ ml/kg/24hr

a. 75

b. 20

c. 10

d. 100

e. 50

40. Involves dystonic movements of the head and neck

a. Hirchsprung's disease

b. Volvulus

c. Malrotation

d. Sandifer's syndrome

e. Intussusception

41. Which of the following is FALSE?

a. Diabetic ketoacidosis may cause severe abdominal pain

b. In 85% of hospital admissions, the pain resolves undiagnosed

c. Primary peritonitis is seen in patients with ascites from nephrotic syndrome

d. Lower lobe pneumonia may cause abdominal pain


e. Urinary tract infection is a possible cause of abdominal pain

42. Usually results from high insensible water losses (high fever or a hot, dry environment)

a. asymptomatic dehydration

b. mild dehydration

c. isonatraemic dehydration

d. hyponatraemic dehydration

e. hypernatraemic dehydration

43. Which of the following is an organic cause of constipation

a. hypothyroidism

b. hypercalcaemia

c. urinary concentrating defect

d. Any of these

e. Hirschsprung's disease

44. Maintenance intravenous fluid requirement for the first 10kg of body weight _____ ml/kg/24hr

a. 10

b. 75

c. 50

d. 100

e. 20

45. The twisting of a loop of bowel around its mesenteric axis. Results in a combination of obstruction
together with occlusion of the main vessels at the base of the involved mesentery.

a. Sandifer's syndrome

b. Volvulus

c. Hirchsprung's disease

d. Malrotation

e. Intussusception

46. An important cause of right iliac fossa pain in children. It is due to non- specific inflammation of lymph
nodes which provokes a mild peritoneal reaction and stimulates painful peristalsis in the terminal ileum.

a. Appendicitis

b. Mesenteric adenitis

c. Faecoliths
d. Intussusception

e. Volvulus

47. Potassium requirement in maintenance intravenous fluid replacement for the first 10kg of body weight
_____ mmol/kg/24hr

a. 4.5-8.5

b. 2.5-4.5

c. 0.5-1.5

d. 1.5-2.5

e. 8.5-16.5

48. Which of the following is NOT a clinical feature of dehydration in an infant?

a. Prolonged cap refill time

b. Increased skin turgor

c. Oliguria

d. Tachypnoea

e. Sunken fontanelle/eyes

49. Following an episode of gastroenteritis, introduction of a normal diet results in a return to diarrhoea.
Temporary lactose intolerance may resolve with a return to ORT for 24 hours followed by a re-introduction
of normal diet.

a. Post infective irritable bowel syndrome

b. Post infective coeliac disease

c. Post gastro-enteritis syndrome

d. Crohn's disease

e. Toddler diarrhoea

50. The invagination of proximal bowel into a distal segment

a. Faecoliths

b. Appendicitis

c. Volvulus

d. Mesenteric adenitis

e. Intussusception

51. Paroxysmal, inconsolable crying or screaming accompanied by drawing up of the kness taking place
several times a day, especially in the evenings

a. Hydronephrosis
b. Porphyria

c. Normal child

d. Colic

e. Duplication

52. Which of the following is recommended in the treatment of gastroenteritis

a. Lomotil

b. All of these

c. Oral or Iv rehydration

d. Anti-emetic drugs

e. Loperamide

53. Usually the commonest bacterial cause of gastroenteritis in developed countries

a. Staphlococcus

b. E.Coli

c. shigella

d. campylobacter jejuni

e. salmonella

54. Sodium requirement in maintenance intravenous fluid replacement for the first 10kg of body weight
_____ mmol/kg/24hr

a. 1-2

b. 0.5-1

c. 8-16

d. 2-4

e. 4-8

55. Blood in the vomit would be suggestive of

a. oesophagitis or peptic ulceration

b. lower intestinal obstruction

c. pyloric stenosis

d. Ulcerative Colitis

e. small bowel obstruction

56. Extraintestinal complications include erythema nodosum, pyoderma gangrenosum, arthritis and
spondylitis
a. Ulcerative Colitis

b. Cystic fibrosis

c. IBS

d. Coeliac

e. Crohn's disease

57. Which of the following is a sign of severe dehydration?

a. weight loss 5-10%

b. deep respiration

c. cap refill > 2 seconds

d. reduced tear production

e. thready/impalpable pulse

58. Which of the following can mimic gastroenteritis in terms of signs and symptoms?

a. pyloric stenosis

b. diabetic ketoacidosis

c. coeliac disease

d. all of these

e. meningitis

59. Presentation is usually in the neonatal period with failure to pass meconium in the first 24 hours of life.
Abdominal distension and later bile stained vomiting develop.

a. Ulcerative Colitis

b. Hirschsprung's disease

c. Cystic fibrosis

d. Coeliac disease

e. Crohn's disease

60. Undigested food may be present in the stools. Probably results from an underlying maturational delay in
intestinal motility. Most children will grown out of symptoms by 5 years of age.

a. Crohn's disease

b. Ulcerative Colitis

c. IBS

d. Coeliac disease

e. Toddler diarrhoea
61. Which of the following is NOT a treatment for GORD?

a. Hyloprimadol

b. Omeprazole

c. Carobel

d. Domperidone

e. Nestargel

62. Characteristically presents with rectal bleeding, diarrhoea, colicky pain and weight loss.

a. Coeliac

b. IBS

c. Crohn's disease

d. Cystic fibrosis

e. Ulcerative Colitis

63. Abdominal distention might be suggestive of

a. pertussis

b. oesophagitis or peptic ulceration

c. pyloric stenosis

d. gastroenteritis

e. lower intestinal obstruction

64. Confirmation of the diagnosis of coeliac disease depends

upon this

a. jejunal biopsy

b. stool cultures

c. anti-endomysial antibodies

d. gluten free test diet

e. tissue transglutaminase antibodies

65. Which of the following is a commonly used stool softener

a. sodium picosulfate

b. movicol

c. senna

d. facilicum

e. postaprandil
66. Attacks of midline, paroxysmal pain associated with facial pallor. Pizotifen is used in prophylaxis.

a. Non ulcer dyspepsia

b. Inflammatory bowel disease

c. Meckel's diverticulum

d. Irritable bowel syndrome

e. Abdominal migraine

67. Peritoneal bands which stretch from the caecum to the subhepatic region. May contribute to an
obstruction.

a. Hallam's bands

b. Ladd's bands

c. Gray's bands

d. Peruvian bands

e. Trussel's bands

68. The absence of ganglion cells from the myenteric and submucosal plexus of part of the large bowel

a. Ulcerative Colitis

b. Hirschsprung's disease

c. Cystic fibrosis

d. Coeliac disease

e. Crohn's disease

69. Which of the following is usually performed to confirm the diagnosis of pyloric stenosis?

a. barium enema

b. abdominal X-ray

c. NGA

d. NPA

e. ultrasound

70. A procedure used to treat pyloric stenosis, where the muscle, but not the mucosa of the pylorus is cut

a. pyloropidication

b. pyomyoloromyotomy

c. pylorotomy

d. pyomylorotomy

e. pyloromyotomy
71. Recurrent abdominal pain, sufficient to interrupt normal activities and lasting for at least 3 months occurs
in ___% of school age children

a. 20

b. 5

c. 15

d. 25

e. 10

72. Any child with dark green vomiting needs one of these, unless signs of vascular compromise is present

a. upper gastrointestinal contrast study

b. MRI scan

c. urgent laparotomy

d. technetium scan

e. CTG scan

73. Intravenous rehydration is always indicated in cases where weight loss due to dehydration is ___ body
weight

a. > 1%

b. > 2%

c. > 7%

d. > 10%

e. > 5%

74. Pyloric stenosis presents between _______ of age

a. 1-2 weeks

b. 5-10 weeks

c. 10-17 weeks

d. 2-7 weeks

e. 7-12 weeks

75. Which of the following is NOT a presentation of intussusception

a. sausage shaped mass palpable in abdomen

b. slapped cheek appearance

c. redcurrant jelly stool

d. abdominal distention
e. paroxysmal severe colicky pain

76. H. Pylori produces urease which forms the basis for the ____ breath test

a. 14C

b. 12C

c. 13C

d. 15C

e. 11C

77. Describes the small amounts of milk which often accompany the return of swallowed air in babies

a. Vomiting

b. Colic

c. Reflux

d. Posseting

e. Regurgitation

78. In ___% of cases of Hirschsprung's disease, the entire colon is involved

a. 20

b. 30

c. 40

d. 50

e. 10
Key test 6

1. a

2. d

3. e

4. a

5. e

6. c

7. e

8. c

9. e

10. e

11. c

12. a

13. b

14. c

15. e

16. a

17. c

18. d

19. e

20. c

21. b

22. a

23. b
24. b

25. c

26. d

27. e

28. e

29. e

30. d

31. e

32. e

33. e

34. a

35. e

36. d

37. a

38. d

39. e

40. d

41. b

42. e

43. d

44. d

45. b

46. b

47. d

48. b

49. c

50. e

51. d

52. c

53. d

54. d
55. a

56. a

57. e

58. d

59. b

60. e

61. a

62. e

63. e

64. a

65. b

66. e

67. b

68. b

69. e

70. e

71. e

72. a

73. d

74. d

75. b

76. c

77. d

78. e
Test 7
Q1. A two and a half-year-old girl is referred for developmental assessment. Her parents report that she has 10
to 15 single words in her vocabulary and one recognisable two word phrase. Her pronunciation of words is not
always clear. She seems to understand most things said to her. Audiological testing is normal. She is physically
very active and finds it difficult to settle to task. However, she can sit and watch television for up to five
minutes. She plays with toy cars by pushing them up and down repeatedly and making engine noises. She also
enjoys playing with dolls and will kiss, hug, scold, pretend to feed them, and push them around in a toy pram.
However, she does not play cooperatively or interactively with other children, and is somewhat self-absorbed.
She runs, climbs a playground slide, scribbles with a crayon and can feed herself with a spoon. She can also
drink from a cup and take off some of her clothes. She tantrums if things do not go her way and screams on
separation from her mother. She becomes highly agitated and cries when the vacuum cleaner is turned on. She
is also frightened of the neighbour's dog.

What is the one best explanation for this child?

A. Anxiety disorder.
B. Autistic spectrum disorder.
C. Intellectual disability.
D. Isolated speech delay.
E. Normal variation.

Q2. An 18-month-old boy was referred by his general practitioner to a paediatrician for advice about febrile
seizures. The child has had three previous episodes of febrile convulsions, the first at 13 months of age. On
each occasion, two seizures have occurred in a 24-hour period. Each seizure was brief (less than five minutes)
in the setting of a high fever (greater than 39°C). The child is developing normally and his neurological
examination is unremarkable. The child’s father has a history of febrile seizures.

The risk of epilepsy in this child is approximately:

A. 0.5%.
B. 2%.
C. 5%.
D. 10%.
E. 15%.

Q3. Which one of the following conditions is most likely to be responsive to treatment with interferon alpha?

A. Arterio-venous malformation.
B. Hepatoblastoma.
C. Large capillary haemangioma.
D. Neuroblastoma.
E. Wilms tumour.

Q4. A 15-year-old girl lives in a rural town where you consult once per month. She presents with a 12-month
history of anxiety-based symptoms, which have prevented her from attending school. She describes a sense
of dread if she is away from home, associated with palpitations, sweating and a heavy feeling in her chest. She
finds it difficult to fall asleep and has numerous nightmares. She worries about germs and frequently washes
her hands. She also describes magical thoughts in that she dreads something bad will happen if she tapes over
her old videos. She believes that these thoughts are silly. Prior to the onset of her symptoms, she functioned
well at school and had many friends. She was not aware of any pressures at the time. She lives with her father
and a 20-year-old sister, her mother having died from a cerebral haemorrhage eight years ago. There is no
family history of psychiatric illness. She refuses to see a psychiatrist but is willing to continue to see you.

In addition to further counselling, which one of the following medications is most appropriate?

A. Amitriptyline.
B. Clomipramine.
C. Oxazepam.
D. Paroxetine.
E. Thioridazine.

Q5. A six-month-old girl presents with a 14-day history of diarrhoea. The illness initially began with fever,
vomiting and diarrhoea. Her vomiting and fever resolved after 36 hours, however, her stools have remained
watery and loose. She has recommenced on her usual cow’s milk based formula and solids including pureed
fruit and vegetables. On examination she is a tired but not unwell looking girl. Her weight is on the 25th
percentile for age and height is on the 50th percentile for age. She is not dehydrated or clinically pale. She has
no rashes. Her abdominal examination reveals a soft non-tender abdomen with no masses or
hepatosplenomegaly. The following blood tests were performed:
sodium 138 mmol/L [135-145]
potassium 4.2 mmol/L [3.5-5.1]
chloride 107 mmol/L [98-110]
urea 2.5 mmol/L [1.3-6.6]
creatinine 0.05 mmol/L [0.01-0.05]

Examination of the stool is most likely to reveal:

A. pH 3.
B. reducing substance negative.
C. rotavirus antigen.
D. sodium 110 mmol/L.
E. white blood cells.
Q6. A four and a half-year-old girl is referred for investigation of short stature. Her birthweight was 2650 g
and length 48 cm at term. Her mid parental height is 164 cm (50th percentile). At age two, her length was on
the 10th percentile. Her health is good except for recurrent otitis media. Physical examination reveals no
dysmorphic features but she has thickened tympanic membranes with fluid behind the drums. Her current
height is 95 cm (1st percentile) and weight is 14 kg (10th percentile).

Which one of the following investigations is most likely to establish a diagnosis?

A. Bone age.
B. Endomysial antibody.
C. Insulin-like growth factor 1.
D. Karyotype.
E. Thyroid-stimulating hormone.

Q7. A five-year-old child presents with fever, vomiting, neck stiffness and a petechial rash. All of the following
are contraindications to immediate lumbar puncture except:
A. hypertension.
B. hypotension.
C. intractable fitting.
D. mild weakness of left arm.
E. moderate drowsiness (Glasgow coma score of 10).

Q8. You are called to see a four-year-old girl with seal-like barking cough, severe inspiratory stridor, marked
wheeze and agitation. Her oxygen saturation is 89% in room air.
Which one of the following would be the most appropriate immediate therapy in addition to oxygen?

A. Nebulised adrenalin.
B. Nebulised ipratropium.
C. Nebulised salbutamol.
D. Nebulised steroids.
E. Oral steroids.

Q9. A seven-year-old boy wakes one morning with severe leg pain, predominantly in the calf muscles after an
upper respiratory tract infection four days previously. He is unable to walk but is otherwise well. The casualty
officer thinks that there is weakness distally and has difficulty obtaining reflexes. The boy experiences a lot of
calf pain on examination. His serum creatine kinase is 2,000 U/L [40-240].

Which one of the following is the most likely diagnosis?

A. Dermatomyositis.
B. Guillain-Barré syndrome.
C. Reactive arthritis.
D. Rhabdomyolysis.
E. Viral myositis.

Q10. An 11-year-old girl presents with a 10-month history of episodic abdominal pain, which clusters over a
two to three-day period. The pain is described as constant, lasting for 40 minutes to one hour and localised to
the mid-abdomen. There is no clear relationship of the pain to meals, specific foods, activity or stool pattern.
She has no associated vomiting, weight loss, or joint pain. On examination she is a well looking pre-pubertal
girl, weight 35 kg (50th percentile) and height 140 cm (25th percentile). Her abdomen is soft with mild diffuse
tenderness noted throughout on deep palpation. There is no rebound tenderness elicited. There is no
organomegaly or masses felt. The remainder of the examination is normal.

Which one of the following would most suggest an organic cause for this girl’s pain?

A. Failure to respond to analgesia.


B. Frequent school absenteeism.
C. Frequent waking from sleep due to pain.
D. Nausea occurring at the time of pain.
E. Positive family history of similar pain.

Q11. The following results are obtained from a two-day-old male infant.
IgG 6.53 g/L [5.34-16.94]
IgA <0.10 g/L [0-0.07]
IgM <0.09 g/L [0-0.18]
IgE <5 kU/L [<25]
Haemoglobin 189 g/L [145-225]
12
Red cell count 5.06 x 10 /L [4.00-6.60]
9
Platelet count 326 x 10 /L [150-400]
9
White cell count 11.5 x 10 /L [5.0-21.0]
Differential:
9
band forms 0.68 x 10 /L (6%)
9
neutrophils 9.01 x 10 /L (78%)
9
lymphocytes 0.10 x 10 /L (1%)
9
monocytes 1.37 x 10 /L (12%)
9
eosinophils 0.34 x 10 /L (3%)

These findings are most consistent with which one of the following?

A. IgA deficiency.
B. Kostmann syndrome.
C. Normal results.
D. Severe combined immune deficiency.
E. X-linked agammaglobulinaemia.

Q12. A six-year-old girl has a three-month history of pubic hair development, body odour and acne. On
examination, she has Tanner stage 1 breasts, stage 3 pubic hair and axillary hair. Her height and weight are on
the 75th percentile.
Investigations reveal:
17-hydroxyprogesterone 1.6 nmol/L [0-6.0]
dehydroepiandrosterone sulphate (DHEAS) 1.5 μmol/L [0.5-1.5]
androstenedione 1.2 nmol/L [0.7-1.7]
testosterone 0.2 nmol/L [<1.0]
oestradiol 32 pmol/L [<50]
bone age six years
pelvic ultrasound normal for age

Based on these investigations, the most likely diagnosis is:

A. adrenal tumour.
B. benign premature adrenarche.
C. congenital adrenal hyperplasia.
D. idiopathic precocious puberty.
E. polycystic ovary syndrome.

Q13. A 28-week gestation infant collapsed on day 12 with necrotising enterocolitis. At laparotomy, the distal
30 cm of ileum was found to be necrotic and was resected. An ileostomy was performed. The ileo-caecal valve
was preserved. An additional 20 cm of the remaining ileum had extensive intramural gas but was thought to
be viable, and was not resected. Enteral feeds were withheld for 14 days and parenteral nutrition (100

kcal/kg/day) was administered. The infant recovered uneventfully after surgery, and oral feeds (Pregestimil
_
20 kcal/30 mL) were recommenced on day 26. There had been a weight gain of 200 g since the laparotomy.
By day 40 enteral feeds had been increased to 90 mL/kg/day (60 kcal/kg/day). The infant was also receiving
parenteral nutrition via a central venous line in a volume of 80 mL/kg/day, providing a further 60 kcal/kg/day.
Sodium, 3 mmol/kg/day, and potassium, 3 mmol/kg/day, were administered with the parenteral nutrition
throughout. Ileostomy fluid losses were 60-80 mL/kg/day after enteral feeds were resumed. No weight gain
occurred between day 26 and day 40. There were no clinical signs of dehydration.
The following laboratory investigations were obtained on day 40:
blood:
sodium 138 mmol/L [135-145]
potassium 3.1 mmol/L [3.4-5.5]
chloride 101 mmol/L [98-110]
pH 7.25
PaCO2 44 mmHg
base excess -9 mmol/L [-4-+3]
bilirubin conjugated 220 μmol/L [<15]
bilirubin unconjugated 110 μmol/L [<15]
alanine aminotransferase (ALT) 320 U/L [0-105]
gamma glutamyltransferase (GGT) 1100 U/L [9-76]

urinary:
sodium <5 mmol/L
potassium 25 mmol/L
osmolality 120 mosmol/kg
ileostomy fluid:
sodium 70 mmol/L
potassium 12 mmol/L
chloride 52 mmol/L

The most important next step in achieving adequate weight gain is to:
A. close the ileostomy.
B. increase enteral caloric intake.
C. increase parenteral caloric intake.
D. increase sodium supplements intravenously.
E. perform a barium study to exclude a stricture.

Q14. A 15-year-old boy presents with a long history of obsessive-compulsive disorder and episodes of
depression. He has previously consulted psychiatrists and psychologists and now, mistrustful of conventional
medicine, sees a naturopath, but without significant alleviation of his symptoms. Although previously an
excellent student, he has lost interest in his studies and sees no point in continuing at school. He denies feeling
unduly sad but admits to some difficulties getting to sleep. Upon further questioning he describes smoking
marijuana three or four times each night to assist his insomnia.

What is the most appropriate first step in his management?

A. Encourage cessation of marijuana.


B. Prescribe sertraline.
C. Prescribe temazepam for two weeks.
D. Recommend a course of hypnosis.
E. Recommend St. John’s wort (Hypericum perforatum).

Q15. Testicular relapse within the first two years following initial diagnosis is more likely to occur in patients
who have which one of the following?

A. Acute monoblastic leukaemia.


B. Acute myeloid leukaemia.
C. B-lineage acute lymphoblastic leukaemia.
D. Chronic myeloid leukaemia.
E. T-lineage acute lymphoblastic leukaemia.

Q16. An 18-month-old child is referred by his general practitioner for paediatric assessment because of concern
about his language development. According to his mother, he seems to understand about 60 words but the only
clear words he says are "mama", "up", "shoe" and "duck". He smiled socially at seven weeks, sat at six and a
half months, crawled at nine months and walked at 14 months. He has had two known episodes of acute otitis
media, at age 10 months and 13 months. He is not yet toilet trained.
He is very active during the assessment, which his mother confirms is usual for him. He is observed to point,
to tug his mother by the hand to show her some toys, and to pretend to drink from a toy cup. Physical
examination is normal.

Which one of the following is most likely to explain this pattern of language development?

A. Asperger disorder.
B. Normal variant.
C. Persistent otitis media with effusion.
D. Sensorineural hearing loss.
E. Specific language disability.

Q17. Which one of the following blood products is likely to have the highest risk of bacterial contamination?
A. Cryoprecipitate.
B. Factor VIII concentrate.
C. Fresh frozen plasma.
D. Platelet concentrate.
E. Suspended red cells.

Q18. An eight-year-old girl presents with recurrent urinary infection causing symptoms of dysuria and
frequency. She ceased wearing nappies during the day at two and a half years and at night at three years. Her
underpants are often damp in the afternoon and evening. Her mother states that she only passes urine two or
three times per day and she does not void at school. Urgency and posturing occur infrequently. She was
constipated as an infant and the only abnormal physical finding is the presence of palpable faecal masses on
abdominal examination.

This girl is likely to have a:

A. bladder with detrusor instability.


B. large capacity highly compliant bladder.
C. normal bladder.
D. psychological non-neuropathic bladder.
E. small capacity hypertonic bladder.

Q19. A general practitioner requests your advice about a six-year-old boy who has sustained a laceration which
has been heavily soiled with manure after a fall in a horse stable. His immunisation status, confirmed by his
child health records, is as follows:
diphtheria-tetanus-pertussis vaccine (DTP) at two and four months
oral polio vaccine (OPV) at two and four months
measles-mumps-rubella vaccine (MMR) at 12 months

Which one of the following should the child now be given?

A. Childhood diphtheria and tetanus toxoids (CDT), tetanus immunoglobulin and OPV.
B. DTP.
C. DTP and OPV.
D. DTP, tetanus immunoglobulin and OPV.
E. Tetanus toxoid and tetanus immunoglobulin.

Q20. A four-month-old infant presents with biphasic stridor since day one of life and an unusual cry, which
has been described as ‘quiet’. The infant sucks and swallows normally. Growth and development are also
normal.

The most likely cause of the stridor is:

A. infantile larynx (laryngomalacia).


B. lingual cyst.
C. subglottic haemangioma.
D. vascular ring.
E. vocal cord lesion.
Q21. In Kawasaki disease, which one of the following laboratory findings would be the least likely?

A. Aseptic meningitis.
B. Elevated serum transaminases.
C. Sterile pyuria.
D. Thrombocytopenia.
E. Toxic granulation of neutrophils.

Q22. A 14-year-old boy in the second year of secondary schooling has always been anxious, insecure and
isolated but has had no previous panic or phobic symptoms. He has a few friends, but at times he behaves in
an inappropriate manner. At school, he is an average student. His teachers believe he is immature but otherwise
normal. For the last two years, he has been masturbating, preoccupied with sexual matters and has interfered
with his parents' conversations with other adults. He worries that whenever his parents go out, they are having
affairs with other people. Over the last four months he has become irritable, moody and angrily reacts to even
mild criticism from his older brothers. He has developed a fear of germs and of being contaminated. After he
has emptied his bowels

he has to have a shower and he worries about stepping on dirty band-aids. He also worries about being attacked
when he rides his bike and has to repeatedly check his windows at night, to ensure that they are locked but
even so, he finds it difficult to sleep. He spends large amounts of time with his mother but worries that if she
touches his clothes, she may become pregnant. He mostly believes that these fears are "silly" but cannot stop
worrying.

Which one of the following is the most likely diagnosis?

A. Anxiety disorder.
B. Depression.
C. Obsessive-compulsive disorder.
D. Phobic disorder.
E. Schizophrenia.

Q23. You are asked to review a 15-year-old boy regarding his short stature. He was treated for
medulloblastoma at age six with cranio-spinal irradiation. His height was on the 50th percentile at diagnosis.
At age 11, when puberty was first noticed, his height was 140 cm (25th percentile). He is now 156 cm tall and
his arm span is 167 cm. His father's height is 172 cm and his mother's height is 158 cm. Preliminary
investigations include:
bone age 15 years
free thyroxine (free T4) 9 pmol/L [8-18]
thyroid-stimulating hormone (TSH) 9 mU/L [<4]
insulin-like growth factor 1 (IGF-1) 15 pmol/L [20-60]

The major cause of his short stature is:

A. attenuated pubertal growth spurt.


B. attenuated spinal growth.
C. familial short stature.
D. growth hormone deficiency.
E. hypothyroidism.
Q24. In an asymptomatic person with human immunodeficiency virus (HIV) infection, which one of the
following is the best predictor of the future rate of decline of immune function?
+
A. CD4 lymphocyte count.
+
B. CD8 lymphocyte count.
C. p24 antigenaemia.
D. Plasma HIV RNA concentration.
E. Serum β2 microglobulin concentration.

Q25. Prophylactic surfactant (given within 15 minutes of birth) has been compared to rescue surfactant (given
immediately after intubation for established hyaline membrane disease), in several controlled trials.
The use of prophylactic surfactant as opposed to rescue surfactant in very low birthweight infants has been
shown to decrease the incidence of which one of the following?

A. Air-leak syndrome.
B. Bronchopulmonary dysplasia.
C. Intraventricular haemorrhage (grade three to four).
D. Patent ductus arteriosus.
E. Periventricular leukomalacia.

Q26. A five-year-old girl is referred with day and night wetting. Her neuro-developmental history is otherwise
normal and bowel training was established prior to three years.
The history reveals that her pants are constantly wet and her teacher has already expressed concern regarding
comments from other children.
Physical examination is normal and urine culture shows no evidence of infection.
A renal ultrasound is suggestive of a duplex right kidney and a normal left kidney, but is otherwise
unremarkable.

Which one of the following is the next most appropriate investigation?

A. Intravenous pyelogram.
B. Micturating cystourethrogram.
C. Nuclear imaging with diuretic washout.
D. Referral for psychological assessment.
E. Urodynamic studies.

Q27. A 13-year-old boy with severe spastic quadriplegia is referred for consideration of placement of a
gastrostomy tube to aid with feeding. He has scoliosis and chronic lung disease.

Which one of the following factors would be a contraindication to percutaneous endoscopic gastrostomy tube
placement in this patient?

A. Oropharyngeal incoordination.
B. Past history of appendicectomy.
C. Recurrent constipation.
D. Severe gastro-oesophageal reflux.
E. Severe generalised hypertonia.
Q28. Which one of the following has most consistently been shown to be associated with an increased risk of
sudden infant death syndrome?
A. Bottle feeding.
B. Exposure to cigarette smoke.
C. Non-immunisation.
D. Room sharing (baby sleeping in own cot in same room as parents).
E. Supine sleeping position.

Q29. Which one of the following sets of urinary electrolytes is most likely to be found in established pyloric
stenosis?

Sodium Potassium Chloride pH


(mmol/L) (mmol/L) (mmol/L)
A. 30 40 0 8
B. 5 5 30 6
C. 40 40 80 8
D. 110 5 60 8
E. 35 45 5 6

Q30. A 2-year-old female child presents with VT, severe ventricular dysfunction,
hypotension, and metabolic acidosis. The patient is cardioverted into ventricular
fibrillation, which degenerates into asystole. What is the most appropriate indication for
using intravenous epinephrine in this patient?

a) Ventricular ectopy
b) Asystole
c) Severe refractory metabolic acidosis and/or hyperkalemia
d) Bradycardia
e) Supraventricular tachycardia

Q31) A 16-year-old female patient presents with short stature and no secondary sexual
characteristics. What diagnosis must be considered?

a) Turner syndrome
b) Isolated growth hormone deficiency
c) Cushing disease
d) Familial short stature
e) Addison disease

Q32) Galactosemia, a disorder of carbohydrate metabolism, is inherited in an autosomal


recessive fashion. What is the risk of galactosemia in a child whose parents are both
carriers for the disorder?

a) 100%
b) 75%
c) 50%
d) 25%
e) 0%

Q33) Which of the following statements is true regarding children with sickle cell disease?
a) Vaccinations are not required because they receive penicillin prophylaxis
b) Gallstones typically develop before the age of 3 years

c) Episodes of dactylitis should be treated with antibiotics


d) Hydroxyurea maintenance therapy decreases the number and severity of vasoocclusive crises
e) Acute chest syndrome requires only supportive care

Q34) A mother brings her 5-year-old son to your office in New Mexico for his regular health
maintenance visit. A quick review of the patient's chart reveals that he and his family are
strict vegans. Their house is very small, so all the children spend a good deal of time
outside. The mother states that her son eats plenty of dark green vegetables and ironfortified grains. She does
not believe in providing supplemental vitamins and minerals.
This child is most at risk for nutritional deficiency involving which of the following?

a) Vitamin B12
b) Vitamin B6
c) Niacin
d) Riboflavin
e) Vitamin D

Q35) A 6-year-old boy presents with a newly appreciated heart murmur. He is


asymptomatic, with normal growth and development and normal exercise tolerance. On
examination S1 and S2 are normal; a II/VI low-frequency midsystolic murmur is heard at
the left lower sternal border. His pulses are normal. The most likely diagnosis is:

a) Bicuspid aortic valve


b) Still's murmur
c) Ventricular septal defect
d) Atrial septal defect
e) Coarctation of the aorta

Q36) You are called to the delivery room for a routine birth. The infant cries when the cord
is cut. You examine the child under the warmer and notice that when he stops crying, his
chest heaves, and he turns blue. You are unable to pass the NG tube through the nose for
suctioning. Which condition is most likely causing this infant's respiratory distress?

a) Choanal atresia or stenosis


b) Vocal cord paralysis
c) Subglottic stenosis
d) Recurrent laryngeal nerve damage
e) Laryngeal web

Q37) A 3-year-old girl is diagnosed with new-onset insulin-dependent diabetes mellitus.


Which of the following laboratory findings is consistent with diabetic ketoacidosis?

a) Hypoglycemia
b) Hypercarbia
c) Ketones in urine
d) Increased venous blood pH
e) Decreased BUN
Q38) During a male newborn examination, the testes are not palpable in the scrotal sacs.
One testis is palpable high in the right inguinal canal and cannot be gently manipulated
into the anatomically correct position. The left testis is not palpable but is discovered in
the abdomen after consultation with a pediatric urologist and an abdominal ultrasound. In counseling the
parents, which one of these statements regarding cryptorchidism is true

a) More than 99% of males have bilateral descended testes at age 1 year
b) Impaired sperm production is not a concern if neither testis descends
c) Malignant degeneration is not a risk factor for testes, which do not descend as
long as they are placed within the scrotal sac through surgery by 1 year of age
d) This infant is no more likely than his peers to manifest an inguinal hernia
e) Microphallus is a common associated condition

Q39) A 5-year-old boy presents with a waddling limp and has had a stiff right hip for the
last 2 months. He has minimal complaints of pain. The most likely diagnosis is:

a) Legg-Calve-Perthes disease
b) Slipped capital femoral epiphysis
c) Toddler's fracture
d) Septic arthritis
e) Juvenile idiopathic arthritis

Q40) A 17-year-old young girl on oral contraceptive therapy for regulation of her
menstrual periods presents with a 1-week history of left leg pain and swelling. Evaluation
with a Doppler ultrasound reveals absence of flow in the left femoral and popliteal veins.
The clot extends proximally to the left external iliac vein. The most important potential
complication that one should be cautious about in this girl is:

a) Venous insufficiency
b) Limb overgrowth
c) Pulmonary embolism
d) Edema
e) Gangrene

Q41) A woman with a seizure disorder under medical management wants to conceive a
child. Her risk of having a child with a neural tube defect is greatest if her current
medical regimen includes which of the following?

a) Phenobarbital
b) Phenytoin
c) Ethosuximide
d) Carbamazepine
e) Primidone

Q42) A 2-month-old infant presents to your emergency department with a heart rate of 220 beats/minute,
pulses, and adequate perfusion. After giving the infant oxygen, you note abnormal P waves and a narrow QRS
(!0.08 sec) on the cardiac monitor. Which of the following is the best course of action?
a) Administer IV/IO epinephrine
b) Administer IV adenosine by rapid bolus
c) Administer IV calcium chloride
d) Administer IV atropine by rapid bolus
e) Administer IV sodium bicarbonate

Q43) A 3-month-old infant presents with a history of abnormal movements that his parents
think might be seizures. You observe an episode of recurrent rhythmic flexor-extensor
spasms that repeat about 30 times before subsiding. The EEG shows hypsarrhythmia, and
a Wood lamp exam is positive for several flat, hypopigmented macules scattered over the
skin surface. This child's infantile spasms are most likely a result of which of the
following underlying disorders?

a) Von Recklinghausen disease


b) Tuberous sclerosis
c) Von Hippel-Lindau disease
d) Sturge-Weber disease
e) Bilateral acoustic neurofibromatosis

Q44) A 21-month-old girl arrives at clinic in May with a vaccination record that indicates
that she has received 3 DTaP doses, 3 Hib doses, 3 IPV doses, 3 pneumococcal conjugate
vaccine doses, 2 hepatitis A vaccine doses, and 3 hepatitis B vaccine doses. Which of the
following should be administered at this visit?

a) DTaP, Hib, IPV, varicella


b) DTaP, Hib, pneumococcal conjugate vaccine, MMR, and varicella
c) DTaP, hepatitis A, IPV, pneumococcal conjugate vaccine
d) DTaP, hepatitis B, MMR, and varicella
e) DTaP, hepatitis A, IPV, MMR, and varicella

Q45) The mother of a 30-month-old boy is concerned that the child's speech is “garbled.”
The child uses “ma-ma” and “da-da” appropriately. He uses about 30 other words, but
most of them are mispronounced (for instance, “boo” instead of “blue”). The boy's aunt,
uncle, and cousins came to visit for a weekend and were unable to understand more than half of what he said.
Examination of the ears reveals normal canals with translucent,
mobile tympanic membranes, and visible landmarks. Which of the following evaluations
for speech delay should be performed first?

a) Receptive language testing


b) Phonetic testing
c) Dysfluency evaluation
d) Tympanogram testing
e) Audiologic (hearing) assessment

Q46) A 13-year-old girl presents with recurrent abdominal pain over the last 3 months. She
has missed a total of 8 days of school. There is no associated fever, weight loss,
gastrointestinal bleeding, and the pain does not occur in relation to meals or awaken her
from sleep. There is diffuse abdominal tenderness but no other abnormal findings on
examination. Which approach is likely to help in the diagnosis and management of her
condition?

a) Abdominal CT scan with contrast


b) Upper and lower endoscopy and biopsies
c) Explaining the likely etiology of her symptoms using a biopsychosocial model
and symptomatic therapy
d) A diet history and a diet elimination trial
e) Referral to a psychiatrist

Q47) A newborn male child has a flat facial profile, upslanted palpebral fissures, epicanthal folds, a small
mouth with a protruding tongue, small genitalia, and simian creases on his hands. What of the following
chromosomal disorders is most likely in this child?

a) Trisomy 21
b) Trisomy 18
c) Trisomy 13
d) Klinefelter syndrome
e) Turner syndrome

Q48) At a 2-year well-child visit, you collect information that your patient lives in a very
old rental home with peeling paint. Both the capillary (screening) and venous blood lead
measurements are 50 "g/dL. The patient has a history of constipation but is otherwise
asymptomatic. Which of the following courses of action is most appropriate?

a) Initiate chelation therapy in a lead-free environment within 48 hours


b) Redraw the blood lead level in 1 week and test all siblings; treat if #50 "g/dL
c) Optimize calcium and iron intake and repeat the blood lead level in 1 month;
treat if #50 "g/dL
d) Refer the family to a lead-removal company; repeat the blood lead level 1
month after decontamination of the home, and treat if #50 "g/dL
e) Refer the case to child protective services for parental neglect

49) A young couple is in your office for their prenatal visit, and you are discussing infant
feeding. The father states that he prefers that the mother breastfeed the baby. The mother is hesitant to commit
to breastfeeding because she plans on returning to full-time
employment 6 weeks after the child is born. Neither her mother nor her sisters chose to
breastfeed. She is concerned that human breast milk may not provide all the nutrients that
the child needs, and she believes formula is a more complete nutritional source for

infants. She is willing to consider exclusive breastfeeding based on the American


Academy of Pediatrics recommendation. If her baby is exclusively breastfed, when
should the child begin receiving oral vitamin D supplementation?
a) Never
b) Within the first month of life
c) Age 2 months
d) Age 4 months
e) Age 6 months
Q50) A 12-year-old female patient presents with fever, night sweats, weight loss, fatigue,
anorexia, and painless, rubbery, cervical lymphadenopathy. What is the most common
presentation of Hodgkin disease?

a) Fever, night sweats, and/or weight loss of >10% in the preceding 6 months
b) Mediastinal lymphadenopathy
c) Painless, rubbery, cervical lymphadenopathy
d) Pruritus
e) Extreme fatigue and anorexia
1. D
2. B
3. C
4. D
5. A
6. D
7. E
8. A
9. E
10. C
11. D
12. B
13. D
14. A
15. E
16. B
17. D
18. B
19. D
20. E
21. D
22. C
23. B
24. D
25. A
26. A
27. D
28. B
29. A
30. B
31. A
32. D
33. D
34. A
35. B
36. A
37. C
38. A
39. A
40. C
41. D
42. B
43. B
44. B
45. E
46. C
47. A
48. A
49. B
50. C

Test 8
Q1. An 8-month-old infant presents with the primary complaint of irritability. He has been
exclusively breastfed since birth. His mother was not interested in providing any supplemental
foods because her milk supply has been adequate. Physical examination reveals a fussy infant
who has frontal bossing and whose weight and height are both at the 25th percentile. The infant
becomes irritable with movement of the left arm. Arm radiography reveals a humeral fracture
and bowing of both radii. Chest radiography demonstrates enlargement of the costochondral
junctions.

Of the following, the MOST likely diagnosis is

A. congenital syphilis
B. osteogenesis imperfecta
C. vitamin D-deficient rickets
D. vitamin D-resistant rickets
E. vitamin E deficiency

Q2. You are evaluating an 8-week-old infant whose birthweight was 1,000 g and who was
delivered at 30 weeks’ gestation. He experienced early respiratory distress and sepsis, but now
these problems have resolved, and he recently progressed from parenteral nutrition to full
enteral feedings. Of the following, the feeding that will provide the BEST mineral content to
ensure healthy bone development for this infant is

A. cow milk-based infant formula


B. human milk
C. premature formula
D. protein hydrolysate formula
E. soy protein-based formula

Q3. You are addressing a group of new mothers regarding infant feeding. One asks you when
an infant can be switched from formula to whole cow milk.

Of the following, you are MOST likely to respond that whole cow milk

A. can be introduced at 6 months of age if an infant has significant gastroesophageal reflux


B. can be given at 9 months of age if the infant is also taking a wide variety of supplemental
foods
C. may be given as a supplement at any age as long as the infant also receives human milk
D. should be avoided until 12 months of age because its iron content is absorbed poorly
E. should be avoided until 2 years of age because its caloric content is inadequate for optimal
growth

Q4. A 2-week-old infant presents to the emergency department with a 1-day history of
decreased feeding, pallor, diaphoresis, and increasing somnolence. He was born at term, and
the delivery was uncomplicated. On physical examination, his heart rate is 190 beats/min, his
respiratory rate is 80 breaths/min, his blood pressure is 50/30 mm Hg, and his extremities are
cool and pale with poor pulses. You place the infant on a cardiorespiratory monitor and begin
your assessment and management.

Of the following, the MOST appropriate pair of tests to consider in this child is

A. complete blood count and bone scan


B. electrocardiography and echocardiography
C. electroencephalography and head ultrasonography
D. lumbar puncture and head computed tomography scan
E. serum electrolytes and chest computed tomography scan

Q5. You are urgently called to the newborn nursery to evaluate a 3-day-old term male infant
who is lethargic. The baby was taking formula well for the first 2 days but vomited after his
last 2 feedings and has become increasingly difficult to arouse. A review of the record reveals
that he was born at 36 weeks’ gestation to a 30-year-old gravida 2, now para 2 woman. The
mother is
Rh-negative and received Rh immune globulin during the pregnancy. Pregnancy, labor, and
delivery were uneventful; Apgar scores were 8 and 9 at 1 and 5 minutes. Family history is
noncontributory. On physical examination, the baby appears well developed, is very jaundiced,
and has hypotonia and tachypnea. He responds with a grimace to sternal rubbing but does not
arouse to voice or to touch. Findings on abdominal examination are normal. You order a series
of laboratory tests, start intravenous fluids, and arrange for transfer to the neonatal intensive
care unit. A laboratory technician subsequently notifies you of some critical laboratory values,
including a platelet count of 35x103/mcL (35x109/L), serum carbon dioxide of 4 mEq/L (4
mmol/L), anion gap of 28 mEq/L (28 mmol/L), serum ammonia of 250 mcmol/L, and total
bilirubin of 20 mg/dL (342 mcmol/L). There are large ketones in the urine.

Of the following, the MOST likely diagnosis is


A. bilirubin encephalopathy
B. citrullinemia
C. hypoxic-ischemic encephalopathy
D. propionic acidemia
E. transient hyperammonemia of the newborn

Q6. During the health supervision visit of a 5-year-old girl, you notice pubic hair (Sexual
Maturity Rating 3). Her height is at the 75th percentile and weight is at the 95th percentile. She
has no acne or clitoromegaly. Her mother tells you the girl developed an adult body odor around
8 months ago, and the mother noticed the pubic hair about 6 months ago. She adds that the
pubic hair is a little more noticeable now than when she first saw it.

Of the following, the MOST important initial screening study is

A. bone age radiograph


B. measurement of dehydroepiandrosterone sulfate (DHEA-S)
C. measurement of 17-hydroxyprogesterone
D. measurement of testosterone
E. pelvic and abdominal ultrasonography
Q7. The parents of a 15-year-old boy previously diagnosed as having constitutional delay of
growth and puberty are concerned that their son is being bullied by his 10th-grade classmates.
His grades have declined from As to Bs, and he says he dislikes school. He has been in good
health, and he eats and exercises in moderation. On physical examination, his height and weight
are at the 10th percentile, and his genitalia are at Sexual Maturity Rating 1. When you speak
to the boy in private, he becomes tearful, complaining that he is afraid to change for gym and
that his friends no longer wish to be with him. He and his parents ask for your advice in this
situation.

Of the following, the BEST response is to

A. offer reassurance and arrange for follow-up evaluation in your office in 6 months
B. recommend caloric supplements
C. refer the family for behavioral counseling
D. refer the boy for a psychoeducational evaluation
E. refer the boy to an endocrinologist for re-evaluation and possible hormonal therapy

Q8. A 16-year-old girl presents to the clinic with a 6-day history of low-grade fever and cough.
On physical examination, she has a temperature of 100.6°F (38.1°C) and widespread crackles
throughout her lung fields. You believe she has a “walking pneumonia” caused by Mycoplasma
pneumoniae.

Of the following, the MOST accurate method used to establish the diagnosis is

A. polymerase chain reaction


B. serum cold agglutinins
C. serum Mycoplasma titers
D. sputum culture
E. sputum Gram stain

Q9. You are evaluating a 12-year-old girl who has a 1-month history of daily fevers (up to
104ºF [40°C]), cervical adenopathy, severe malaise, headache, and lower back pain. She lives
at home with her parents and two sisters, all of whom have been well. She has a 5-year-old cat
and two birds for pets. Six months ago, she spent 2 weeks visiting relatives who live on a ranch
in Mexico where she learned to milk the cows, feed the pigs and chickens, and ride horses. She
also sampled the local cuisine. Physical examination reveals a febrile, tired-appearing girl who
is having rigors. She has diffuse 1 x 1-cm nontender cervical adenopathy, with splenomegaly
and tenderness to palpation of her lower back. Her white blood cell count is 4.9x103/mcL
(4.9x109/L) with 31% polymorphonuclear leukocytes, 15% band forms, 48% lymphocytes,
and 6% monocytes; erythrocyte sedimentation rate is 70 mm/hr; and C-reactive protein
concentration is 6.8 mg/dL.

Of the following, the MOST likely diagnosis is

A. brucellosis
B. cat-scratch disease
C. Epstein-Barr virus mononucleosis
D. leptospirosis
E. toxocariasis
10.A male infant who is experiencing failure to thrive and hypernatremic dehydration is
admitted to the hospital. After administration of intravenous fluids, euvolemia is restored and
the serum sodium is normalized to 140 mEq/L (140 mmol/L). A water deprivation test results
in a 5% loss of body weight over 4 hours and an increase in serum sodium concentration to
145 mEq/L (145 mmol/L) and serum osmolality to 310 mOsm/kg (310 mmol/kg).
Simultaneous urine osmolality is 50 mOsm/kg (50 mmol/kg). Subcutaneous administration of
desmopressin does not reduce urine output or increase urine osmolality. You decide to place
the infant on a formula that possesses a low renal solute load.

Of the following, the statement that BEST describes the properties of infant formulas as they
relate to renal solute load is that

A. human milk has a greater renal solute load than do cow milk-based formulas
B. the carbohydrate composition of the formula increases the renal solute load
C. the fat composition of infant formulas more greatly alters the renal solute load when the
medium-chain triglycerides component is increased
D. the primary minerals involved in renal solute load are sodium, potassium, chloride, and
phosphorus
E. the protein composition of the formula has no effect on renal solute load

11. A 10-year-old African-American boy presents to your office complaining of a 12-month


history of stomach pain, nausea, bloating, and diarrhea that occurs 45 to 60 minutes after eating
dairy foods. He states that his symptoms occur only when he eats “too much.” He denies
emesis, hematochezia, or pruritus associated with these episodes. On physical examination, the
boy appears healthy and has normal vital signs. His abdomen is soft and has normal bowel
sounds, and results of a stool guaiac test are negative.

Of the following, the MOST likely cause for this boy’s symptoms is

A. allergic eosinophilic gastroenteritis


B. lactose intolerance
C. milk protein allergy
D. milk protein enterocolitis
E. oral allergy syndrome

12. A 2-year-old boy is brought to the emergency department after his father found the boy in
the garage gagging, coughing, and drooling profusely. His parents report that he had opened a
number of containers, but they are unable to recall the names of the products. On physical
examination, the boy is awake and crying, his heart rate is 160 beats/min, his respiratory rate
is 24 breaths/min, he has clear lung sounds, and his blood pressure is 100/60 mm Hg. He has
reactive, mid-sized pupils and white eschars on his tongue and soft palate. Shortly after his
examination, he begins to vomit.

Of the following, the MOST likely cause of this patient’s symptoms is

A. antifreeze
B. drain cleaner
C. gasoline
D. glyphosophate weed killer
E. organophosphate insecticide

13. You are evaluating a 4-year-old girl who has a 2-day history of perineal pruritus and
dysuria. There is no history of trauma or sexual abuse. Physical examination reveals Sexual
Maturity Rating 1 genitalia, with erythema of the labia majora, labia minora, and vaginal
introitus. The hymenal tissue appears normal, and there is no vaginal discharge.

Of the following, the MOST appropriate management is

A. administration of fluconazole orally


B. administration of mebendazole orally
C. examination under anesthesia for vaginal foreign body
D. report to child protective services for sexual abuse
E. sitz baths followed by the application of an emollient

14. You are examining a 12-week-old infant who was born at 28 weeks’ gestation, developed
necrotizing enterocolitis 7 days after birth, and underwent a bowel resection of 20 cm of ileum.
The baby has been dependent on parenteral nutrition since shortly after birth and has cholestasis
due to the parenteral nutrition. Attempts to feed her with a cow milk-based formula have
resulted in diarrhea. Stool analysis demonstrates 0.25% (trace) reducing substances and 3+
fecal fat.

Of the following, the intervention that is MOST likely to decrease the infant’s diarrhea is

A. addition of a vitamin supplement to the formula


B. addition of ranitidine to the formula
C. increase in the protein content of the formula
D. use of a formula that has a high medium-chain triglyceride content
E. use of a formula that has a lower concentration of carbohydrates

15. A term infant is delivered by emergency cesarean section following the acute onset of
maternal vaginal bleeding and profound fetal bradycardia. The Apgar scores are 1, 2, and 3 at
1, 5, and 10 minutes, respectively. Resuscitation includes intubation and assisted ventilation,
chest compressions, and intravenous epinephrine. The infant is admitted to the neonatal
intensive care unit and has seizures 6 hours after birth.

Of the following, a TRUE statement about other organ-system injury that may occur in the
infant is that

A. cardiovascular injury is uncommon


B. hypoxic-ischemic encephalopathy usually is an isolated condition
C. liver injury may result in a coagulopathy
D. most infants who have seizures develop cerebral palsy
E. necrotizing enterocolitis does not occur in term infants
16.A medical student rotating in your clinic tells you about a 5-month-old infant he has
evaluated. He reports that the infant is fed goat milk exclusively and asks you if this is adequate
nutrition at this age.

Of the following, the MOST likely deficiency in this infant is of


A. folate
B. iron
C. niacin
D. vitamin A
E. vitamin D

17. You are seeing a 4-year-old boy for a health supervision visit prior to enrollment in
preschool. His mother expresses some concerns about his speech articulation, although he uses
many words. Results of his physical examination, including pneumatic otoscopy, are normal,
as are results of tympanometry. Routine screening audiometry in your office, despite a
cooperative child and a quiet examination room, yields equivocal results.

Of the following, the BEST next step in the evaluation of this child’s hearing is

A. assessment for hearing aid placement


B. reassurance that the child will “grow out of it”
C. referral to an audiologist
D. referral to an otolaryngologist
E. sedated auditory brainstem evoked potentials

18. Yesterday, you received a call from the newborn nursery that they were referring to you a
term infant who was being discharged at 4 days of age. The female newborn’s birthweight was
3.3 kg and the delivery was by repeat cesarean section. Findings on physical examination at
discharge, including heart rate, respiratory rate, and blood pressure, were normal. Her lungs
were clear, and no murmurs were noted. She was breastfeeding without difficulty. Today, her
mother calls to tell you that she is difficult to awaken, pale, and breathing much more rapidly
than she was in the hospital nursery. She has had one wet diaper in the last 12 hours. When you
meet them in the emergency department, you note that the infant has cool extremities, weak
pulses, and lethargy.

Of the following, the MOST likely cause of this newborn’s condition is

A. aortic coarctation
B. atrioventricular septal (canal) defect
C. tetralogy of Fallot
D. transposition of the great arteries
E. ventricular septal defect
19. A 14-year-old girl who has a 1-year history of migraine headaches presents to the
emergency department with a severe headache that she calls “the worst headache of my life.”
The headache occurred suddenly after she lifted a heavy box. Her mother says that the girl has
been holding her head stiffly. On physical examination, she appears in severe pain and has
meningismus. Other findings on the physical examination are normal.

Of the following, the MOST appropriate initial course of action is

A. emergent noncontrast head computed tomography scan


B. intravenous administration of ceftriaxone
C. intravenous administration of dihydroergotamine
D. lumbar puncture
E. oral administration of sumatriptan

20. You care for a newborn who has Down syndrome due to an unbalanced 14;21 translocation.
At the request of the cytogenetics laboratory director, you arranged for the parents’ blood to be
collected and karyotyped. You now are notified that the baby’s mother has an unusual
karyotype with a balanced 14;21 translocation. You plan to refer her for genetic counseling.

Of the following, the recurrence risk for having a baby who has Down syndrome in a future
pregnancy for this woman is CLOSEST to

A. 1% added to her age-related risk


B. 15%
C. 30%
D. 50%
E. 100%

21. A 15-year-old girl comes to your office because she never has had a menstrual period. She
has no chronic illnesses and is active playing softball once a week. Her mother and sister both
had menarche at age 13 years. On physical examination, she is at the 15th percentile for height
and weight and has no hirsutism or acne, no breast development, and Sexual Maturity Rating
3 pubic hair development.

Of the following, the MOST appropriate initial laboratory evaluations are

A. antiovarian antibody and antithyroid antibody concentrations


B. follicle-stimulating hormone concentration and karyotype
C. progesterone and 17-hydroxyprogesterone concentrations
D. testosterone and androstenedione concentrations
E. thyroid-stimulating hormone and thyroxine concentrations

22. You are seeing a 10-year-old girl for her yearly health supervision visit. On physical
examination, you palpate a smooth and symmetric thyroid that seems twice normal size. There
are no palpable nodules. Serum free thyroxine and thyroid-stimulating hormone (TSH) values
are both normal. Serum thyroperoxidase antibody concentrations are elevated.

Of the following, the initial BEST approach to management is to

A. obtain a 123-I thyroid scan


B. obtain thyroid ultrasonography
C. recheck TSH concentration in 6 months
D. start treatment with triiodothyronine
E. start treatment with TSH

23. A 7-year-old girl is being treated with a gonadotropin-releasing hormone agonist by an


endocrinologist for precocious puberty. On physical examination, she is at Sexual Maturity
Rating 3. Her parents are pleased with the medical treatment, but are concerned that their
daughter is having problems fitting in with the 7-year-old girls with whom she used to play,
who
are teasing her. She is now gravitating toward playing with older children in her neighborhood.
The parents ask for guidance regarding their daughter’s behavior.

Of the following, your BEST response to their concerns is to

A. explain that having older friends is beneficial to her self-esteem and should be encouraged
B. explain that their daughter should be treated more maturely because her body is maturing at
a faster rate
C. recommend psychological counseling to help the girl deal with her feelings regarding the
changes in her body
D. suggest that her parents call the girl’s playmates and tell them to stop teasing her
E. suggest that the parents get a pet for their daughter so she will spend less time with the
neighborhood children

24. A 6-year-old boy presents to the clinic with a 2-day history of fever and noisy breathing.
His mother is concerned because she feels that her son is “going to die.” His past medical
history is unremarkable, but he has not yet received his fifth diphtheria-tetanus-acellular
pertussis (DTaP) or his second measles-mumps-rubella (MMR) vaccination. On physical
examination, the boy
appears scared and toxic and has labored respirations and a very harsh cough. He is not drooling
and can lie flat while you examine him. His temperature is 103.5°F (39.7°C), respiratory rate
is 35 breaths/min, heart rate is 168 beats/min, and blood pressure is 107/68 mm Hg. Although
he has tachypnea, his lungs are clear to auscultation, he has no heart murmur, and findings on
his abdominal examination are benign.

Of the following, the MOST likely diagnosis is


A. bacterial tracheitis
B. bronchitis
C. epiglottitis
D. foreign body aspiration
E. laryngotracheobronchitis

25. You are speaking to the pediatric ward nurses regarding prevention of the nosocomial
spread of rotavirus disease after three patients developed the disease while hospitalized. You
stress the importance of good handwashing practices.

Of the following, the MOST appropriate additional statement regarding transmission-based


precautions for rotavirus is that health-care personnel who have patient contact

A. do not need to take transmission-based precautions


B. need to put on gloves before entering patient rooms
C. need to put on gowns and gloves before entering patient rooms
D. need to put on gowns, gloves, and mask before entering patient rooms
E. need to put on gowns, gloves, mask, and shoe covers before entering patient rooms

26. You are asked to evaluate a 3,500-g term infant who was found to have unilateral renal
agenesis on prenatal ultrasonography. Laboratory tests reveal a sodium concentration of 140
mEq/L (140 mmol/L), potassium of 4.1 mEq/L (4.1 mmol/L), chloride of 110 mEq/L (110
mmol/L), and
bicarbonate of 19 mEq/L (19 mmol/L).

Of the following, the serum bicarbonate value can be explained BEST by

A. a normal value for age


B. an inborn error of metabolism
C. renal compensation for primary respiratory alkalosis
D. renal tubular acidosis
E. stool losses of bicarbonate resulting in metabolic acidosis

27. A 9-year-old boy who has a history of asthma and allergic rhinitis presents with coughing,
wheezing, and chest tightness of 4 days’ duration. He only has 1 week of school left before
summer break, but his parents have kept him home because they are administering a beta-2
agonist inhaler every 4 hours to control his symptoms. They mention that “everyone was sick”
in his class, and he developed clear rhinorrhea and a temperature of 99.0°F (37.3°C) at the start
of his current illness.

Of the following, the MOST likely causative virus is


A. coronavirus
B. influenza virus
C. parainfluenza virus
D. respiratory syncytial virus
E. rhinovirus

28. A 2-year-old boy is brought to the emergency department because of moderate respiratory
distress. History reveals that he was born at 25 weeks’ gestation and had bronchopulmonary
dysplasia. He has had rhinorrhea and cough for the past 2 days, and this morning he developed
retractions and wheezing that were unresponsive to albuterol. His usual medications include
albuterol and oxygen at 1 L/min by nasal cannula to maintain his oxygen saturation at 93%.
On physical examination, his heart rate is 160 beats/min, respiratory rate is 60 breaths/min, and
oxygen saturation is 82%. You place him on 100% oxygen using a nonrebreather mask and
obtain blood gases, which reveal a pH of 7.35, Pco2 of 70 mm Hg, Po2 of 226 mm Hg, and
HCO3 of 35 mEq/L (35 mmol/L). As you are interpreting the blood gases, the nurse notifies
you that the boy has developed apnea.

Of the following, the MOST likely explanation for his sudden deterioration is

A. acute pulmonary embolus with increased arterial-alveolar gradient


B. elimination of respiratory drive by correction of chronic hypoxemia
C. respiratory muscle fatigue due to acute metabolic acidosis
D. severe bronchospasm with acute respiratory acidosis
E. spontaneous pneumothorax with acute hypoxemia

29. A 10-week-old infant has undergone abdominal surgery for gastroschisis. After 6 weeks of
parenteral nutrition (PN), cholestasis has developed.

Of the following, the intervention that is MOST likely to reduce the severity of cholestatic liver
disease due to PN is

A. addition of 800 IU of alpha-tocopherol (vitamin E) to the daily PN


B. early introduction of hypocaloric (trophic) enteral feeding
C. elimination of intravenous long-chain triglyceride supplementation
D. reduction of the dextrose concentration of the PN to 15% (15 g/100 mL)
E. removal of branched-chain amino acids from the PN
30. You are evaluating a 3-day-old preterm infant who was born at 26 weeks’ gestation and
weighed 800 g. Her blood pressure has dropped acutely, and she has developed seizures.
Physical examination demonstrates equal mechanical breath sounds, no heart murmur,
hypotonia, a bulging anterior fontanelle, and lethargy. Laboratory evaluation reveals anemia,
metabolic acidemia, and hyperglycemia.

Of the following, the MOST likely explanation for these findings is

A. acute pneumothorax
B. intracranial hemorrhage
C. late-onset sepsis
D. patent ductus arteriosus
E. perinatal asphyxia

31. During the prenatal visit with new parents, a mother expresses concern about regulating
the temperature of the bath water for the new baby. You tell her that standards regarding hot
water heaters have been determined.

Of the following, the temperature that has been determined to be appropriate for hot water
heaters is CLOSEST to

A. 110ºF
B. 120ºF
C. 130ºF
D. 140ºF
E. 150ºF

32. A 7-year-old child who has autism presents with purulent, bloody discharge from the ear
of 1 day’s duration. His mother is unaware of any antecedent events. The child is unable to
provide any history, but his mother explains that he is attending day camp. Examination of the
affected ear reveals blood-tinged, purulent discharge that prohibits visualization of the
tympanic membrane and apparent tenderness to touch of the pinna.

Of the following, the MOST appropriate next step is to

A. lavage the ear in an attempt to examine the tympanic membrane


B. prescribe oral trimethoprim-sulfamethoxazole
C. prescribe topical fluoroquinolone solution
D. prescribe topical tobramycin
E. refer the patient immediately to an otolaryngologist

33. A 1-week-old infant presents to the emergency department with a 1-day history of poor
feeding, pallor, diaphoresis, and increasing somnolence. She was born at term, and the delivery
was uncomplicated. On physical examination, her heart rate is 180 beats/min, respiratory rate
is 90 breaths/min, and blood pressure is 50/30 mm Hg. Her breath sounds are shallow, and
cardiac evaluation reveals no murmurs but a gallop rhythm. Her liver is palpable at 3 cm below
the costal margin. Her extremities are cool, pale, and mottled, and she has poor distal pulses.
After you administer normal saline at 20 mL/kg, her heart rate is 194 beats/min.

Of the following, the MOST appropriate next step is

A. adenosine infusion at 50 mcg/kg


B. computed tomography scan of the head
C. dopamine infusion at 10 mcg/kg per minute
D. lumbar puncture followed by antibiotics
E. normal saline infusion at 20 mL/kg

34. A 10-year-old boy presents with leg weakness that has progressed over 24 hours, bladder
and bowel incontinence, and back pain. There is no history of trauma. On physical examination,
leg reflexes are diminished, and there is numbness in the legs and lower trunk. Rectal
examination demonstrates decreased tone. Sensory examination shows absent pinprick
sensation below T6.

Of the following, the MOST appropriate initial diagnostic test is

A. brain magnetic resonance imaging with contrast


B. electromyography of the legs
C. lumbar puncture
D. nerve conduction velocities
E. spine magnetic resonance imaging with contrast

35. A 16-year-old girl comes to your office with a 2-day history of moderate pelvic pain. She
is sexually active and using no contraception. She has had a scant yellowish-white discharge
from her vagina, but no vaginal itching. On physical examination, she is afebrile, has a heart
rate of 95 beats/min and a blood pressure of 110/75 mm Hg. Bowel sounds are of normal pitch
and frequency. She has no rebound or guarding on abdominal evaluation, but has moderate
pelvic tenderness to palpation. She has no costovertebral angle tenderness. Findings on pelvic
examination include scant yellow cervical discharge, a friable inflamed cervix, cervical motion
tenderness, and mild uterine tenderness but no adnexal masses. Results of her pregnancy test
are negative, and findings on urinalysis are normal.

Of the following, the MOST appropriate next step is

A. C-reactive protein measurement


B. endometrial biopsy
C. laparoscopy
D. pelvic computed tomography scan
E. testing for gonorrhea and chlamydia

36. A 1-year-old boy presents with generalized seizures. His general physical examination
findings are normal except for a prominently positive Chvostek response. Results of laboratory
studies include total serum calcium of 4.5 mg/dL (1.1 mmol/L) and phosphorus of 8.2 mg/dL
(2.73 mmol/L). Blood urea nitrogen and creatinine values are normal for age.

Of the following, the MOST likely diagnosis is

A. dietary calcium deficiency


B. hypoparathyroidism
C. hyperphosphatasia
D. vitamin D deficiency rickets
E. vitamin D-resistant rickets

37. The parents of a 12-year-old boy are concerned about his changing behavior. He was
previously a straight A student, but now he is getting Cs in most of his classes. He no longer
wants to be with his friends after school, and he recently quit the basketball team. On weekends,
he sleeps much of the day, and when awake, he stays alone in his room. Findings on his physical
examination are normal. He appears apathetic and speaks softly in a monotone.

Of the following, your MOST appropriate next step is to

A. evaluate him for attention-deficit/hyperactivity disorder


B. recommend that he rejoin his basketball team
C. refer him for educational evaluation
D. refer him for mental health evaluation
E. schedule a follow-up evaluation in 1 month

38. You are treating a 2-year-old girl who has suspected meningococcal bacteremia and
meningitis. Over the past 2 hours, she has required multiple fluid boluses and inotropic support
to help maintain her blood pressure. She has been intubated due to respiratory failure. Her
temperature is 96°F (35.6°C), and she is covered in a petechial and purpuric rash (Item Q44).
Her most recent laboratory results reveal a white blood cell count of 1.2x103/mcL (1.2x109/L)
with 80% lymphocytes, 10% neutrophils, and 10% band forms and a platelet count of
32x103/mcL (32x109/L).
Of the following, the MOST important additional laboratory test is

A. erythrocyte sedimentation rate


B. measurement of creatine kinase
C. measurement of fibrinogen
D. measurement of lactic acid
E. review of peripheral blood smear

39. A 6-year-old child presents for a health supervision visit. On physical examination, his
weight is 18 kg, height is 102 cm (<3rd percentile),pulse rate is 90 beats/min, respiratory rate
is 18 breaths/min, and blood pressure is 134/88 mm Hg. Of note, he has pale conjunctivae and
mild edema. Among the results of laboratory evaluation are:
• Hemoglobin, 7.5 g/dL (75 g/L)
• White blood cell count, 6.0 x103/mcL (6.0 x109/L)
• Platelet count, 275x103/mcL (275x109/L)
• Mean cell volume, 82 fL
• Reticulocyte count, 0.4% (0.004)
• Blood urea nitrogen, 94 mg/dL (33.6 mmol/L)
• Serum creatinine, 12.1 mg/dL (1,070 mcmol/L)
The stool is negative for occult blood.

Of the following, the MOST likely explanation for this patient’s anemia is

A. chronic gastrointestinal blood loss


B. erythropoietin deficiency
C. folic acid deficiency
D. hemolysis
E. iron deficiency

40. You are working with a medical student in an outpatient pediatric clinic. His first case is a
4- month-old male infant who has a 4-day history of rhinorrhea, coughing, and tachypnea. On
physical examination, the infant is fussy and has a temperature of 99.2°F (37.4°C), pulse rate
of 110 beats/min, respiratory rate of 60 breaths/min, and pulse oximetry of 92% on room air.
On
auscultation, expiratory wheezes are audible in all lung fields. You discuss the differential
diagnosis and ask the student what he knows about respiratory syncytial virus (RSV)
bronchiolitis.

Of the following, the MOST accurate statement regarding RSV bronchiolitis is that

A. household pets can be colonized with RSV and may act as reservoirs for transmission to
family members
B. most chest radiographs of infants hospitalized for RSV bronchiolitis appear normal
C. most infants who develop RSV bronchiolitis require hospitalization
D. recurrent wheezing may occur for several years after mild or severe RSV bronchiolitis
E. the initial RSV infection usually occurs in school-age children

41. A mother brings in her 5-year-old son in for a health supervision visit. Family history
reveals that the boy’s father has had a soft-tissue sarcoma and a colectomy for “colon polyps.”
Results of the boy’s physical examination are within normal limits. The mother asks if her son
is at increased risk for polyps and cancer. You review the father’s medical records, which
indicate that his colectomy was performed at age 20 because of the discovery of 50 adenomas
in the colon.

Of the following, the BEST recommendation for the son at this time is

A. a colonoscopy to survey for polyps


B. annual fecal occult blood testing
C. annual screening of serum alpha-fetoprotein
D. genetic testing to determine his risk
E. ultrasonography for testicular tumors

42. You are conducting prenatal counseling with a 30-year-old mother of two healthy children.
She is 34 weeks pregnant and asks you about potential newborn problems for her soon-to-be
delivered infant, reminding you that her last child was treated for early-onset group B
streptococcal (GBS) infection.

Of the following, your MOST likely reply is that

A. antibiotic treatment during pregnancy will eradicate maternal GBS


B. both the mother and her newborn will require treatment
C. intrapartum chemoprophylaxis is indicated
D. previous GBS infection has no bearing on this pregnancy
E. urinary tract infection with GBS is uncommon

43. You are seeing 16-year-old twin brothers for health supervision visits. They tell you that
they plan to spend most of the summer boating and fishing at their camp on the lake.

Of the following, the advice that is MOST likely to decrease their risk of a boating-related
fatality is to

A. conduct regularly scheduled engine maintenance


B. have both boys take swimming lessons before the summer
C. install a carbon monoxide detector on the boat
D. post the phone number to the United States Coast Guard on the boat
E. wear life jackets at all times while on the boat

44. You care for a 6-month-old boy who was born with pulmonary atresia and ventricular septal
defect. He received a modified Blalock-Taussig (systemic-to-pulmonary artery) shunt 5 days
after birth. His oxygen saturations have ranged between 70% and 84% at office visits over the
past 2 months. During a health supervision visit, you record a hematocrit of 57% (0.57).
Of the following, this child’s polycythemia puts him at INCREASED risk for

A. acute leukemia
B. bacteremia
C. cerebrovascular accident
D. congestive heart failure
E. necrotizing enterocolitis

45. You are called to the nursery to evaluate a newborn who has respiratory distress. On
observation, the baby appears pink while crying but becomes dusky when calm. You note a
hypoplastic nose and depressed nasal bridge on physical examination. A review of her chest
radiographs reveals stippling of the vertebrae.

Of the following, the MOST likely teratogen to cause these findings is

A. alcohol
B. hydantoin
C. retinoic acid
D. valproic acid
E. warfarin

46. A 15-year-old boy comes to your office for a sports physical examination. Compared with
when you saw him at his 10-year-old health supervision visit, he has progressed from Sexual
Maturity Rating 1 to 4.

Of the following, the laboratory value that is MOST likely to have changed is

A. calcium
B. hemoglobin
C. platelet count
D. serum albumin
E. white blood cell count

47. A 12-year-old girl is receiving learning support because of difficulty with reading and
language arts. She struggles to do well in the classroom but realizes she can never compete
successfully with her older sister academically. She is well-coordinated and enjoys playing
basketball in her
yard. Her parents are concerned by her negative comments about herself and ask your advice
on how they can help her improve her self-image.

Of the following, your BEST recommendation is to

A. evaluate the girl for attention-deficit/hyperactivity disorder


B. recommend extracurricular activities for the girl
C. repeat the psychoeducational evaluation
D. suggest that her older sister help with her homework
E. suggest the parents hire an educational advocate

48. A 12-year-old boy who has a history of Hodgkin disease presents to the emergency
department with fever, headache, and malaise of 4 hours’ duration. He has not received any
cancer treatment in more than 1 year. His immunizations are up to date, including the 23-valent
pneumococcal, Haemophilus influenzae type b, and polysaccharide meningococcal vaccines.
He has had a splenectomy. He currently takes oral penicillin twice daily. Physical examination
reveals a toxic-appearing boy who has a temperature of 104°F (40°C), heart rate of 110
beats/min, respiratory rate of 30 breaths/min, and a blood pressure of 118/76 mm Hg. No other
findings on the physical examination suggest the source of his fever.

Of the following, the MOST likely cause of his fever is infection with
A. Haemophilus influenzae type b
B. Pseudomonas aeruginosa
C. Staphylococcus aureus
D. Streptococcus pneumoniae
E. Streptococcus pyogenes

49. You are evaluating a 6-year-old boy who has a 2-day history of fever (102ºF [38.9°C]),
headache, vomiting, moderate crampy periumbilical pain, and watery diarrheal stools (six to
eight a day) that have become blood-streaked in the last 12 hours. He just returned yesterday
from a 3-week visit

to his grandparents’ farm in India. Physical examination reveals a tired boy who has a
temperature of 102.5ºF (39.2°C), tacky mucous membranes, and dry lips. He has diffuse
abdominal tenderness to palpation that is most pronounced in the periumbilical area
andassociated with some voluntary guarding but no rebound. Laboratory tests show a
peripheral white blood cell count of 9.0 x103/mcL (9.0x109/L) with 60% polymorphonuclear
leukocytes, 5% band forms, 30% lymphocytes, and 5% monocytes. A test for the presence of
fecal leukocytes is positive. You suspect Campylobacter sp as the cause of this patient’s
condition.

Of the following, the laboratory condition that is REQUIRED to isolate this organism is

A. aerobic culture conditions


B. antibiotic-free culture media
C. blood agar media
D. incubation temperature of 35ºC
E. incubation temperature of 42ºC

50. A 1-year-old year boy who has chronic kidney disease from posterior urethral valves
presents to your office because his breathing has been noisy for the past 2 hours. His usual
medications include calcium carbonate and vitamin D. On physical examination, you note
inspiratory stridor. He has not had upper respiratory tract symptoms or fever, and there is no
history of choking. The mother reports that the boy was seen by the nephrologist 1 week ago,
and because of worsening renal function, he was begun on sodium bicarbonate.

Of the following, the electrolyte abnormality that BEST explains his current symptoms is

A. hyperkalemia
B. hypermagnesemia
C. hypocalcemia
D. hyponatremia
E. hypophosphatemia

Key test 8
1. C
2. C
3. D
4. B
5. D
6. A
7. E
8. C
9. A
10. D
11. B
12. B
13. E
14. D
15. C
16. A
17. C
18. A
19. A
20. B
21. B
22. C
23. C
24. A
25. C
26. A
27. E
28. B
29. B
30. B
31. B
32. C
33. C
34. E
35. E
36. B
37. D
38. C
39. B
40. D
41. D
42. C
43. E
44. C
45. E
46. B
47. B
48. D
49. E
50. C
test nine
1. Which of the following is FALSE regarding asthma treatment?

a. After an acute exacerbation, maintenance treatment should be reviewed

b. A B2 bronchodilator via a nebuliser may be needed

c. the addition of nebulised ipratropium in severe asthma is beneficial

d. for moderate to severe asthma, 10 puffs of a B2 inhaler should be given

e. IV therapy has no role in asthma treatment

2. Which of the following is widely used for croup despite unproven benefits?

a. nebulised steroids

b. oral dexamethasone

c. inhalation of warm moist air

d. nebulised adrenaline
e. oral prednisolone

3. The most common mutation in CF in the UK

a. F509

b. F508

c. F805

d. F908

e. F505

4. The commonest respiratory infection of infancy. Rare after 1 year of age. 80% of cases are caused
by RSV

a. tracheitis

b. pharyngitis

c. bronchiolitis

d. pertussis

e. pneumonia

5. The combination of situs inversus, chronic sinusitis, and bronchiectasis

a. Kartmann's syndrome

b. Kartheiger's syndrome

c. Kartagener's syndrome

d. Kahlmeter syndrome

e. Kartinger's syndrome

6. Theophylline is an example of a(n)

a. Anticholinergic bronchodilator

b. long acting B2 bronchodilator

c. methylxanthine

d. inhaled steroid

e. B2 agonist

7. In infancy; meconium ileus, prolonged neonatal jaundice, failure to thrive, recurrent chest
infections, and steathorrea may indicate...

a. coryza
b. cystic fibrosis

c. bronchiectasis

d. allergic rhinitis

e. asthma

8. A hereditary disease affecting the exocrine glands of the lungs, liver, pancreas, and intestines,
causing progressive disability due to multisystem failure

a. cystic fibrosis

b. ulcerative colitis

c. asthma

d. allergic rhinitis

e. bronchiectasis

9. Ipratropium bromide is an example of a(n)

a. long acting B2 bronchodilator

b. B2 agonist

c. Anticholinergic bronchodilator

d. leukotriene inhibitor

e. inhaled steroid

10. Which of the following is NOT advised for the management of acute upper airways obstruction?

a. If in doubt, administer nebulised adrenaline

b. Urgent tracheal intubation may be required

c. Observe carefully for signs of hypoxia

d. Examine the throat

e. Reduce anxiety by remaining calm

11. Uncommon in the first decade of life

a. acute otitis media

b. frontal sinusitis

c. pharyngitis

d. bronchiolitis

e. coryza
12. The commonest infection of childhood

a. Pharyngitis

b. Bronchiolitis

c. Sinusitis

d. Coryza

e. Otitis media

13. The most common cause of viral croup

a. adenovirus

b. Parainfluenza viruses

c. RSV

d. rhinovirus

e. Metapneumovirus

14. Which of the following is NOT an URTI?

a. Bronchiolitis

b. Coryza

c. Otitis media

d. Sinusitis

e. Pharyngitis

15. Implies severe or life threatening asthma

a. None of these

b. Oxygen saturation < 92%

c. All of these

d. Silent chest on auscultation

e. Cyanosis, tagigue and drowsiness

16. Provides a simple objective measure of the severity of airflow obstruction in asthma

a. RAST IgE test

b. Chest X ray

c. chest expansion
d. PEFR

e. Skin prick testing

17. On which step of the stepwise approach to asthma management would leukotriene receptor
antagonists normally be added

a. 4

b. 3

c. 2

d. 1

e. 5

18. The most common chronic illness of childhood in the UK

a. sinusitis

b. bronchiectasis

c. cystic fibrosis

d. bronchiolitis

e. asthma

19. Cystic fibrosis is a _______ disease

a. X linked recessive

b. autosomal dominant

c. autosomal recessive

d. X linked dominant

e. mitochondrial inherited

20. Bacterial tracheitis is usually caused by

a. H. influenzae

b. B or C

c. Moraxella catarrhalis

d. A or B

e. Staphylococcus aureus

21. A disease that causes localized, irreversible dilatation of part of the bronchial tree. Involved
bronchi are dilated, inflamed, and easily collapsible, resulting in airflow obstruction and impaired
clearance of secretions
a. primary ciliary dyskinesia

b. tuberculosis

c. bronchiectasis

d. pneumonia

e. cystic fibrosis

22. The most commonly used prophylactic therapy for asthma

a. long acting B2 bronchodilators

b. inhaled steroids

c. Anticholinergic bronchodilators

d. methylxanthines

e. B2 agonists

23. Atopic asthma is also known as

a. IgG-mediated

b. IgM-mediated

c. IgA-mediated

d. IgD-mediated

e. IgE-mediated

24. How is the bordetella pertussis organism usually identified in a child?

a. lung aspirate

b. blood cultures

c. per nasal swab

d. breath test

e. sputum sample

25. A life threatening emergency due to respiratory infection caused by H. Influenza type b

a. Acute epiglottitis

b. Tonsillitis

c. Pharyngitis

d. Coryza
e. Croup

26. CF affects _________ live births

a. 1 in 5000

b. 1 in 2500

c. 1 in 10000

d. 1 in 25

e. 1 in 250

27. The most reliable objective measure of hypoxaemia

a. FBC

b degree of subcostal recession

c. Pulse oximetry

d. Heart rate

e. respiratory rate

28. A gene located on chromosome ___ is responsible for CF

a. 9

b. 5

c. 7

d. 11

e. 6

29. Widely used for acute otitis media

a. Amoxicillin

b. Erythromycin

c. Tetracycline

d. Fluticasone

e. Theophylline

30. In caucasians the carrier rate for CF is

a. 1 in 250

b. 1 in 5000
c.1 in 10000

d. 1 in 2500

e. 1 in 25

31. Terbutaline is an example of a(n)

a. B2 agonist

b. Anticholinergic bronchodilator

c. leukotriene inhibitor

d. inhaled steroid

e. long acting B2 bronchodilator

32. Prednisolone is an example of a(n)

a. Anticholinergic bronchodilator

b. B2 agonist

c. oral steroid

d. long acting B2 bronchodilator

e. inhaled steroid

33. A condition where the microcilia of the respiratory epithelium are abnormal in structure or
function

a. cystic fibrosis

b. primary ciliary dyskinesia

c. bronchiectasis

d. pneumonia

e. tuberculosis

34. Can produce systemic side effects, inclusing impaired growth, adrenal suppression and altered
bone metabolism when high doese are used.

a. B2 agonists

b. methylxanthines

c. Anticholinergic bronchodilators

d. long acting B2 bronchodilators

e. steroids
35. Cause 80-90% of childhood respiratory infections

a. fungi

b. bacteria

c. parasites

d. viruses

e. mycoplasma

36. Common pathogens for tonsilitis?

a. RSV and coronaviruses

b. Adenoviruses and enteroviruses

c. Enteroviruses and rhinoviruses

d. Group A streptococci and Epstein Barr

e. adenoviruses and RSV

37. An infection which causes the alveoli, and the smaller bronchial tubes to become inflamed and fill
with fluid. Viruses are the most common cause in younger children, bacteria commoner in older
children.

a. pharyngitis

b. croup

c. pertussis

d. pneumonia

e. coryza

38. Have a high incidence of side effects including vomiting, insomnia, headaches, and poor
concentration, and so are rarely used in children

a. B2 agonists

b. long acting B2 bronchodilators

c. methylxanthines

d. Anticholinergic bronchodilators

e. inhaled steroids

39. On which step of the stepwise approach to asthma management would daily oral steroids be
added?

a. 1
b. 5

c. 3

d. 2

e. 4

40. The depressions at the base of the thorax associated with the muscular insertion of the diaphragm

a. Henshaw's sulcus

b. Harrison's sulcus

c. Hoffman's sulcus

d. Henchman's sulcus

e. Harriet's sulcus

41. Wheezing is very common in infancy with approximately ____% wheezing at some stage

a. 70

b. 30

c. 60

d. 50

e. 40

42. Used in distal intestinal obstruction syndrome

a. nebulised DNase

b. nebulised hypertonic saline

c. flucloxacillin

d. Gastrografin

e. azithromycin

43. Which of the following is NOT a risk factor for transient early wheezing?

a. maternal smoking after pregnancy

b. maternal smoking during pregnancy

c. all of these are risk factors!

d. family history of atopy

e. prematurity
44. A diffuse inflammatory process of the larynx, trachea, and bronchi with adherent or semiadherent
mucopurulent membranes within the trachea

a. Acute epiglottitis

b. Bacterial tracheitis

c. Infectious mononucleosis

d. Pharyngitis

e. Coryza

45. Salmeterol and Formoterol are example of

a. long acting B2 bronchodilators

b. Anticholinergic bronchodilators

c. inhaled steroids

d. leukotriene inhibitors

e. B2 agonists

46. A chest X ray and Mantoux test are used to exclude

a. tuberculosis

b. ciliary dyskinesia

c. allergic rhinitis

d. asthma

e. cystic fibrosis

47. May be helpful to decrease the viscosity of the sputum in patients with CF

a. Portacath

b. azithromycin

c. flucloxacillin

d. nebulised DNase

e. nebulised hypertonic saline

48. Levels of > 15 x 10^9 / L of these is normally found with bordetella pertussis infection in
whooping cough

a. Lymphocytes

b. Monocytes
c. Basophil

d. Eosinophil

e. Neutrophil

49. Which of the following is NOT a typical feature of epiglottitis?

a. onset over hours

b. preceding coryza

c. drooling saliva

d. Temp > 38.5 degrees C

e. absent or slight cough

50. The commonest cause of recurrent cough in childhood

a. allergic rhinitis

b. tuberculosis

c. cystic fibrosis

d. ciliary dyskinesia

e. asthma

51. Which of the following is NOT a characteristic finding for bronchiolitis?

a. subcostal recession

b. hyperinflation of chest

c. sharp dry cough

d. tachypnoea

e. bradycardia

52. The most common chronic respiratory disorder in childhood, affecting 15-20% of children

a. pneumonia

b. asthma

c. bronchiolitis

d. croup

e. pertussis

53. Usually a self-limiting disease caused by infection of B lymphocytes by the Epstein-Barr virus
a. Coryza

b. pneumonia

c. Pharyngitis

d. infectious mononucleosis

e. bronchiolitis

54. A collection of pus arising outside the capsule of the tonsil in close relationship to its upper pole

a. Kwinsey

b. Quinzi

c. Quincey

d. Kwinsi

e. Quinsy


 55. Lobar consolidation is a feature of

a. viral pneumonia

b. pertussis

c. pneumococcal pneumonia

d. bacterial tracheitis

e. bronchiolitis

56. Eryhtromycin decreases the symptoms of whooping cough if started during the _______ phase

a. pre symptomatic

b. paradoxical

c. mucosal phase

d. catarrhal phase

e. paroxysmal

57. A condition of infancy, in which the soft, immature cartilage of the upper larynx collapses inward
during inhalation, causing airway obstruction.

a. laryngobtusion

b. laryngobfusion

c. laryngitis

d. laryngotomia
e. laryngomalacia

58. A monoclonal antibody to RSV reduces the number of admissions in high risk preterm infants but
use is limited by cost

a. zulivamib

b. mabiluzav

c. palbimizilub

d. malilizume

e. palivizumab

59. An anticholinergic sometimes given to youung infants when other bronchodilators are found to be
ineffective

a. ipratropium bromide

b. budesonide

c. formoterol

d. theophylline

e. terbutaline

60. The most common cause of conductive hearing loss in children - can interfere with normal speech
development and result in learning difficulties at school

a. Rubella

b. Otitis media with effusion

c. Meningitis

d. Treacher Collins syndrome

e. Waardenburg syndrome

61. The most common inherited life limiting disorder in Caucasians

a. DiGeorge's syndrome

b. asthma

c. MBL deficiency

d. Wiskott-Aldrich syndrome

e. cystic fibrosis

62. What percentage of 11-15 year olds smoke?

a. 10%
b. 15%

c. 20%

d. 2%

e. 5%

63. If a child finds that exercise brings on his/her asthma, what might you advise

a. take antihistamines

b. take oral steroids

c. avoid exercising

d. take B2 agonist before exercise

e. consider decongestants

64. Which of the following is NOT a typical feature of croup?

a. onset over days

b. severe barking cough

c. able to drink

d. temp < 38.5 degrees C

e. drooling saliva

65. Typical features are a barking cough, stridor, and hoarseness usually preceded by fever and coryza

a. Pharyngitis

b. Tonsillitis

c. Croup

d. Bronchiolitis

e. Otitis media

66. With regards to PEFR, which of the following is true?

a. diurnal variability is uncommon

b. morning PEFR is usually lower than evening PEFR

c. day-to-day variability is usually absent

d. evening PEFR is usually lower than morning PEFR

e. PEFR usually decreases following use of a bronchodilator


67. Peak incidence of croup is in the ____ year of life

a. 3rd

b. 1st

c. 5th

d. 2nd

e. 4th

68. Also known as viral laryngotracheobronchitis

a. acute epiglottitis

b. croup

c. angioneurotic oedema

d. bacterial tracheitis

e. retropharyngitis

69. Respiratory illness leads to ____% of acute paediatric admissions to hospital

a. 10-20

b. 50-65

c. 5-10

d. 35-50

e. 20-35

70. Beclometasone is an example of a(n)

a. long acting B2 bronchodilator

b. leukotriene inhibitor

c. B2 agonist

d. Anticholinergic bronchodilator

e. inhaled steroid

71. Which of the following causes whooping cough?

a. Haemophilus influenzae

b. Streptococcus pneumoniae

c. Mycobacterium tuberculosis
d. Bordetella pertussis

e. Mycoplasma pneumoniae

Key test 9
1. E.
2. C.
3. B
4. C
5. C
6. C
7. B
8. A
9. C
10. D
11. B
12. D
13. B
14. A
15. C
16. D
17. A
18. E
19. C
20. D
21. C
22. B
23. E
24. C
25. A
26. B
27. C
28. C
29. A
30. E
31. A
32. C
33. B
34. E
35. D
36. D
37. D
38. C
39. B
40. B
41. D
42. D
43. D
44. B
45. A
46. A
47. D
48. A
49. B
50. E
51. E
52. B
53. D
54. E
55. C
56. D
57. E
58. E
59. A
60. B
61. E
62. A
63. D
64. E
65. C
66. B
67. D
68. B
69. E
70. E
71. D
Test 10
Question 1.

A rhythm strip from an electrocardiogram (ECG) is shown above. Which of the following is the most
likely cause for the abnormality displayed?
A. Hyperkalaemia.
B. Hyperthermia.
C. Hyperthyroidism.
D. Hypocalcaemia.
E. Uraemia.

2. Steroids used in the treatment of childhood acute lymphoblastic leukaemia are least frequently
associated with which one of the following side effects?
A. Avascular necrosis of bone.
B. Behavioural disturbances.
C. Bone demineralisation.
D. Renal tubular defects.
E. Weight gain.

3. The most important known factor which influences the rate of development of microvascular
complications in patients with diabetes is:
A. duration of diabetes.
B. hyperglycaemia.
C. hyperlipidaemia.
D. hypertension.
E. smoking.

4. Which of the following anticonvulsants is most likely to increase the half-life of topiramate?
A. Carbamazepine.
B. Phenobarbitone.
C. Phenytoin.
D. Primidone.
E. Sodium valproate.

5. Which of the following antibiotics would be expected to be least active in treating a patient with
infection caused by Pseudomonas aeruginosa?
A. Cefotaxime.
B. Gentamicin.
C. Imipenem.
D. Piperacillin.
E. Ticarcillin.

6. A 24-day-old boy presents with a one-week history of projectile vomiting and weight loss. Which of
the following findings would be most likely on arterial blood gas measurement?
A. Metabolic acidosis.
B. Metabolic alkalosis.
C. Mixed metabolic acidosis and respiratory alkalosis.
D. Mixed metabolic alkalosis and respiratory acidosis.
E. Normal blood gas.

7. In a child with a massive abdominal Burkitt lymphoma, the most useful treatment to prevent tumour
lysis syndrome is:
A. allopurinol.
B. frusemide.
C. mannitol.
D. prednisolone.
E. urate oxidase.

8. Idiopathic ascending aortic dilatation would be most likely to accompany which of the following
syndromes/associations?
A. Down syndrome.
B. Noonan syndrome.
C. Turner syndrome.
D. VACTERL association.
E. Velocardiofacial syndrome.

9. A breastfed two-day-old girl presents with profuse diarrhoea and develops severe dehydration. The
diarrhoea ceased during resuscitation with intravenous (IV) fluids when feeds were paused. However,
when breastfeeds were gradually introduced in the recovery period, the diarrhoea returned and IV fluids
were recommenced. A trial of oral rehydration solution also resulted in severe diarrhoea. Analysis of
the faecal fluid revealed the following electrolyte profile:
sodium 20 mmol/L
potassium 15 mmol/L
chloride 25 mmol/L
Which one of the following is the most likely diagnosis?
A. Congenital chloride diarrhoea.
B. Glucose-galactose malabsorption.
C. Primary lactose intolerance.
D. Secondary lactose intolerance.
E. Sucrase-isomaltase deficiency.

10. The daily energy requirement of a parenterally fed three-week-old neonate is estimated to be 100
kcal (420 kJ) per kilogram body weight. Which of the following intravenous glucose and lipid solutions
will provide adequate daily energy for an infant of 4 kg body weight?

A. 400 mL of glucose 10% plus 20 mL of intravenous lipid 20%.


B. 400 mL of glucose 15% plus 30 mL of intravenous lipid 20%.
C. 400 mL of glucose 15% plus 40 mL of intravenous lipid 20%.
D. 400 mL of glucose 15% plus 50 mL of intravenous lipid 20%.
E. 400 mL of glucose 20% plus 60 mL of intravenous lipid 20%.
11. Which of the following antipsychotic medications is most likely to cause the greatest weight
increase when prescribed to adolescents?
A. Haloperidol.
B. Olanzapine.
C. Pimozide.
D. Quetiapine.
E. Risperidone.

12. Low serum vitamin B12 levels are least likely to be seen in which one of the following?
A. Crohn disease.
B. Pernicious anaemia.
C. Severe folate deficiency.
D. Short bowel syndrome.
E. Transcobalamin II deficiency.

13. The word ‘complex’ in complex partial seizures refers to:


A. a seizure lasting >10 minutes.
B. automatisms occurring during the seizure.
C. focal motor seizures.
D. impaired consciousness during the seizure.
E. multiple seizures in a 24-hour period.

14. The most effective way of preventing mother-to-child transmission of human immunodeficiency
virus (HIV) is:
A. antiretroviral therapy to the mother during delivery.
B. antiretroviral therapy to the mother during pregnancy.
C. antiretroviral therapy to the newborn baby.
D. bottle-feeding.
E. Caesarean section.

15. Which of the following most accurately describes the known effects of supplementation of artificial
milk formulae with long chain polyunsaturated fatty acids (LCPUFA) in term infants?
A. A beneficial effect on bone maturation.
B. A beneficial effect on growth.
C. A beneficial effect on neurodevelopmental outcome.
D. A beneficial effect on visual maturation.
E. No additional benefit demonstrated.

16. If two or more first-degree relatives have atopic disorders, the risk of a subsequent infant developing
an atopic disorder is closest to:
A. <10%.
B. 20%.
C. 33%.
D. 75%.
E. 90%.

17. Activated protein C resistance is most likely to be associated with which one of the following states?
A. Antithrombin III deficiency.
B. Factor V Leiden mutation.
C. Factor VIII deficiency.
D. Lupus anticoagulant.
E. Prothrombin gene mutation.
18. A six-year-old patient with coeliac disease undergoes dual sugar intestinal permeability testing.
Two sugars, rhamnose and lactulose, are administered orally and the subsequent urinary excretion is
measured over five hours. Rhamnose is actively absorbed by the small intestinal villus, whereas
lactulose is absorbed via passive paracellular transport. Which of the following findings in urine
indicate poor adherence to a gluten-free diet?
Rhamnose Lactulose

A. Decreased Decreased

B. Decreased Normal

C. Increased Decreased

D. Increased Normal

E. Normal Increased

19. An ambulance attends a 13-year-old girl who collapses at school and is unconscious for a brief
period. Witnesses to the event said that she had some brief jerking of her limbs when on the ground. By
the time she is brought to the emergency room she is back to normal. On further questioning, there have
been three previous events.
An association with which of the following is most likely to raise concerns that the diagnosis is not
simple syncope?
A. Exercise.
B. Mild trauma.
C. Prolonged standing.
D. Venipuncture.
E. Vomiting.

20. Among children and adolescents with significant behavioural disturbances, a poor prognosis is most
likely with which of the following?
A. Alcoholic or sociopathic father.
B. Greater severity of problem behaviour.
C. Lower age of presentation.
D. Maternal psychosocial problems.
E. Parental divorce.

21. A 12-year-old girl completed treatment for a Ewing sarcoma four years ago. The chemotherapy
agent most likely to increase her risk of second malignancy is:
A. actinomycin-D.
B. cyclophosphamide.
C. doxorubicin.
D. methotrexate.
E. vincristine.

22. Which one of the following interventions has most consistently been shown to decrease the duration
of hospital stay with acute asthma?
A. Inhaled ipratropium bromide.
B. Inhaled salbutamol.
C. Intravenous salbutamol.
D. Oral prednisolone.
E. Oral theophylline.
23. A 15-year-old boy has been on an anticonvulsant for three years. The dosage has been stable. He
recently noticed difficulties in seeing at night. A formal ophthalmological assessment demonstrated
visual field constriction.
Which one of the following anticonvulsants is most likely to be the cause of his symptoms?
A. Carbamazepine.
B. Gabapentin.
C. Lamotrigine.
D. Sodium valproate.
E. Vigabatrin.

24. A term neonate presents with ‘jitteriness’ at three hours of age. The following recordings are made:
weight 3450 g (50th percentile)
length 50 cm (50th percentile)
head circumference 36 cm (50th percentile)
penile length 1.8 cm (<10th percentile)
plasma glucose 0.7 mmol/L
The remainder of the examination is normal.
The most likely cause is:
A. congenital adrenal hyperplasia.
B. 5 alpha-reductase deficiency.
C. hyperinsulinism.
D. hypopituitarism.
E. 45XO/46XY mosaicism.

25. The following results are obtained from a 14-year-old boy, prior to elective hernia surgery, who
gives a history of excessive bleeding after dental extraction.
Bleeding time 6 minutes [2-9]
Prothrombin time 12 seconds [12-14]
Prothrombin time-international normalised ratio (PT-INR) 1.0 [0.9-1.2]
Activated partial thromboplastin time (APTT) 56 seconds [26-35]
Fibrinogen 2.4 g/L [1.8-4.0]
Mixing the patient's plasma with an equal volume of normal plasma normalises the APTT.
Which one of the following is the most likely diagnosis?
A. Antiphospholipid antibody syndrome.
B. Factor VII deficiency.
C. Haemophilia.
D. Recent aspirin ingestion.
E. Von Willebrand disease.

26. Second generation H1 receptor antagonists effectively relieve most of the symptoms of allergic
rhinoconjunctivitis.
Which one of the following symptoms is least effectively relieved?
A. Nasal congestion.
B. Nasal itch.
C. Ocular tearing.
D. Rhinorrhoea.
E. Sneezing.

27. A six-month-old girl develops a chylothorax post-cardiac surgery. After two days her chest drain
continues to drain chylous fluid.
The most appropriate feed for the management of chylothorax in this infant is:
A. breast milk.
B. Isomil (soy milk formula).
C. Neocate (single amino acid infant formula).
D. Portagen (casein hydrolysate formula).
E. S26 (whole protein cow’s milk formula

28. Which one of the following vaccines is contraindicated in a child with anaphylactic egg allergy?
A. Bacille Calmette-Guérin (BCG).
B. Influenza.
C. Measles-mumps-rubella (MMR).
D. Oral polio.
E. Whole cell pertussis.

29. An 18-month-old boy has recurrent blue breath-holding episodes, followed on one occasion by a
15-second generalised seizure.
The investigation most likely to be useful in directing therapy is:
A. electrocardiogram (ECG).
B. electroencephalogram (EEG).
C. iron studies.
D. plasma calcium.
E. plasma glucose.

30. A three-year-old boy presents with a six-month history of polyarticular arthritis. He had been well
until age 12 months when he required hospitalisation for left lower lobe pneumonia. Recurrent otitis
media, intermittent fevers and persistent diarrhoea developed in the second year of life. There was no
history of oral thrush or other fungal infections and no history of opportunistic infections.
Developmental milestones and physical growth were normal and immunisations were appropriate for
age.
Physical examination shows a febrile boy who refuses to weight bear. He has purulent nasal
discharge. The lung fields are clear. Both knees are swollen, tender and warm with effusions and
decreased range of movement. Both ankles are tender with synovial swelling and there is a reduced
range of movement. Examination of the heart is normal. Skin examination is normal.
Which one of the following is the most likely diagnosis?
A. Chronic meningococcaemia.
B. Juvenile chronic arthritis.
C. Late onset hypogammaglobulinaemia (common variable immunodeficiency).
D. Rheumatic fever.
E. Yersinia arthritis.

31. Which feature distinguishes hyper-IgE syndrome from severe atopic dermatitis?
+
A. Decreased number of peripheral blood CD8 lymphocytes.
B. Distribution of the eczematous rash.
C. Extremely high serum IgE levels.
D. Negative delayed skin tests to Candida.
E. Staphylococcal abscesses in the axillary lymph nodes.

32. A four-year-old child has a pneumonia with an effusion that is unresponsive to antibiotic therapy
given intravenously in appropriate doses (ceftriaxone, tobramycin and flucloxacillin) and formal
surgical drainage of the effusion/empyema. Five days after the surgery the child still has temperatures
greater than 39°C and looks unwell. Bacteroides fragilis is grown from the effusion.
A change to which one of the following antibiotics is likely to improve the child?
A. Ceftazidime.
B. Clindamycin.
C. Erythromycin.
D. Penicillin G.
E. Vancomycin.
33. You are asked for a second opinion regarding a 10-year-old boy who has a long history of
hyperactive, impulsive, attention-seeking and destructive behaviour, both at home and at school. A
paediatrician and a child psychiatrist have previously diagnosed him as having attention deficit
hyperactivity disorder (ADHD). He responded poorly to dexamphetamine but concentration and
attention have improved on methylphenidate. He is currently on 30 mg/day. His weight is 35 kg.
Further assessment demonstrated average intelligence but with a significant language-based learning
disorder. He is at the 3rd percentile for receptive language and the 14th percentile for expressive
language. He is making progress at school with the provision of remedial help and an integration aide.
He also receives on-going counselling and cognitive behavioural management from his psychiatrist.
His parents are divorced and are unable to manage him consistently. His father is strict, distant and
loses his temper with his son. His mother is caring, finds it hard to set limits and is overwhelmed by his
demands. The boy dominates and intrudes on all aspects of family life. His father denigrates his ex-
wife in front of the boy.

Which one of the following is likely to be the most helpful?


A. Alter medication regime.
B. Family therapy.
C. Foster care.
D. Individual parental guidance.
E. Transfer to a special school.

34. A 14-month-old infant has been dependent on parenteral nutrition since birth due to surgical
resection secondary to volvulus of the small bowel. He currently receives 30% of his caloric
requirement by the parenteral route and the remainder as Pregestimil® via a nasogastric tube. On
examination he is icteric. He has 3 cm of splenomegaly, the liver is not palpable and there is no ascites.
His blood tests reveal:
bilirubin (total) 120 μmol/L [<10]
bilirubin conjugated 96 μmol/L [<10]
alanine aminotransferase (ALT) 203 U/L [10-50]
gamma glutamyltransferase (GGT) 593 U/L [5-24]
albumin 23 g/L [34-52]
Abdominal ultrasound shows a small liver with normal intra and extra hepatic ducts and an enlarged
spleen. A Doppler study of his portal vessels shows blood flowing from the liver to the spleen. The
most effective intervention to arrest the progression of his liver disease would be to:
A. administer chenodeoxycholic acid.
B. administer phenobarbitone.
C. administer rifampicin.
D. cease parenteral nutrition.
E. perform a portacaval shunt.

35. A four-year-old boy presents with the sole symptom of a generally dry cough of four months
duration, which has been affecting his sleep. He is well grown and there are no localising or generalised
signs. He has not responded to nebulised salbutamol prescribed by his general practitioner. His blood
count and chest X-ray are normal.
Which one of the following is the most appropriate management?
A. Bedroom humidification.
B. Erythromycin.
C. Inhaled corticosteroids.
D. No therapy.
E. Oral steroids.

36. A four-year-old child presents to the emergency department with a two-day history of multiple
bruises and a bleeding nose, two weeks after an upper respiratory tract infection. His past medical
history is unremarkable. On examination, in addition to the features described in the history, there is a
widespread petechial rash noted mainly over the trunk but there are no other abnormal features. Full
blood count shows the following results:
haemoglobin 117 g/L [110-150]
mean corpuscular volume 79 fL [75-90]
red cell morphology normal
9
platelet count <10 x 10 /L [150-400]
9
white cell count 9.8 x 10 /L [5.0-14.5]
differential:
9
lymphocytes 5.8 x 10 /L [1.5-10.0]
9
neutrophils 3.8 x 10 /L [1.0-8.0]
9
monocytes 0.2 x 10 /L [0.2-1.2]
Which one of the following treatments will result in the most rapid rise in the platelet count?
A. Anti-D immunoglobulin.
B. Danazol.
C. Dexamethasone.
D. Intravenous immunoglobulin.
E. Prednisolone.

37. Which one of the following factors is most likely to be associated with the occurrence of late-onset
haemorrhagic disease (vitamin K deficiency bleeding)?
A. Breastfeeding.
B. Formula feeding.
C. Maternal anticonvulsants.
D. Post-natal antibiotic administration.
E. Prematurity.

38. A previously well two and a half-year-old girl, whose parents are divorced, returns from weekend
access with her father. On return home the child is a little more demanding than usual but is otherwise
behaviourally normal. At bed-time the mother notices that her daughter has a significantly reddened
vulva.
Which one of the following is the most likely diagnosis?

A. Candidiasis.
B. Child sexual abuse.
C. Lichen sclerosis.
D. Non-specific vulvovaginitis.
E. Straddle injury.

39. A 15-year-old girl presents with a three-year history of school avoidance, attending approximately
one week per month. She held a scholarship in secondary school but this was withdrawn due to poor
attendance.
At home, she spends her day watching television, drinking alcohol or smoking marijuana. She has a
few friends but is reluctant to leave the house. She is preoccupied with her body and feels herself to be
a freak because she perceives asymmetry in her facial appearance. She panics in social situations,
believing that other people see this appearance. She is volatile and flies into a rage if she is thwarted.
She hates herself, has grazed her wrists and there are scars from cigarette burns on her arms. She denies
suicidal ideation.
At night, she finds it difficult to fall asleep, worries about being attacked and often checks the door and
window locks. She has frequent nightmares and on waking, she thinks she hears strangers in her room.
Her parents divorced eight years ago and do not get on well. She lives with her mother and brother, the
latter having similar but less severe difficulties. He is now at university. Her mother is anxious and
ineffectual and it is clear that the children control the family.
Which one of the following diagnoses best explains this picture?
A. Affective disorder (depression).
B. Borderline personality disorder.
C. Obsessive-compulsive disorder.
D. Panic disorder.
E. Schizophrenia.

40. Which one of the following would not be consistent with a diagnosis of night terrors?
A. Associated somnambulism.
B. Associated sweating and pupillary dilatation.
C. Inability to recall the event.
D. Multiple episodes per night.
E. Occurrence at age seven years.

41. A 13-year-old boy is brought to the emergency room following an intentional ingestion of battery
acid 15 minutes previously. He complains of abdominal pain. On examination he is pale with a pulse
rate of 100/minute, respiratory rate of 30/minute and a systolic blood pressure of 120 mmHg. He has
upper abdominal tenderness with no obvious rebound tenderness. Bowel sounds are present and normal
in character.
Which one of the following is the most appropriate immediate management?
A. Administer antacid.
B. Administer corticosteroids.
C. Administer ipecac.
D. Insert a nasogastric tube and suction gastric contents.

E. Perform a gastroscopy.

42. A nine-month-old boy, who has been fully immunised, presents with a pneumonia and has the
following results:
IgG 1.1 g/L [2.1-12.2]
IgA <0.10 g/L [0.17-1.20]
IgM 0.15 g/L [0.32-1.40]
IgE <5 kU/L [0-35]
Lymphocyte markers:
CD3 (T cells) 93% [53-71]
CD4 (T helper) 68% [28-52]
CD8 (T suppressor) 24% [13-31]
CD19 (B cells) 0% [19-38]
natural killer (NK) cells 4% [3-12]
His pneumonia is most likely to be due to which one of the following organisms?
A. Escherichia coli.
B. Haemophilus influenzae type b.
C. Mycoplasma pneumoniae.
D. Pneumocystis carinii.
E. Staphylococcus aureus

43. An eight-month-old breast-fed baby girl is brought to the emergency department with a 10-day
history of vomiting, irritability and weight loss. The baby developed normally until six months of age
but since then her development has regressed and she is no longer able to sit unsupported.
She is afebrile, pale and listless. Her weight is 6.5 kg (3-10th percentile), length 68 cm (50th
percentile) and head circumference is 43 cm (10-50th percentile). She is generally hypotonic and has
brisk reflexes with positive Babinski responses. Abdominal examination is normal. Investigations
show:
haemoglobin 98 g/L [95-140]
mean corpuscular volume 106.5 fL [70.0-85.0]
mean corpuscular haemoglobin 34.9 pg [24.0-36.0]
white cell count 5.8 x 109/L [5.0-17.0]
neutrophils 1.1 x 109/L [1.0-8.0]
occasional hypersegmented neutrophils seen on blood film
platelet count 445 x 109/L [150-600]
sodium 145 mmol/L [135-145]
chloride 99 mmol/L [95-110]
potassium 4.5 mmol/L [3.5-5.5]
bicarbonate 26 mmol/L [22-26]
urea 6.9 mmol/L [1.3-6.6]
creatinine 0.045 mmol/L [0.020-0.050]
glucose 4.3 mmol/L [2.5-5.5]
Urine metabolic screen gross increase in methylmalonic acid and homocystine
Which one of the following is the most likely diagnosis?
A. Cobalamin C deficiency.
B. Glutaric aciduria type II.
C. Homocystinuria.
D. Methylmalonic acidaemia.
E. Vitamin B12 deficiency.

44. Which anti-arrhythmic drug is inappropriate in the treatment of the arrhythmia with which it is
paired?
A. Atrial flutter: sotalol.
B. Atrial tachycardia in Wolff-Parkinson-White syndrome: flecainide.
C. Prolonged QT syndromes: propranolol.
D. Sinus node dysfunction: digoxin.
E. Ventricular tachycardia in cardiomyopathy: amiodarone.

45. A 30-year-old mother and her five-year-old son are both HIV (human immunodeficiency virus)-
positive. She is well and takes all her own medication reliably. She is pregnant and would like to know
what are the chances of her baby developing HIV infection. She understands that both baby and she
will be treated with the best available current treatments.
The risk of the baby acquiring HIV is closest to:
A. 90%.
B. 70%.
C. 50%.
D. 30%.
E. 10%.

46. Stevens-Johnson syndrome is most likely to occur with which one of the following anticonvulsant
drugs?
A. Carbamazepine.
B. Lamotrigine.
C. Phenytoin.
D. Sodium valproate.
E. Vigabatrin.

47. A 10-year-old girl presents with a recurrent persistent distressing cough of four months duration.
The cough does not occur during sleep, but starts on awakening and fluctuates in intensity and frequency
throughout the day. Her mother describes the cough as very loud. The cough has been present daily and
it started with an upper respiratory tract infection. There was a similar episode 12 months ago at the
same time of the year. The cough was not associated with wheeze, dyspnoea or any sputum production.
Her chest X-ray is normal.
Which one of the following is the most likely diagnosis?
A. Cough variant asthma.
B. Episodic asthma.
C. Pertussis.
D. Post-viral chronic bronchitis.
E. Psychogenic cough.

48. You have been asked to review the biochemical profile of a 13-year-old girl who is undergoing
nasogastric feeding for severe anorexia nervosa, which was initiated three days earlier. You calculate
that she is receiving 100 calories per hour. She had normal biochemistry on admission.
Her current biochemical profile is:
sodium 135 mmol/L [134-142]
potassium 2.7 mmol/L [3.5-4.5]
chloride 98 mmol/L [96-110]
urea 1.0 mmol/L [2.1-6.5]
creatinine 0.02 mmol/L [0.03-0.08]
glucose 2.4 mmol/L [3.5-5.4]
calcium 1.99 mmol/L [2.10-2.60]
phosphate 0.8 mmol/L [1.1-1.8]
albumin 30 g/L [35-50]
This picture is most likely to be due to which one of the following?
A. Addisonian crisis.
B. Diuretic abuse.
C. Laxative abuse.
D. Secondary renal tubular acidosis.
E. The enteral nutrition.

49. Which one of the following cardiac lesions would be an unexpected finding in a baby with a 22q11
deletion?
A. Infradiaphragmatic totally anomalous pulmonary venous return.
B. Interrupted aortic arch.
C. Pulmonary atresia, ventricular septal defect and major aorto-pulmonary collaterals.
D. Tetralogy of Fallot.
E. Truncus arteriosus.

50. Which one of the following measures is most effective in preventing neonatal early onset group B
streptococcal infection?
A. Antibiotics (single intramuscular dose) given after delivery to babies of colonised mothers.
B. Antibiotics given during pregnancy to colonised mothers at 28 weeks gestation.
C. Antibiotics given during pregnancy to colonised mothers with group B streptococcal antigenuria.
D. Maternal intrapartum antibiotics to colonised mothers.
E. Vaginal antiseptic douches before and during delivery to intrapartum colonised mothers.
Key test 10

1.D
2.D
3.B
4.E
5.A
6.B
7.E
8.C
9.B
10.E
11. B
12.E
13.D
14.B
15.E
16.D
17.B
18.B
19.A
20.B
21.B
22.D
23.E
24.D
25.C
26.A
27.D
28.B
29.C
30.B
31.E
32.B
33.D
34.D
35.D
36.D
37.A
38.D
39.B
40.D
41.A
42.B
43.E
44.D
45.E
46.E
47.E
48.E
49.A
50.D
Test 11
1. Following curative therapy, which one of the following primary malignancies is most likely
associated with a future second malignant neoplasm?
A. Acute lymphoblastic leukaemia.
B. Hodgkin lymphoma.
C. Medulloblastoma.
D. Sporadic retinoblastoma.
E. Wilms tumour.

2. An eight-year-old boy is referred to you with tiredness and short stature. He has a long
history of thirst and nocturia. There is no family history of note. On examination he is short
compared with his five year- old sister. Apart from obvious pallor, there are no other abnormal
physical features.

Investigations show the following:

mid-stream urine 50 leucocytes


20 red cells
trace protein
no bacterial growth
haemogloblin 74 g/L [115-155]
serum creatinine 0.20 mmol/L [0.03-0.07]

What is the most likely diagnosis?


A. Familial juvenile nephronophthisis.
B. Medullary cystic disease.
C. Polycystic kidney disease.
D. Reflux nephropathy.
E. Renal Fanconi syndrome.

3. Which one of the following medications is most likely to cause foetal abnormalities if taken
in the first trimester of pregnancy?
A. Enalapril.
B. Gentamicin.
C. Ibuprofen.
D. Tetracycline.
E. Warfarin.

4. In children with reading disability (developmental dyslexia) the underlying neurocognitive


deficit most commonly occurs in:
A. attention.
B. auditory processing.
C. phonemic awareness.
D. saccadic visual tracking.
E. sensorimotor perception.
5. What is the prefered pharmacological treatment for attention deficit hyperactivity disorder
in children with comorbid intellectual disability?
A. Clonidine.
B. Fluoxetine.
C. Haloperidol.
D. Methylplenidate.
E. Risperidone.

6. A seven-year-old girl who underwent surgical repair of coarctation of the aorta as a neonate
requires a dental extraction because of severe dental caries. She has no known drug allergies.
The most appropriate recommendation for antibiotic prophylaxis is:
A. intravenous amoxycillin and intravenous gentamicin 30 minutes before and six hours after
procedure.
B. no antibiotic prophylaxis is required.
C. oral amoxycillin one hour before procedure.
D. oral azithromycin one hour before procedure.
E. oral clindamycin one hour before and six hours after procedure.

7. A 15-year-old male has recurrent haematuria. He had microscopic haematuria noted during
an admission with pneumonia six months earlier. Follow up urine microscopy was normal a
month later. He has developed macroscopic haematuria following a recent throat infection
which lasted six days. Examination is unremarkable. Investigations show a normal full blood
count, creatinine, electrolytes, calcium, liver function, clotting screen and chest radiograph. A
mid-stream specimen of urine shows 90 red blood cells per high powered field but no casts or
protein. Urine culture is sterile. The most likely diagnosis is:
A. Alport.
B. Haemohorregic cystitis.
C. Henoch-Schönlein purpura.
D. IgA nephropathy.
E. Wegener’s granulomatosis.

8. A 12-year-old boy presents with a two year history of tremor predominantly affecting his
hands. The tremor is worse when he is anxious, upset or active. There is a family history of
tremor in a paternal grandparent. On examination, the tremor is present when his hands are
outstretched in front of him. The tremor worsens when asked to perform finger-to-nose
movements. Examination is otherwise normal, including gait. The most likely diagnosis is:
A. anxiety.
B. cerebellar tremor.
C. dystonia.
D. essential tremor.
E. Wilson disease.
9. A ten-week-old male infant presents to the emergency department with a two day history
of coryza and poor feeding. He has not had a wet nappy for at least eight hours. His older
brother has had an upper respiratory tract infection over the past week. On examination, he is
tachypnoeic with respiratory rate of 80/minute, pulse rate 152/minute and is mildly febrile. His
oxygen saturation on monitoring was 92% in air and auscultation of his chest revealed bilateral
crackles. In addition to supplemental oxygen, which of the following should be commenced?
A. Intravenous antibiotics.
B. Intravenous dexamethasone.
C. Intravenous fluids.
D. Nasal CPAP (Continuous Positive Airway Pressure).
E. Nebulized salbutamol.
10. A ten-year-old boy has nephrogenic diabetes insipidus. He consumes a normal diet and is
well. His daily urine output would be most effectively controlled by reducing which one of the
following food items?
A. Carbohydrate.
B. Fat.
C. Potassium.
D. Protein.
E. Sodium.

11. You are asked to see a term 4.1kg newborn infant who is now six hours old. The nursing
staff in the special care nursery are concerned that the infant may be having seizures. In two
separate episodes, the infant had jerking of the upper limbs that lasted for two to three
minutes. The infant was admitted to the nursery after a vaginal delivery with a prolonged
second stage of labour. Blood gas analysis from the umbilical cord at the time of delivery
revealed pH 7.03 [7.34-7.43], pO2 18 mmHg [50-90], pCO2 90 mmHg [31-42], base deficit of
-24 [Base excess -5-+5]. You arrive to find a male infant who has eyes tonically deviated to the
left, and has 30 seconds of rhythmic jerking of the left upper limb. The infant then cries briefly
before settling. Your initial investigations are as follows:
Sodium 134 mmol/l [135-145]
Potassium 5.3 mmol/l [3.7 - 6.0]
Calcium 1.8 mmol/l [1.90 - 2.70]
Glucose 2.1 mmol/l [3.0 - 8.0]
Creatinine 0.08 mmol/L [0.01 - 0.03]
International normalised ratio (INR) 1.7 [0.8 - 1.5]
Haemoglobin (Hb) 176 g/L [145 - 225]
White cell count (WCC) 17.0 x 109 /L [5.0 - 25.0]
Platelets 172 x 109 /L [150 - 400]
The most likely cause of this infant's seizures is:
A. hypocalcaemia.
B. hypoglycaemia.
C. hypoxic-ischaemic encephalopathy.
D. intracranial haemorrhage.
E. stroke.

12. A child with egg allergy is most likely to develop an adverse reaction to which of the
following vaccines?
A. Influenza.
B. Measles-mumps-rubella.
C. Pneumococcal.
D. Polio.
E. Varicella.
13. A four-year-old boy with known peanut allergy and asthma developed angioedema of the
lips followed by generalized urticaria within minutes of blowing up a balloon at a birthday party.
An ambulance was called and on arrival he was wheezy and distressed. Intra-muscular
adrenalin was given with total resolution of symptoms within 20 minutes. The most likely cause
of his reaction was:
A. biphasic reaction to inadvertent peanut exposure.
B. IgE mediated reaction to inadvertent peanut exposure.
C. IgE mediated reaction to latex exposure.
D. peanut induced exacerbation of asthma.
E. T cell mediated reaction to latex exposure.
14. A 13-year-old girl presents with a fractured ankle requiring surgical fixation. Full blood
examination shows:
haemoglobin 105 g/L [120-160]
red cell count 5.6 x 1012 /L [4.0-5.7]
mean cell volume 62 fL [80-97]
white cell count 12.5 x 109 /L [4.0-11.0]
platelet count 390 x 109 /L [150-400]
Haemoglobin (Hb) studies:
HbA2 5.2% [1.8-3.5]
HbF 1.2% [0-2.0]
HbH preparation no HbH bodies seen
Hb electrophoresis no abnormal Hb bands seen
The most likely explanation for her anaemia is:
A. α -thalassaemia trait.
B. β -thalassaemia trait.
C. congenital sideroblastosis.
D. iron-deficiency anaemia.
E. sickle cell trait.
15. A 13-year-old girl who has a body mass index (BMI) of 31kg/m2 [95th centile for age and
sex =28kg/m2 ], is investigated for obesity. Her periods are irregular and she has a family
history of diabetes. On examination she has acanthosis nigricans of the neck and the axillae.
Investigations are shown.
Alanine aminotransferase (ALT) 89 U/L [0-30]
Aspartate aminotransferase (AST) 84 U/L [10-30]
Bilirubin 15 umol/L [<20]
The most likely cause of these findings is:
A. gallstones.
B. non-alcoholic fatty liver disease.
C. polycystic ovary disease.
D. type 2 diabetes mellitus.
E. viral hepatitis.

16. A two-month-old female infant presents with a three week history of a scaly erythematous
patchy rash on the face and arms. Her mother is known to have systemic lupus erythematosis
but is in remission. The auto-antibody most likely to be positive on testing is:
A. anti-double-stranded DNA.
B. anti-Jo-1.
C. anti-Ro/SS-A.
D. anti-RNP.
E. anti-Sd-70.

17. In the context of IgE mediated food allergy, skin prick testing is likely to predict which one
of the following on subsequent exposure to the positive food?
A. Duration of food allergy.
B. Duration of symptoms.
C. Likelihood of a reaction (independent of severity).
D. Severity of a reaction.
E. Timing (onset) of symptoms.
18. A 15-year-old male with Duchenne muscular dystrophy is referred for evaluation of
daytime tiredness, poor school progress and headaches on rising in the morning. On
examination he has a weak cough, and air entry is reduced in both lung bases.
Spirometry reveals the following:
Forced Vital Capacity (FVC) 60% predicted
Forced Expiratory Volume in one second (FEV1) 55% predicted
Maximal mid expiratory flow (MMEF); 55% predicted.
An overnight polysomnogram demonstrates an Obstructive Respiratory Disturbance Index of
3.5/Hr [< 1].
A blood gas examination demonstrates the following:
pH 7.34 [7.34-7.45]
pCO2 59mmHg [34-45]
HCO3 35mmHg [20-28]
What is most appropriate treatment?
A. Adenotonsillectomy.
B. Bilevel ventilation (Bilevel positive airways pressure).
C. Continuous positive airway pressure treatment (CPAP).
D. Supplemental oxygen therapy.
E. Ventilation via tracheostomy.

19. The mother of a four-week-old boy is concerned that her infant has small streaks of bright
red blood and mucus in his stools. The infant is entirely breast-fed and otherwise well. The
physical examination is unremarkable.
What is the most likely diagnosis?
A. Allergic proctocolitis.
B. Anal fissure.
C. Duplication cyst.
D. Gastroenteritis.
E. Rectal polyp.

20. A six-year-old boy with Down syndrome presents to your practice with persistent diarrhoea
for the past four weeks. Stool microscopy reveals the following findings:
Fat globules: +
Fatty acid crystals: +++
Red blood cells: 0
White blood cells: 0
Faecal pH: 5.5 [> 6.5]
Reducing sugars: 1% [< 0.25%]
What is the most likely diagnosis?
A. Coeliac disease.
B. Cow's milk enteropathy.
C. Crohn disease.
D. Infectious colitis.
E. Lactose intolerance.

21. A six-week-old girl is referred for evaluation of noisy breathing. An extrathoracic narrowing
of the trachea is found on bronchoscopy. The most likely type of noisy breathing in this case
is:
A. grunting.
B. rattling.
C. snorting.
D. stridor.
E. wheeze.

22. Which of the following drugs may be taken up to eight hours prior to a histamine bronchial
challenge?
A. Montelucast.
B. Salbutamol.
C. Salmeterol.
D. Nedocromil Sodium.
E. Theophylline.

23. A four-month-old girl with staphylococcal pneumonia acutely deteriorates on the ward
with desaturation and increasing tachypnoea. She is commenced on face-mask continuous
positive airway pressure (CPAP) at 5cm H2 O with initial clinical improvement, but again
deteriorates with increased work of breathing, desaturation to 85%, tachycardia (heart rate
(HR)=200/minute) and tachypnea (respiratory rate (RR)=80/minute). On examination she has
decreased air entry and hyperresonance on the right side of her chest with tracheal deviation
to the left. Which of the following should be the next step in her management?
A. Endotracheal intubation and mechanical ventilation.
B. Increase face mask CPAP to 7cm H2 O.
C. Right-sided intercostal catheter.
D. Right-sided needle thoracocentesis.
E. Urgent chest radiograph.

24. A newborn baby is noticed to have genital ambiguity. Gonads are palpable in the
labioscrotal region. Investigation reveals XY Karyotype. Following HCG (human chorionic
gonadotrophin) stimulation test at two weeks the maximum testosterone level is 6.6 nmol/L
[<1.7].
The most likely diagnosis is:
A. androgen insensitivity syndrome.
B. congenital adrenal hyperplasia.
C. gonadal dygenesis.
D. Klinefelter syndrome.
E. testosterone biosynthetic defect

25. A test has a sensitivity of 95% and a specificity of 90%. It is used to screen the general
population for a condition that has a prevalence of 1 in 100,000. What will the positive
predictive value be nearest to?
A. 0.01%.
B. 0.05%.
C. 0.1%.
D. 0.5%.
E. 1%.

26. A four-year-old girl presents for investigation of developmental regression. Early


development had been normal, but starting from nine months of age she had lost social and
motor skills, and by the age of two years had almost no purposeful hand movements. By the
age of four years, she exhibited stereotyped behaviours and frequent episodes of
hyperventilation. She has a history of two generalised seizures.
The most likely diagnosis is:
A. autism.
B. Leigh disease.
C. metachromatic leukodystrophy.
D. neuronal ceroid lipofuscinosis.
E. Rett syndrome.

27. An 18-month-old baby of consanguineous parents of Muslim faith is found to be delayed


in his motor milestones. In particular he is not walking. He is breast fed, and tolerates feeds
well. He is the third child in the family and his cognitive development is similar to his two older
siblings. The following investigations are done:
25-Vitamin D 16 nmol/L [30-150]
Calcium 2.23 mmol/L [2.15-2.60]
Phosphate (PO4 ) 1.82 mmol/L [1.10-1.80]
Parathyroid hormone 14 pmol/L [0.8-8.0]
Maternal vitamin D 21 nmol/L [35-150]
The most likely cause of these findings is:
A. Familial hyperphophataemic rickets.
B. Nutritional vitamin D deficiency.
C. Primary hyperparathyroidism.
D. Pseudohypoparathyroidism.
E. Vitamin D resistant rickets

28. A three-year-old girl is seen with a history of a 24-hour viral illness. She is brought to the
hospital following a 15-minute generalised seizure. Blood glucose is 2.6 mmol/L [4.5-6.2].
Further investigations are as follows:
Insulin 1 mU/L [<12]
Beta hydroxy butyrate 2.4 mmol/L [<0.3]
Ammonium 35 umol/L [<50]
What is the most likely diagnosis in this child?C
A. Glucokinase enzyme mutation.
B. Glutamate dehydrogenase enzyme mutation.
C. Ketotic hypoglycaemia.
D. Malabsorption.
E. Maple syrup urine disease.

30. Following an uneventful labour and delivery, a term female infant is born in good condition.
At six hours of age, she is noted to be cyanosed. She is vigorous with a pulse rate of 140/minute,
and no signs of respiratory distress or heart murmur. Her pulses are normal volume. A pulse
oximeter placed on her right hand reveals an oxygen saturation of 75%. The most likely
diagnosis is:
A. hypoplastic left heart.
B. persistent pulmonary hypertension of the newborn.
C. tetralogy of Fallot.
D. total anomalous pulmonary venous drainage.
E. transposition of the great vessel

31. A 15-year-old boy is seen with concerns about his lack of pubertal development. On
examination his height is on the third percentile, pubic hair Tanner stage 1, genital
development Tanner stage 1 and testes 6ml bilaterally. His bone age is 12-years- and-six-
months. The most likely diagnosis is:
A. constitutional growth delay.
B. growth hormone deficiency.
C. hypergonadotrophic hypogonadism.
D. hypogonadotrophic hypogonadism.
E. Kallman syndrome.
32. A 4.9kg term infant is delivered by emergency caesarian section for foetal distress. His
mother had no antenatal care. He requires four minutes of bag and mask ventilation but by 10
minutes of age has an Apgar score of 9. He tolerates a bottle feed of 45ml of milk formula at
60 minutes of age. Laboratory blood sugar is 0.9 mmol/L at four-hours of age and he is
admitted to the Neonatal Intensive Care Unit (NICU). On examination he is macrosomic, hirsute
and has a large tongue but no other dysmorphic features.
Laboratory investigations include:
Blood glucose 1.2 mmol/L [> 2.6]
Plasma cortisol 350 nmol/L [250-800]
Plasma growth hormone 43.4 μg/L [5-53]
Plasma insulin 80 pmol/L [9-80]
beta-hydroxybutyrate 20 μmol/L [20-270]
Free fatty acids 0.0 mEq/L [0.0-0.6]
Plasma ammonia 91 μmol/L [<100]
Blood pyruvate 151 μmol/L [30-165]
Urinary ketones negative
Which of the following is the most likely cause of the hypoglycaemia?
A. Adrenal insufficiency.
B. Fatty acid oxidation defect.
C. Hyperinsulinism.
D. Hypopituitarism.
E. Simple substrate deficiency.

33. A three-year-old boy presents after developing severe swelling of the left foot and leg to
mid thigh immediately after being stung by a bee on the left foot. There is a past history of
large local reactions to mosquito bites. He has mild eczema and wheezes in association with
viral upper respiratory tract infections. His risk of a severe systemic reaction to a subsequent
bee sting is closest to:
A. 10%.
B. 30%.
C. 50%.
D. 70%.
E. 90%.

34. A seven-day-old term infant is noted to have a bilateral purulent eye discharge which is
yellow in colour. There is associated cellulitis of both eyelids. The most likely cause of this
infant's conjunctivitis is:
A. Chlamydia trachomatis .
B. Neisseria meningitidis .
C. Herpes simplex virus.
D. Pseudomonas aeruginosa .
E. Staphylococcus aureus.

35. A three-year-old boy presents to the emergency department with a fever and coryza. On
examination he has a temperature of 37.8°C, heart rate 110/minute, respiratory rate
30/minute, blood pressure 88/60 mmHg. He is miserable with a runny nose and palpable small
cervical nodes. He has bilaterally red tympanic membranes which are not bulging and his
pharynx is slightly inflamed with no purulent exudate. His chest and abdominal examination
are unremarkable and he has no evidence of oedema. His urinalysis demonstrates 3+
haematuria, with no proteinuria. A spun urine demonstrates the
following:
Result
Glucose Negative
Bilirubin Negative
Ketones Negative
Specific Gravity 1.010
Blood 2+
pH 7.0
Protein Negative
Urobilinogen Negative
Nitrites Negative
Leukocytes Negative
Red Blood Cell Number/High Power Field 2-3
Dysmorphic Red Blood Cell 20%
Casts Number/Low Power Field 0
Casts Number/Slide 0
White Blood Cell Number/High Power Field 0
Other Cells None Seen
Bacteria None Seen
Which of the following is the most appropriate next investigation?
A. Coagulation studies.
B. Renal ultrasound.
C. Repeat urinalysis in 3-6 months.
D. Serum creatinine.
E. Urinary calcium / creatinine ratio.

36. A nine-month-old girl is referred by her general practitioner because of recurrent events
occurring on a daily basis over the last two weeks. The episodes are stereotyped and consist
of her stopping what she is doing, flexing at her trunk, and pressing her hands above her
inguinal region. There is associated tremulousness and jaw rigidity. The events last one to two
minutes with her becoming red in the face and grunting. She seems to be preoccupied and
gets distressed if touched or moved. After the event, the child goes to sleep. The events never
occur in sleep.
The most likely diagnosis is:
A. dystonia.
B. frontal lobe seizures.
C. gastroesophageal reflux.
D. infantile masturbation.
E. urinary infection.
37. A ten-year-old boy presents with facial swelling, hypertension, proteinuria and haematuria.
Complement levels are 0.7 mg/dL [0.8-1.8]; antistreptolysin-O titre (ASOT) and anti-DNase are
positive. He is excreting 500 mg of urinary protein in 24 hours [<200]. Anti-nuclear antibodies
are negative, and he has no other symptoms or signs suggesting autoimmune phenomena.
After initial treatment with fluid restriction, diet and antihypertensives, his urinary sediment
improves, and blood pressure returns to normal. At two months he is reassessed. Which of the
following would most strongly indicate the need for renal biopsy?
A. Creatinine of 0.10 mmol/L [0.05 – 0.10].
B. Haematuria.
C. Hypertension.
D. Hypocomplementaemia.
E. Proteinuria of > 200 mg/24 hours.
38. A three-and-a-half-year-old boy is referred because his parents are concerned about his
poor social skills. He prefers to be alone. He can be quite insistent on routines and is easily
upset if he does not get his way. Some of his tantrums last up to half an hour. He is interested
in mechanical things such as DVD, tape recorder etc. His attentional skills are not good,
however he has recently developed some good imaginative play. He has good expressive and
receptive language skills. He has no repetitive behaviours or unusual body movement and no
fascinations or obsessions. He is an only child. His father tends to be obsessive and anxious,
and his mother was very shy when she was small. A first cousin has autism.
He is very wary of you initially, but after some time begins to respond to your requests and
follows some single and two stages commands. He is quiet with poor eye contact, but has some
reasonably communicative facial expressions and gestures. A single five word sentence is
heard. Physically examination is normal. His weight and height are both on the 10th percentile
for age, head circumference 50th percentile. Before leaving he brings a toy car out of his pocket
to show you.
What is the most likely diagnosis?
A. Asperger syndrome.
B. Attention deficit hyperactivity disorder.
C. Autism.
D. Fragile X syndrome.
E. Normal child.

39. A ten-year-old boy is investigated for excessive day time sleepiness. He sleeps 11 hours per
night and snores most nights. He wakes frequently at night and calls out to his mother.
He describes periods of being not being able to move in bed as though he is paralysed. He
occasionally falls to the ground when laughing.
The most likely diagnosis is:
A. atonic epilepsy.
B. narcolepsy.
C. nocturnal epilepsy.
D. obstructive sleep apnoea.
E. parasomnia.

40. A previously well eight-year-old boy presents to the emergency department with two days
of persistent vomiting and several loose bowel motions. He has had no urine output for at least
12 hours. Examination shows that he is mildly dehydrated but his vital signs are normal for age.
He is given a bolus of 500mL of 0.9% saline intravenously and discharged home with advice to
visit the family doctor the next day. He represents five hours later with persistent vomiting and
increasing lethargy. He passes 20ml of dark urine. Initial tests show the following:
Sodium 132 mmol/L [134-144]
Potassium 7.5 mmol/L [3.5-5.0]
Urea 39 mmol/L [3.0-7.5]
Creatinine 0.35 mmol/L [0.04-0.08]
Haemogloblin 105 g/L [120-155]
Platelet count 235 x 109 [150-400]
Urinalysis shows 3+blood, 3+protein
The most likely diagnosis is:
A. acute dehydration.
B. acute glomerulonephritis.
C. acute interstitial nephritis.
D. acute tubular necrosis.
E. nephrotic syndrome.

41. A ten-year-old boy presents after a hypoglycaemic fit preceded by 24 hours of vomiting.
He has a past medical history of asthma with no recent exacerbations or significant interval
symptoms. He is on a Seretide Accuhaler (500 micrograms of fluticasone and 50 micrograms
salmeterol), one inhalation twice daily. His physical examination is unremarkable.
Which investigation is most likely to confirm the cause of his hypoglycaemia?
A. Adrenal antibodies.
B. Adrenocorticotrophic Hormone (ACTH) level.
C. Random cortisol level.
D. Stimulated cortisol level.
E. Very long chain fatty acid levels.
42. A four-year-old boy is brought to the emergency department with a one day history of
fever, vomiting and lethargy. His parents are concerned that he is pale and that his eyes look
yellow. On examination his temperature is 37.5°C, heart rate is 110/minute, blood pressure
90/60 mmHg and respiratory rate is 25/minute. He is pale and quiet, but responsive, with
mildly icteric sclera. His examination is otherwise unremarkable with no evidence of
hepatosplenomegaly. A urinalysis is performed and is positive for blood. His blood test results
are displayed below.
Haemoglobin (g/L) 78 [116-136]
MCV (fL) 83 [75-85]
MCH (pg) 29 [24-30]
MCHC (g/L) 345 [310-360]
Platelets (x 109 /L) 292 [150-450]
Reticulocytes (x 109 /L) 154.2 [20-105 x 109 /L]
Heinz Bodies Negative
Poikilocytosis +
Spherocytosis Occasional
White Blood Cells (x 109 /L) 5.83 [6.10-11.00]
Bilirubin Total (μmol/L) 84 [2-20]
Bilirubin Conjugated (μmol/L) 8 [<10]
Haptoglobin (g/L) < 0.06 [0.35-2.10]
Direct Antiglobulin Test Poly Negative
Direct Antiglobulin Test IgG Negative
Direct Antiglobulin Test C3d Negative
Which of the following diagnoses is most likely?
A. Cold antibody autoimmune haemolytic anaemia.
B. Glucose-6-phosphate dehydrogenase deficiency.
C. Hereditary spherocytosis.
D. Paroxysmal nocturnal haemoglobinuria.
E. Warm antibody autoimmune haemolytic anaemia.

43. A five-year-old boy is referred to the haematology clinic with a history of easy bruising and
excessive bleeding from mild trauma. The following investigations were undertaken:
Haemoglobin (Hb) 109 g/L [115-155]
White cell count (WCC) 7.7x109 /L [4.5-14.5]
Platelets 257x109 /L [150-400]
Prothombin time 13.7 second [14-18]
Activated partial thromboplastin time 72.1 seconds [22.5-34.5]
50:50 Mix 30.4 seconds
Thrombin clotting time 15.5 seconds [14-18]
F.VIIIc 3% [50 -150]
vWF Ristocetin cofactor 18% [50-200]
vWF Antigen 21% [50-200]
Collagen Binding Assay 14% [50-200]
The most likely diagnosis is:
A. Factor VII deficiency.
B. Glanzmann’s syndrome.
C. Haemophilia A.
D. Haemophilia B.
E. von Willebrand Disease.

44. An otherwise normal six-year-old boy is diagnosed with moderate sensorineural hearing
loss. There is no significant family history. The most likely cause of his hearing loss is:
A. congenital cytomegalovirus.
B. congenital rubella.
C. Connexin 26 mutation.
D. Pendred syndrome.
E. Usher syndrome.

45. Neuroadaptation (tolerance) syndrome is the phenomenon of loss of treatment response


generally after months of a clear and definite response to medication. With which of the
following medications is this most likely to occur in children?
A. Carbamazepine.
B. Clonazepam.
C. Fluoxetine.
D. Methylphenidate.
E. Sodium valproate.

46. A previously well two-year-old boy accidentally ingests a large dose of ibuprofen, estimated
to be 6-8 times the recommended dose. Emergency decontamination of his stomach is carried
out in the Emergency department. Two hours after ingestion, his serum creatinine is
0.05mmol/l [0.03-0.07]. What is the next most appropriate next step?
A. Check serum creatinine in 6-8 hours.
B. Forced alkaline diuresis.
C. High dose prednisolone.
D. Prostaglandin agonist.
E. Urgent urinalysis.

47. A 12-year-old boy with a past history of allogeneic bone marrow transplantation (BMT) for
severe aplastic anaemia is on cyclosporin and prednisone for active extensive chronic graft-vs-
host disease (GVHD). He presents to emergency with a high fever. The following investigations
were performed:
Haemoglobin (Hb) 129 g/L [125-170]
White cell count (WCC) 26.5 x109 /L [4.5-13]
Platelets 91 x109 /L [150-400]
Sodium 136 mmol/L [37-145]
Potassium 4.2 mmol/L [3.5-5]
HCO2 18 mmol/L [22-30]
Urea 10.8 mmol/L [3.2-7.1]
Creatinine 0.221 mmol/L [<0.133]
C-reactive protein (CRP) 133 mg/L [0 -10]
His blood film shows Howell-Jolly bodies. Which one of the following organisms is most likely
responsible for his clinical presentation?
A. Escherichia coli .
B. Pseudomonas aeruginosa .
C. Staphylococcus aureus .
D. Streptococcus pneumoniae .
E. Streptococcus viridans .

48. The diagnosis of neurofibromatosis type 1 (NF-1) is made in a seven-year-old boy. He is


developmentally normal. Of the following, which complication of NF-1 is he most likely to
develop during childhood or adolescence?
A. Hypertension.
B. Malignant transformation of a neurofibroma.
C. Scoliosis.
D. Seizures.
E. Sensorineural deafness.

49. A full-term neonate is born following a pregnancy complicated by polyhydramnios. The


baby was hydropic, and died at six hours of age.
Which one of the following is least likely to have caused this?
A. Alpha-thalassaemia.
B. Beta-thalassaemia.
C. Blackfan-Diamond anaemia.
D. Fetal parvovirus B19 infection.
E. Rh disease of the newborn.

50. A 13-month-old male infant is admitted to hospital with acute gastroenteritis and
dehydration. Investigations included full blood count, electrolytes and liver function tests. All
are normal apart from a raised alkaline phosphatase of 1350 U/L [30-300]. An X-ray of the knee
was normal. The most likely diagnosis is:
A. Alagille syndrome.
B. cystic fibrosis.
C. osteogenesis imperfecta.
D. transient hyperphosphatasaemia.
E. vitamin D resistant rickets.

51. When counselling parents of a child born with tetralogy of Fallot, you are asked what the
risk is of having an infant with any form of congenital heart disease in future pregnancies. The
risk of recurrence in this family is closest to:
A. 0.5%.
B. 2.5%.
C. 5%.
D. 12.5%.
E. 25%.

52. An eight-month-old girl starts to wake repeatedly at night after settling well to sleep. She
is breastfed to settle once she wakes. This has been occurring for three weeks despite her
having slept through from six-months of age. Her growth and development are normal. She
eats a variety of solids and is breastfed five to six times during the day. Which of the following
is the most likely explanation for her behaviour?
A. Diminishing breast milk production.
B. Food allergy.
C. Gastroesophageal reflux.
D. Inappropriate sleep association.
E. Nightmares.

53. A previously well 13-year-old boy presents for investigation of persistent lip swelling and
bleeding gums for the past four months. For about 12 months he has experienced colicky
abdominal pains after eating. He has had no vomiting or diarrhoea but complains of pain on
defaecation, with some bright red blood on the toilet paper. He has lost 3 kg in weight over
the past six months. On examination he looks pale and has marked gingivitis and swollen upper
and lower lips. His abdomen is soft, without palpable organomegaly, but with mild tenderness
and fullness in the right iliac fossa. He has a perianal skin tag.
What is the most likely diagnosis?
A. Chronic granulomatous disease.
B. Crohn's disease.
C. Gastrointestinal tuberculosis.
D. Sarcoidosis.
E. Systemic lupus erythematosus

54. An eight-year-old boy presents with moderately severe cellulitis involving the left shin. The most
appropriate empiric antibiotic treatment would be:
A. benzylpenicillin plus flucloxacillin.
B. benzylpenicillin plus gentamicin.
C. cefotaxime.
D. flucloxacillin.
E. vancomycin.

55. An eight-year-old boy with attention deficit hyperactivity disorder (ADHD) and oppositional-defiant
disorder is being treated with stimulant medication, with benefits reported at both school and home.
What further functional improvement might be expected from adding intensive clinic-based
behavioural intervention to his stimulant medication therapy?
A. Improved compliance in the classroom.
B. Reduced hyperactivity.
C. Reduced impulsivity.
D. Reduced inattention.
E. Reduced oppositional/aggressive behaviour.
56. A 14-year-old girl presents to the emergency department with a four-day history of headache and
blurring of vision. She has previously been well. She is on no medication.
On examination she is very confused and disorientated. Her blood pressure is 230/140 mmHg. Her
jugular venous pressure (JVP) is not elevated, heart sounds are normal and lung bases clear. Her
reflexes are generally brisk. Fundoscopy shows bilateral haemorrhages, exudates and papilloedema.
Which of the following is the most appropriate treatment?
A. Intramuscular hydralazine.
B. Intravenous frusemide.
C. Intravenous sodium nitroprusside.
D. Oral enalapril.
E. Oral nifedipine.
57. A seven-year-old girl is referred with a history of increasing daytime wetting, characterised by loss
of urine control when laughing and particularly when being tickled. Her classmates are causing
embarrassment by tickling her in the playground. Her past history suggests no neurodevelopmental
problems. She had a single urinary tract infection at 18 months of age, followed by a normal renal
ultrasound. Physical examination is normal.
Which of the following is the most appropriate therapy?
A. Behavioural therapy.
B. Imipramine.
C. Oxybutynin.
D. Pelvic floor exercises.
E. Vasopressin spray.

58. A six-year-old boy presents with a five-week history of polyuria and polydipsia. There is no family
history of autoimmune disease. He has otherwise been well and has not lost weight.
Investigations are as follows:
blood glucose 4.3 mmol/L [3.6 – 5. 4]
electrolytes normal
urinalysis negative glucose, negative ketones, negative protein
Specific Gravity 1.001 [1.002 – 1.035]
Which of the following tests would confirm the diagnosis?
A. Antibodies for type 1 (insulin-dependent) diabetes mellitus.
B. Magnetic resonance imaging (MRI) of brain.
C. Oral glucose tolerance test.
D. Renal ultrasound.
E. Water deprivation test.

59. An otherwise normal two-year-old girl requires surgical repair of a large secundum atrial defect.
There is no other family history of congenital heart disease. Her parents are concerned about the risk
that their next child will also have congenital heart disease. This risk is closest to:
A. 0.5%.
B. 2.5%.
C. 6%.
D. 12.5%.
E. 25%.
60. A mother notices low-grade rectal bleeding in her breast-fed daughter. The three-week-old infant
passes four to five semi-formed stools per day with visible streaks of blood and some mucus. The infant
is well and thriving.
What is the most likely diagnosis?
A Anal fissure.
B. Bacterial gastroenteritis.
C. Food protein proctocolitis.
D. Juvenile polyp.
E. Swallowed maternal blood.
61. A 28-week gestation infant, intubated since birth for moderate respiratory distress syndrome, has
been steadily improving and is now 48 hours old. The level of ventilatory support has been progressively
weaned and is currently as follows:
Mode: Synchronised intermittent positive pressure ventilation (SIPPV) (assist control)
fractional inspired oxygen concentration 0.24
peak inspiratory pressure (PIP) 20 cm H2O
positive end-expiratory pressure (PEEP) 6 cm H2O
ventilator rate 30/minute
inspiratory time 0.35 seconds
The most recent arterial blood gas is as follows:
pH 7.43 mmHg [7.34-7.43]
PaCO2 31 mmHg [31-42]
PaO2 68 mmHg [45-60]
bicarbonate 20 mmol/L [20-26]
base excess -1.6 [-5.0-5.0]
What would be the most appropriate next step?
A. Extubate to nasal continuous positive airways pressure (CPAP).
B. Make no ventilator changes.
C. Reduce inspiratory time to 0.3 seconds.
D. Reduce PIP by 2 cm H2O.
E. Wean rate to 20/minute.

62. A 16-year-old girl is referred for evaluation of daytime tiredness. She is reported to go to bed at
1.00 a.m. and to have difficulty rising in the morning for school. At weekends she sleeps until early
afternoon. No medical or psychiatric symptoms are detectable.
In addition to gradually advancing her bedtime to an earlier time, bright light therapy is recommended.
This is most effective if undertaken at which of the following time periods?
A. Early morning.
B. Late morning.
C. Mid afternoon.
D. Evening.
E. Prior to retiring.

63. A nine-year-old girl with cystic fibrosis presents to clinic with a cough productive of brown sputum.
Her chest X-ray is shown opposite. The most likely diagnosis is:
A. allergic bronchopulmonary aspergillosis.
B. atypical mycobacterium infection.
C. Burkholderia cepacia infection.
D. Staphylococcus aureus infection.
E. Stenotrophomonas maltophilia infection.

64. An eight-year-old girl presents with ketoacidosis and is diagnosed with type 1 (insulin-dependent)
diabetes mellitus.
Which of the following autoimmune comorbidities is she most at risk of developing?
A. Addison disease.
B. Hypoparathyroidism.
C. Hypothyroidism.
D. Primary ovarian failure.
E. Vitiligo.

65. A three-year-old boy is brought to the emergency department of a small rural hospital 30 minutes
after being involved in an unwitnessed explosion at his family home. On examination he has sustained
facial burns with singeing of his eyebrows and eyelashes. He has a non-circumferential burn to the
anterior neck and his estimated percentage of body surface area (BSA) burnt is 5%. He is alert, pink in
air and in no respiratory distress. His parents feel that his voice is now ‘croaky’ and you notice a hoarse
cough.
Which of the following is the first priority in caring for this patient?
A. Commence intravenous fluids at maintenance.
B. Commence intravenous morphine infusion.
C. Prepare for endotracheal intubation.
D. Urgent ambulance transfer to a tertiary level paediatric hospital.
E. Urgent medical retrieval team to come to the patient.

66. A term male infant with a birth weight of 2800 g is born at a level 2 rural hospital. He is noted to
have a large cleft palate and significant micrognathia. At two hours of age he is noted to be in severe
respiratory distress with marked subcostal and sternal recession. On auscultation, very poor breath
sounds are audible bilaterally. His condition is improved in the prone position, however he becomes
intermittently cyanosed and severe respiratory distress is punctuated by brief apnoeic episodes.
The most appropriate course of action would be:
A. administer head box oxygen.
B. administer oxygen via nasal prongs.
C. insert an endotracheal tube.
D. insert a nasopharyngeal tube.
E. insert an oral airway.

67. A three-year-old girl is admitted to hospital with a two-day history of purpuric rash on the buttocks.
Over the last day, she has also developed a painful, swollen left ankle.
On examination she has a temperature of 38°C, palpable purpura on her buttocks and calves, and a
warm, swollen left ankle. Her hands and feet are slightly swollen.
Admission investigations show:
haemoglobin 120 g/L [115-140]
9
white cell count 11.5 x 10 /L [4.0-15.0]
9
platelets 410 x 10 /L [150-450]
ESR 10 mm/hr [1-10]
C-reactive protein (CRP) 10 mg/L [< 3]
international normalized ratio (INR) 1.0 [0.8-1.2]
activated partial thromboplastin time (APTT) 30 seconds [25-38]
urinalysis normal
Over the next two days she develops intermittent, severe, colicky central abdominal pain. She has non-
bilious vomiting and passes blood per rectum. In between bouts of pain her abdomen is soft,
nondistended and generally tender with no guarding. There are no masses palpable.
The most appropriate next investigation is:
A. abdominal ultrasound.
B. air enema.
C. barium enema.
D. diagnostic laparoscopy.
E. stool culture.

68. What is the most common sign of Fragile X syndrome in prepubertal boys?
A. High-arched palate.
B. Long face.
C. Macro-orchidism.
D. Motor tics.
E. Poor eye contact.

69. A nine-year-old boy is brought in by his mother, with a story of increasingly difficult behaviour at
home, and especially at school. He is in grade three. He is disruptive, calls out, distracts other children,
and consistently fails to complete written work. He has been lashing out at peers and the teacher, and
spends most of his lunchbreaks in class to catch up on work. His mother reports that his behaviour in
preschool and grade one presented few problems. He shows strengths in maths and science, but
struggles with reading and written work. He is left handed, and his handwriting is particularly messy.
When made to repeat work, he becomes angry and has occasionally abused his teacher. At home,
there are constant battles to get him to sit down in the evening to do his homework with his sister. He
constantly interrupts his sister, and intentionally destroys her work.
The most likely primary diagnosis is:
A. attention deficit disorder (without hyperactivity).
B. autism spectrum disorder.
C. intellectual impairment.
D. oppositional defiant disorder.
E. specific learning disorder.

70. Middle ear effusion is an essential criterion for the diagnosis of both acute otitis media and otitis
media with effusion. Compared to myringotomy as the gold standard, which diagnostic technique is
most accurate (i.e. has the highest sensitivity and specificity)?
A. Acoustic reflectometry.
B. Audiometry.
C. Otoscopy.
D. Pneumatic otoscopy.
E. Tympanometry.

71. A four-week-old infant presents with persistent diarrhoea. On examination the infant displays
swollen upper eyelids and ascites. His investigations are shown below.
Faecal microscopy:
fat globules +++
fatty acid crystals negative
white blood cells negative
Full blood count:
haemoglobin 130 g/L [90-180]
9
white cell count 5.5 x 10 /L [5.0-19.5]
9
neutrophils 5.0 x 10 /L [1.0-9.0]
9
lymphocytes 0.4 x 10 /L [2.5-9.0]
serum albumin 18 g/L [29-45]
Which of the following is the most likely diagnosis?
A. Alpha-1-antitrypsin deficiency.
B. Coeliac disease.
C. Cystic fibrosis.
D. Intestinal lymphangiectasia.
E. Nephrotic syndrome.

72. A 16-year-old girl is brought to the emergency department by her mother following the ingestion
of an unknown quantity of paracetamol. Four hours post-ingestion her paracetamol serum levels are
non-toxic. She is poorly communicative but states that she does not feel life is worth living anymore
and that she had hoped she would be able to ‘end it all’. Her mother states that the girl has attempted
suicide previously and that she is currently under the care of a private psychiatrist with whom she has
an appointment in one week. The mother feels the girl is overly histrionic and attention seeking. The
girl is attempting to leave the department.
Which of the following is the most appropriate next step in management?
A. Detain under the mental health act.
B. Discharge home in care of her mother.
C. Refer back to private psychiatrist as planned.
D. Refer on to community mental health team semi-urgently.
E. Urgent psychiatric review in the emergency department.

73. Which one of the following confers the greatest risk for the development of fungal infection in a
patient undergoing chemotherapy?
A. Central venous lines.
B. Long-term antibiotics.
C. Prolonged severe neutropenia.
D. Use of monoclonal antibodies.
E. Use of steroids.

74. An eight-year-old boy is referred to you with tiredness and short stature. He has a long history of
thirst and nocturia. There is no family history of note. On examination he is short compared with his
five-year-old sister. Apart from obvious pallor, there are no other abnormal physical features.
Investigations show the following:
mid-stream urine 50 leucocytes
20 red cells
trace protein
no bacterial growth
haemogloblin 74 g/L [115-155]
serum creatinine 0.20 mmol/L [0.03-0.07]
What is the most likely diagnosis?
A. Familial juvenile nephronophthisis.
B. Medullary cystic disease.
C. Polycystic kidney disease.
D. Reflux nephropathy.
E. Renal Fanconi syndrome.

74. A three-week-old male infant is brought to the emergency department with a left sided groin lump
noticed by his parents during a nappy change today. On examination the baby is afebrile, thriving and
in no distress, and has an easily reducible left sided inguinal hernia.
Which of the following is the most appropriate next step in patient care?
A. Elective surgical repair after 12 months of age.
B. Elective surgical repair after two years of age.
C. Reassure parents of likely spontaneous resolution prior to 12 months of age.
D. Semi-urgent operative repair in a week.
E. Urgent operative repair.

75. A four-year-old boy, who started talking at around 18 months, has speech that is difficult for
strangers to understand. His family can understand him around 80% of the time. He seems to follow
instructions well, but consistently leaves sounds out of his words. He shows frustration when other
children make fun of his speech.
When tested, he can produce all consonant sounds by themselves, but when he looks at a picture book
he makes the following errors:
Car > dar
Book > boo
Sun > dun
Hat > ha
Fishing > bittin
Five > bye
The most likely cause of his speech disorder is:
A. dysarthria.
B. dysphonia.
C. hearing impairment.
D. intellectual impairment.
E. phonological delay.

76. A previously well 10-month-old infant is admitted to hospital with a four-day history of increasing
cough, fever and rapid breathing. Chest X-ray shows a right upper lobe pneumonia and he is started on
intravenous antibiotics. Blood cultures grow Streptococcus pneumoniae. By day two his fever has
settled, but on day five he is noted to be rather pale and lethargic.
Blood tests show the following:
haemogloblin 75 g/L [105-135]
9
white cell count 6.2 x 10 /L [6.0-15.0]
9
platelet count 50 x 10 /L [150-400]
serum creatinine 0.23 mmol/L [0.02-0.06]
What is the most likely diagnosis?
A. Acute lymphoblastic leukaemia.
B. Aplastic crisis.
C. Autoimmune haemolytic anaemia.
D. Drug-induced haemolysis.
E. Haemolytic-uraemic syndrome.

77. A six-month-old child presents with high fever and poor feeding. Which of the following findings in
the child would be the most valid reason for not performing a lumbar puncture?
A. Bulging fontanelle.
B. Glasgow coma score of 12.
C. Inability to abduct the right eye.
D. Marked irritability.
E. Petechial rash.

78. A seven-year-old girl has a six-week history of urticaria occurring most days. She is otherwise well.
She had an allergy to eggs when an infant. There is a family history of asthma and a maternal aunt has
lupus erythematosus. Antinuclear antibody testing was positive with a titre of 1 in 80, with a speckled
pattern.
The most likely cause of her urticaria is:
A. autoimmune connective tissue disease.
B. autoimmune thyroid disease.
C. IgE-mediated food allergy.
D. recent viral infection.
E. sensitivity to food chemicals and preservatives.

79. A 14-year-old girl presents with jaundice, arthralgia and pruritus over the past week. She has
recently returned from a trip to South-East Asia.
Her liver function and serology results are shown below:
bilirubin 175 μmol/L [0-15]
alanine aminotransferase (ALT) 1350 U/L [<55]
alkaline phosphatase (ALP) 687 U/L [100-350]

gamma glutamyltransferase (GGT) 425 U/L [0-40]


total protein 70 U/L [57-80]
albumin 24 g/L [33-47]
Epstein-Barr Virus (EBV)-IgG positive
Epstein-Barr Virus (EBV)-IgM negative
hepatitis A virus (HAV)-IgM negative
hepatitis B surface antigen (HBsAg) negative
anti-hepatitis B core antibody (anti-HBc) positive
anti-hepatitis C virus antibody (anti-HCV) negative
antinuclear antibody (ANA) negative
smooth muscle antibody positive
What is the most likely diagnosis?
A. Acute hepatitis A.
B. Acute hepatitis B.
C. Autoimmune hepatitis.
D. Epstein-Barr virus hepatitis.
E. Systemic lupus erythematosus.

80. An eight-week-old girl presents for investigation of jaundice. She was born at term, and her birth
weight was 3800 g. She is breast-fed, thriving and appears generally well and alert. Examination of the
heart is normal. Her mother describes the infant’s stools as mustard-coloured to three weeks of age,
but becoming very pale over time. The gallbladder is not seen on two fasting abdominal ultrasounds,
but normal intrahepatic ducts are seen.
Which of the following is the most likely diagnosis?
A. Alagille syndrome.
B. Alpha-1-antitrypsin deficiency.
C. Biliary atresia.
D. Choledochal cyst.
E. Gallstones.

81. A four-year-old boy with a hereditary anaemia presents with progressive pallor and lethargy of one-
week duration. On physical examination he is normotensive but tachycardic and has moderate
splenomegaly but no clinical jaundice. The following blood tests are performed:
haemoglobin 41 g/L [115-155]
9
white cell count 12.1 x10 /L [4.5-14.5]; normal differential
9
platelets 175 x10 /L [150-400]
reticulocytes 1% [<2%]
Which one of the following viruses is most likely to be responsible for the boy’s clinical presentation?
A. Cytomegalovirus (CMV).
B. Epstein-Barr virus (EBV).
C. Hepatitis B virus (HBV).
D. Human immunodeficiency virus (HIV).
E. Parvovirus B19.

82. An infant is born at term by normal vaginal delivery. A cloudy cornea of the right eye only is noted
at initial examination. The most likely diagnosis is:
A. Chlamydia trachomatis infection.
B. congenital glaucoma.
C. herpes simplex virus (HSV) infection.
D. mucopolysaccharidosis.
E. retinoblastoma.

83. Antenatal maternal smoking is least associated with which one of the following effects in children?
A. Decreased lung compliance.
B. Decreased maximal expiratory flow.
C. Increased risk of lower respiratory tract infections.
D. Increased risk of sudden infant death syndrome.
E. Increased risk of wheezing.

84. A nine-month-old girl is referred by her general practitioner because of recurrent events occurring
on a daily basis over the last two weeks. The episodes are stereotyped and consist of her stopping what
she is doing, flexing at her trunk, and pressing her hands above her inguinal region. There is associated
tremulousness and jaw rigidity. The events last one to two minutes with her becoming red in the face
and grunting. She seems to be preoccupied and gets distressed if touched or moved. After the event,
the child goes to sleep. The events never occur in sleep.
The most likely diagnosis is:
A. dystonia.
B. frontal lobe seizures.
C. gastroesophageal reflux.
D. infantile masturbation.
E. urinary infection.

85. A 13-year-old boy presents with nonspecific abdominal pain. The boy has a history of seizures. His
mother is known to have chronic renal failure. Physical examination shows a small 13-year-old who is
normotensive with small areas of hypopigmentation on his trunk. The abdominal pain quickly subsides
following hospital admission. A renal ultrasound scan shows enlarged kidneys and multiple echogenic
foci throughout both kidneys.
Which one of the following is the most likely explanation of the renal abnormality in this boy?
A. Dominant polycystic kidney disease.
B. Juvenile nephronophthisis.
C. Papillary necrosis.
D. Recessive polycystic kidney disease.
E. Renal angiomyolipomata.

86. A nine-year-old girl is seen with polyuria and polydipsia. She is dehydrated and on routine
assessment is found to have the following blood test results:
blood glucose 38.2 mmol/L [3.8-6.2]
sodium 130 mmol/L [134-143]
potassium 4.2 mmol/L [3.5-5.6]
pH 7.12 [7.35-7.45]
bicarbonate 9 mmol/L [18-29]
creatinine 0.035 mmol/L [<0.074]
The most likely explanation for the low serum sodium level is:
A. Addison disease.
B. diabetes insipidus.
C. malnutrition.
D. pseudohyponatraemia.
E. renal sodium losses.
87. A 4800 g female infant is delivered by emergency caesarean section following an obstructed labour.
Her mother’s pregnancy had been uncomplicated. The infant is in an excellent condition at delivery. A
true blood glucose performed at 10 minutes of age is 1.5 mmol/L [<2.5]. In the management of the
infant’s hypoglycaemia, the most appropriate next step is:
A. administer 10% dextrose intravenously at 12.0 mL/hr.
B. administer 10% dextrose intravenously at 20.0 mL/hr.
C. administer intramuscular glucagon.
D. commence feeds at 36 mL per feed three hourly.
E. insert a nasogastric tube and commence 5% dextrose at 12 mL/hr.
88. A 13-month-old boy presents to the emergency department with a one-week history of intermittent
vomiting and increasing lethargy. Over the preceding day his parents feel he has had increasing
respiratory difficulty. His examination reveals deep sighing respiration with severe dehydration. He is
afebrile with acute otitis media. The initial blood chemistry is demonstrated.
sodium 133 mmol/L [133-143]
potassium 4.8 mmol/L [3.8-6.0]
chloride 101 mmol/L [95-110]
bicarbonate 5.0 mmol/L [18-24]
anion gap 31.8 [< 18.0]
glucose 29.5 mmol/L [3.5-5.5]
pH 7.09 [7.36-7.44]
pCO2 22 mmHg [35-45]
base excess -23 mmol/L [0-2]
Which of the following is the most appropriate next step in management?
A. Intravenous insulin infusion.
B. Intravenous fluids – 5% dextrose.
C. Intravenous fluids – 0.9% saline.
D. Intravenous fluids – 0.45% saline + 2.5% dextrose.
E. Subcutaneous insulin injection.
89. A normal but shy 12-year-old girl with overly protective parents presents with recurrent abdominal
pain. She has missed school for the last four months. She is occasionally constipated. Clinical
examination is unremarkable. Baseline investigations, including abdominal X-ray, abdominal ultrasound
and urinary examination, are also normal. The clinical treatment or factor most likely to influence the
duration of her symptoms is:
A. parental insight and encouragement of independence.
B. prescription of fluoxetine.
C. prescription of lactulose.
D. prescription of pizotifen.
E. self management / pain distraction strategies.
90. A two-and-a-half-year-old boy is referred with concerns regarding aggressive behaviour. He often
appears “blank” when spoken to, and is non-compliant with instructions. He has 10 to 20 single words,
and a couple of two-word combinations. There were a few words he used some months ago that he is
no longer saying. He likes being with other children but does not play cooperatively, frequently fighting
over toys. He is good at puzzles, can work the DVD player at home, and particularly likes trains.
He is active and difficult to engage. He does not carry out your instruction to “give the book to mummy”.
He points to two of four named body parts. When given a crayon and invited to copy a circle he scribbles
on the paper. He plays with the train set in your office and seems to quickly make up an elaborate
game, chatting incomprehensibly to himself. Physical examination and audiology assessment are
normal.
What is the most likely primary diagnosis?
A. Acquired epileptic aphasia.
B. Attention deficit hyperactivity disorder (ADHD).
C. Autistic spectrum disorder.
D. Intellectual disability.
E. Specific language impairment.
91. A child presents with a sore throat. Which of the following clinical features is least likely in Group A
streptococcal tonsillitis?
A. Age 10 years.
B. Cough.
C. Exudate.
D. Fever >39°C.
E. Tender enlarged cervical lymph nodes.
92. A three-year-old girl was diagnosed with nephrotic syndrome three months ago. She was initially
responsive to steroids but quickly relapsed when steroids were discontinued. She has now been on
daily prednis(ol)one (2 mg/kg) for 30 days without a significant improvement in her proteinuria. Her
renal function and blood pressure remain normal. Her renal biopsy is consistent with minimal-change
disease.
Which of the following is the most appropriate next treatment?
A. Azathioprine.
B. Chlorambucil.
C. Cyclophosphamide.
D. Levamisole.
E. Vincristine.
93. A 10-week-old baby boy has been failing to thrive since birth. His birth weight was 3300 g. He now
weighs 3400 g. He is breastfed and is reported to feed well.
Investigations are as follows:
plasma sodium 122 mmol/L [134 – 143]
plasma potassium 6.8 mmol/L [3.4 – 5.0]
urea 6.9 mmol/L [< 6.0]
creatinine 0.060 mmol/L [< 0.062]
17-hydroxyprogesterone 1.7 mmol/L [< 5.2]
Which of the following is the most likely diagnosis?
A. Adrenal hypoplasia.
B. Congenital adrenal hyperplasia.
C. Inappropriate antidiuretic hormone (ADH) secretion.
D. Posterior urethral valves.
E. Urinary tract infection.
94. A 13-year-old girl presents with a fractured ankle requiring surgical fixation.
Full blood examination shows:
haemoglobin 105 g/L [120-160]
12
red cell count 5.6 x 10 /L [4.0-5.7]
mean cell volume 62 fL [80-97]
9
white cell count 12.5 x 10 /L [4.0-11.0]
9
platelet count 390 x 10 /L [150-400]
Haemoglobin (Hb) studies:
HbA2 5.2% [1.8-3.5]
HbF 1.2% [0-2.0]
HbH preparation no HbH bodies seen
Hb electrophoresis no abnormal Hb bands seen
The most likely explanation for her anaemia is:
A. α-thalassaemia trait.
B. β-thalassaemia trait.
C. congenital sideroblastosis.
D. iron-deficiency anaemia.
E. sickle cell trait.
95. A male infant is found to have supravalvular aortic stenosis. What is the best way of confirming
whether he has Williams syndrome?
A. Detailed clinical examination.
B. FISH for microdeletion on 7p.
C. Karyotype.
D. Measurement of plasma calcium.
E. TBX1 mutation analysis
96. A six-month-old boy is taken by ambulance to the emergency department after an episode of
collapse and unresponsiveness at home. He presents with profuse vomiting and watery diarrhoea
which contains a small amount of blood. On arrival in hospital his heart rate is 160/minute, and his
blood pressure 60/30 mmHg. He is afebrile, his breathing is normal, and he has no rashes. He is
resuscitated with intravenous fluids. He was exclusively breast-fed until two months and was then
changed to a cow’s milk formula. He commenced a range of weaning solids from five months of age,
and had tried chicken meat for the first time one hour before his collapse. His arterial blood gas analysis
is shown below.
pH 7.1 [7.34-7.43]
bicarbonate 12 mmol/L [20-26]
base excess -8 [-5-5]
Which of the following is the most likely diagnosis?
A. Bacterial gastroenteritis.
B. Food anaphylaxis.
C. Food protein-induced enterocolitis.
D. Fulminant septicaemia.
E. Intussusception.
97. An eight-year-old boy is admitted with an episode of acute asthma. His oxygen saturation is 92% in
air. He is prescribed prednisolone 1 mg/kg and hourly salbutamol 12 puffs. Three hours later his oxygen
saturation in air has dropped to 89%. He looks well and is less distressed than when admitted. He has
widespread wheeze with good air entry.
Based on these findings, the most appropriate next step in his management is:
A. add ipratroprium bromide.
B. blood gas analysis.
C. change to intravenous salbutamol.
D. chest X-ray to rule out a pneumothorax.
E. reduce frequency of salbutamol.
98. A one-year-old boy has infection with Enterobius vermicularis (threadworm/pinworm). Which of the
following is the most appropriate first-line treatment?
A. Co-trimoxazole.
B. Ivermectin.
C. Metronidazole.
D. Permethrin.
E. Pyrantel.
99. A toddler is admitted following ingestion of a strong alcoholic drink. The most common serious
complication is:
A. acute hepatitis.
B. cerebral oedema.
C. dehydration.
D. hypoglycaemia.
E. pancreatitis.
100. A seven-year-old boy presents with a four-month history of dry hacking cough which is maximal in
the morning and resolves during sleep. A sibling has asthma and his parents are very anxious about the
aetiology of the cough. Pulmonary function testing demonstrates a forced vital capacity (FVC) of 90%
predicted, forced expiratory volume in 1 second (FEV1) of 86% predicted, and maximum mid-expiratory
flow rate (MMEF) of 62% predicted. His chest X-ray is normal. A histamine challenge test shows a 14%
fall in FEV1.
The most appropriate next step in management is:
A. antibiotics.
B. behavioural therapy.
C. bronchoscopy.
D. high resolution computerised tomography (CT) scan of chest.
E. inhaled steroids.
101. Which of the following is least consistent with night terrors?
A. Amnesia of the episode by the child.
B. Family history.
C. Occurrence in rapid eye movement (REM) sleep.
D. Occurrence in the first third of the night.
E. Onset at five years of age.
102. A 12-year-old girl with type 1 (insulin-dependent) diabetes mellitus presents with a 12-month
history of episodes of diarrhoea, flatulence and intermittent dull abdominal pains. Her insulin
requirements over the past year have fluctuated significantly. She noticed that drinking cow’s milk
increases her abdominal symptoms. She has not previously had any gastrointestinal problems, and her
weight has remained stable.
Which of the following investigations will most likely reveal the correct diagnosis?
A. Colonoscopy.
B. Lactose breath hydrogen test.
C. Serum IgE antibody to cow’s milk protein.
D. Small bowel biopsy.
E. Stool microscopy for parasites.
103. What is the most common motive given by adolescents who have deliberately cut themselves?
A wish to:
A. die.
B. get my own back on someone.
C. get relief from a terrible state of mind.
D. punish myself.
E. see if someone really loved me.
104. A term male infant is delivered vaginally following a vacuum extraction which results in a small
cephalhaematoma. There is no family history of significance. At birth the infant is in good condition
with Apgar scores of 7 at one minute and 9 at five minutes. His birth weight is 3600 g and head
circumference is 35.5 cm. He establishes breastfeeding very well and is discharged to home at 72 hours
of age.
At six days of age, he becomes irritable with poor feeding. He develops severe non-bile-stained vomiting
and presents to the emergency department where he is noted to be pale, apnoeic and hypotonic. His
head circumference is 37 cm and his fontanelle is tense. Following intubation and ventilation, the
following investigations are performed:
haemoglobin 85 g/L [140-200]
9
white cell count 18.7 x10 /L [6.0-18.0]
9
platelets 252 x x10 /L [150-400]
Coagulation screen:
international normalised ratio (INR) 1.3 [0.8-1.5]
activated partial thromboplastin time (APTT) > 200 seconds [27-69]
fibrinogen 3.0 g/L [1.9 – 4.3]
Cranial ultrasound shows no abnormalities.
The most likely diagnosis is:
A. disseminated intravascular coagulation.
B. haemophilia A.
C. haemorrhagic disease of the newborn.
D. herpes encephalitis.
E. non-accidental injury.
105. A 10-day-old male neonate presents with fever associated with an inguinal node abscess.
Following surgical incision and drainage, cultures grew Staphylococcus aureus. He is treated with
flucloxacillin for 14 days with full recovery. However, he has persistent neutropenia (absolute
neutrophil count (ANC) < 100). The most likely cause of this clinical picture is:
A. chronic benign neutropenia.
B. cyclic neutropenia.
C. post-viral neutropenia.
D. severe congenital neutropenia.
E. Shwachman-Diamond syndrome.
106. A ten-year-old boy presents with facial swelling, hypertension, proteinuria and haematuria.
Complement levels are 0.7 mg/dL [0.8-1.8]; antistreptolysin-O titre (ASOT) and anti-DNase are positive.
He is excreting 500 mg of urinary protein in 24 hours [<200]. Anti-nuclear antibodies are negative, and
he has no other symptoms or signs suggesting autoimmune phenomena. After initial treatment with
fluid restriction, diet and antihypertensives, his urinary sediment improves, and blood pressure returns
to normal. At one month he is reassessed.
Which of the following would most strongly indicate the need for renal biopsy?
A. Creatinine of 0.11 mmol/L [0.05 – 0.10].
B. Haematuria.
C. Hypertension.
D. Hypocomplementaemia.
E. Proteinuria of > 200 mg/24 hours.
107. An eight-week-old formula-fed baby is referred because he has been crying excessively since four
weeks of age. He pulls up his legs, arches his back and goes red in the face. He is most distressed in the
evenings, but can cry at any time of day.
He possets after feeds but has not been vomiting, has had no diarrhoea or eczema, and is thriving.
What is the most appropriate first step?
A. Change to a cow’s milk-free formula.
B. Change to a lactose-free formula.
C. Discuss normal infant sleep and cry patterns.
D. Start anti-reflux therapy.
E. Test for urinary tract infection.
108. Which one of the following, occurring in the first year of life, is most indicative of atopy?
A. Chronic rhinitis.
B. Chronic serous otitis media.
C. Eczema.
D. Recurrent wheeze.
E. Urticaria.
109. A nine-year-old girl presents with obesity. Her height is on the 10th percentile, weight on

the 97th percentile and she has a round face. Shortening of the fourth metacarpals bilaterally

are found and confirmed by X-ray.

Which finding would be most helpful in establishing a diagnosis of pseudohypoparathyroidism?


A. Basal ganglia calcification.
B. Increased fasting plasma insulin/glucose ratio.
C. Increased plasma calcium.
D. Increased serum alkaline phosphatase.
E. Increased serum parathyroid hormone.
110. An 18-month-old girl presents with a history of persistent diarrhoea and poor weight gain
since eight months of age. Her weight has progressively fallen from the 50th percentile to
below the 3rd percentile. Over the past month her abdomen has become increasingly
distended and on physical examination she has evidence of ascites. Faecal microscopy reveals
large amounts of fat globules. Her laboratory findings are shown below.
Haemoglobin 125 g/L [105-135]
9
White cell count 7.1 x 10 /L [6.0-18.0]
differential:
9
neutrophils 6.4 x 10 /L [1.0-8.5]
9
lymphocytes 0.4 x 10 /L [4.0-10.0]
9
eosinophils 0.3 x 10 /L [0-0.8]
9
Platelet count 355 x 10 /L [150-400]
Serum IgG 2.14 g/L [2.71-13.78]
Serum IgM 0.42 g/L [0.35-1.35]
Serum IgA <0.07 g/L [0.17-1.34]
Which one of the following is the most likely diagnosis?
A. Chronic liver disease.
B. Coeliac disease.
C. Common variable immunodeficiency.
D. Congenital cytomegalovirus infection.
E. Intestinal lymphangiectasia.
111. A two-day-old neonate presents with copious purulent eye and nose discharge.
Laboratory examination is performed to diagnose the organism involved.
Which one of the following types of organism would carry the worst prognosis if not treated
immediately?
A. Gram-negative coccobacillus.
B. Gram-negative diplococcus.
C. Gram-positive bacillus.
D. Gram-positive coccus.
E. Intra-cytoplasmic inclusion bodies.
112. A three-year-old boy is referred with a history of snoring loudly over the previous nine
months. On history no obstructive apnoea has been noted by his mother. A lateral X-ray of
neck shows only moderate adenoidal hypertrophy with a clearly patent airway. A
polysomnogram is reported as showing an obstructive respiratory disturbance index of 5
events/hr with oxygen desaturation decreases of 4% of baseline, and an arousal index of 20
events/hr [<10] with reduced rapid eye movement (REM) sleep.
The most appropriate management of this child is:
A. adeno/tonsillectomy.
B. continuous positive airway pressure therapy.
C. inhaled nasal steroids.
D. reassurance.
E. supplemental oxygen.
113. A four-month-old boy presents with significant oozing of blood from the wound following

a talipes repair. He is fully breastfed and previously well with no individual or family history of

bleeding.

Investigations show:

haemoglobin 45 g/L [95-140]

prothrombin time-international normalised ratio (PT-INR) 1.2 [1.0-1.4]

activated partial thromboplastin time (APTT) 120 seconds [25-35]

The most likely diagnosis is:

A. factor VIII deficiency.


B. factor X deficiency.
C. factor XI deficiency.
D. vitamin K deficiency.
E. von Willebrand disease.
114. In which one of the following clinical settings is there the best evidence that prophylactic
antibiotics protect against infection?
A. Cerebrospinal fluid shunt insertion.
B. Chest drain insertion for pneumothorax.
C. Occipital skull fracture.
D. Umbilical artery catheterisation.
E. Urinary catheterisation.
115. Severe congenital bilateral hearing loss is diagnosed in the first-born child of first cousin
parents. Both parents are Caucasian. Which one of the following investigations is most likely
to reveal the underlying cause of this child’s deafness?
A. Electrocardiogram (ECG).
B. Measurement of inner canthal distance.
C. Polymerase chain reaction (PCR) for common Connexin 26 mutation.
D. Rubella titres (IgG and IgM).
E. Thyroid function testing.
116. A five-year-old boy presents to hospital with a four-day history of lethargy and dark
coloured urine. On examination his blood pressure is 130/90 mmHg. He has mild periorbital
oedema and his neck veins are easily visible. No neurological abnormalities are detected. His
mid stream urine reveals >1000 red cells, few red and white cell casts and 3+ protein on
albustix.
His initial blood results show the following:
haemoglobin 100 g/L [110-145]
plasma sodium 136 mmol/L [135-146]
plasma potassium 5.6 mmol/L [3.5-5.0]
plasma urea 30.0 mmol/L [1.8-5.5]
plasma creatinine 0.38 mmol/L [0.04-0.07]
serum complement C3 0.15 g/L [0.80-1.80]
Which one of the following is the most appropriate initial management?
A. Albumin infusion.
B. Fluid restriction.
C. Glucose and insulin infusion.
D. Intravenous diazoxide.
E. Protein restriction.
117. A six-year-old boy presents with a history of being difficult to manage in class because of

fidgetiness and poor concentration that disturbs other class members, and outbursts of poor

temper with apparently little provocation. His learning is progressing erratically, though there

are periods where he appears to concentrate well.

A medical history reveals that he has had ear infections in the past treated with drainage tubes,

without hearing impairment. He snores at night and tends to wake up in a poor mood in the

morning. He is on no current medication.

The most appropriate next step in management is:


A. behavioural modification.
B. psychometric assessment with special education planning.
C. refer for sleep studies.
D. stimulant medication.
E. stimulant medication and behavioural modification.
118. A 14-year-old girl presents to the emergency department with a 12-hour history of a
blistering rash. The rash is extremely itchy and is spreading up her arm. She is otherwise well
and afebrile. A history is obtained that she had been gardening on the day prior to the rash
developing. A diagnosis of an allergic contact dermatitis is made.
Which one of the following is the most appropriate initial therapy?
A. Intravenous antihistamines.
B. Oral antihistamines.
C. Oral corticosteroids.
D. Subcutaneous adrenaline.
E. Topical corticosteroids.
119. A two-year-old girl is referred because of breast development for the past six months.
There is no history of a growth spurt and she has not been on any medications. Her height and
weight are on the 25th percentile. She has Tanner stage 3 breast development, 3 cm in
diameter, and pubic hair stage 1. There are no skin lesions and her neurological examination is
normal.
Relevant investigations show:
luteinising hormone 0.3 U/L [0.1-1.0]
follicle-stimulating hormone 1.1 U/L [0.3-1.1]
oestradiol 40 pmol/L [30-60]
bone age 2.5 years
Pelvic ultrasound:
uterus:cervix ratio 0.95 (prepubertal <1)
left ovary 1.2 mL (prepubertal volume <3 mL)

right ovary 2.5 mL with several 5 mm follicles


The most likely diagnosis is:
A. granulosa cell tumour of the ovary.
B. idiopathic precocious puberty.
C. idiopathic premature adrenarche.
D. idiopathic premature thelarche.
E. McCune-Albright syndrome.
120. A three-month-old baby with vomiting and profuse watery diarrhoea presents 10%
dehydrated. The following results are obtained:
serum:

sodium 160 mmol/L [135-145]

potassium 3.9 mmol/L [3.5-4.5]

urea 9.6 mmol/L [1.0-6.0]


glucose 13.8 mmol/L [4.0-8.0]
blood pressure 75 mmHg systolic
A stool sample is positive for rotavirus.
The most appropriate fluid management is:
A. 0.18% sodium chloride plus 4.3% dextrose plus 20 mmol/L potassium chloride intravenously

(IV) to rehydrate over 48 hours.

B. 0.45% sodium chloride plus 5.0% dextrose plus 20 mmol/L potassium chloride IV to
rehydrate over 24 hours.
C. 0.45% sodium chloride plus 5.0% dextrose plus 20 mmol/L potassium chloride IV to
rehydrate over 48 hours.
D. intensive breastfeeding.
E. oral rehydration fluid given nasogastrically to rehydrate over 24 hours.
121. An 18-month-old boy has recurrent blue breath-holding episodes, followed on one
occasion by a 15-second generalised seizure.
The investigation most likely to be useful in directing therapy is:
A. electrocardiogram (ECG).
B. electroencephalogram (EEG).
C. iron studies.
D. plasma calcium.
E. plasma glucose.
122. There is strongest evidence for an association between measles-mumps-rubella (MMR)
vaccine and which one of the following adverse events?
A. Autism.
B. Crohn disease.
C. Encephalitis.
D. Guillain-Barré syndrome.
E. Multiple sclerosis.
123. A 24-hour-old infant is found in the postnatal ward with poor perfusion and significant
respiratory distress.
Which one of the following would be the best indication to start a prostaglandin infusion?
A. Active praecordium.
B. Diminished pulses.
C. Heart murmur.
D. Hepatomegaly.
E. Tachycardia.
124. A three-year-old boy presents with a one-week history of bruising and severe, persistent
epistaxis for 24 hours. Two weeks ago he had a mild illness consisting of a sore throat and low
grade fever for which he took aspirin on one occasion. Clinical examination was unremarkable
except for pallor, cutaneous bruising and fresh blood in the nasal antra. Full blood count shows:
haemoglobin 80 g/L [110-145]
9
white cell count 6 x 10 /L [5-17]
9
platelet count 8 x 10 /L [150-400]
Which one of the following is the most appropriate next step in management?
A. Intravenous gamma globulin.
B. Intravenous vincristine.
C. Oral prednisolone.
D. Platelet transfusion.
E. No treatment.
125. A 40-year-old pregnant woman and her partner choose to have an amniocentesis. The
karyotype of the foetus is 47,XXY.
Which one of the following is the most likely to occur in this condition?
A. Gynaecomastia.
B. Homosexuality.
C. Infertility.
D. Intellectual disability.
E. Tall stature (>97th percentile).
126. Tacrolimus administration after renal transplantation is least likely to cause which of the
following?
A. Diabetes mellitus.
B. Hypercholesterolaemia.
C. Hypomagnesaemia.
D. Seizures.
E. Tremors.
127. A seven-year-old girl is referred to the outpatients department for the evaluation of
wheezing episodes which have occurred every month during the previous three months. She
was born at 24 weeks gestation, ventilated for one month and was on supplemental oxygen
for a further 12 weeks. Her clinical examination is normal. Her baseline lung function tests
showed the following:
forced vital capacity (FVC) 80% predicted
forced expiratory volume in one second (FEV1) 70% predicted
maximum mid-expiratory flow (MMEF) 30% predicted
She was treated with prednisolone for two weeks (1 mg/kg/day). Repeat lung function tests
post Ventolin were:
FVC 83% predicted
FEV1 75% predicted
MMEF 40% predicted
Her histamine challenge demonstrated a fall in FEV1 of 25%.
Which one of the following is the most likely diagnosis?
A. Asthma.
B. Bronchial stenosis.
C. Bronchomalacia.
D. Bronchopulmonary dysplasia.
E. Reflux aspiration.
128. The mother of a seven-year-old boy on maintenance chemotherapy for acute
lymphoblastic leukaemia telephones the oncology department to say that a child in her son’s
class at school developed chickenpox yesterday. Her son has no history of having had
chickenpox in the past.
Which one of the following actions would you advise for her son?
A. Admission for treatment with intravenous aciclovir.
B. Immediate administration of varicella zoster immune globulin (ZIG).
C. Immediate immunisation with varicella vaccine.
D. Measurement of serum varicella antibody status and administration of ZIG if he is
seronegative.
E. Observe and treat with intravenous aciclovir if chickenpox develops.
129. Infection with which one of the following organisms is best described as having its peak
incidence in the first two years of life, with a secondary peak between age 15 and 24 years?
A. Bordetella pertussis.
B. Haemophilus influenzae type b.
C. Influenza virus.
D. Neisseria meningitidis.
E. Streptococcus pneumoniae.
130. An eight-month-old boy presents to the emergency room with a four-hour history of
profound lethargy. When he woke that morning he was irritable and had episodes of
unexplained crying.
On examination he is a lethargic, pale infant. He has a temperature of 37.5°C. He is not clinically
dehydrated. His cardiovascular and respiratory examination is normal. He has no focal
neurological signs. His abdomen is slightly distended and there is fullness in the right upper
quadrant. Bowel sounds are normal.
The most likely cause for this infant’s condition is:
A. allergic enterocolitis.
B. constipation.
C. intussusception.
D. malrotation.
E. Salmonella dysentery.
131. A mother brings her one-year-old son to see you. For the past three months he has been
waking every night, usually two hours after settling. He falls asleep at the start of the night with
a bottle of milk. When he wakes during the night, he cries, stands up in the cot and drools.
When his mother is unable to calm him with patting she gives him a bottle of 30 mL of formula.
He settles back to sleep after drinking the formula. The mother says he is irritable during the
day and this is ‘getting her down’. Physical examination is normal.
Which one of the following is the most likely diagnosis?
A. Hunger.
B. Night terrors.
C. Separation anxiety disorder.
D. Sleep association disorder.
E. Teething.
132. A foetus is found to have an isolated echogenic focus in the left ventricle on a routine 18-

week ultrasound. Which one of the following is the most likely explanation for this finding?

A. A variant of normal.
B. Down syndrome.
C. Neurofibromatosis.
D. Papillary muscle myxoma.
E. Tuberous sclerosis (rhabdomyoma).
133. A six-year-old boy with chronic granulomatous disease presents with a cough of three
weeks duration. A chest X-ray shows right upper lobe consolidation. A computed tomography
(CT) scan of the chest shows destruction of the right second rib.
Infection with which one of the following is most likely to give this clinical picture?
A. Aspergillus.
B. Atypical mycobacteria.
C. Candida albicans.
D. Nocardia.
E. Staphylococcus aureus.
134. A 10-year-old girl has a history of mucocutaneous candidiasis since age two years and
hypoparathyroidism since age four years. She now has increasing tiredness but is clinically and
biochemically euthyroid.
Which one of the following would be the most useful next test to confirm a diagnosis of adrenal
insufficiency?
A. Adrenal autoantibodies.
B. C26:C22 long chain fatty acid ratio.
C. Plasma adrenocorticotrophic hormone.
D. Plasma cortisol.
E. Plasma renin activity.
135. A seven-year-old boy with a history of frequent episodic asthma presents to his
paediatrician with a six-month history of recurrent headaches with frequent school absences.
The headaches are described as being frontal, “banging”, and associated with pallor. Sleep
typically relieves the headaches. His neurologic examination is normal.
A diagnosis of migraine is made and recommendations are made with regard to the use of
abortive treatment and looking for specific triggers.
On review, two months later his headaches continue unabated and a decision is made to
recommend prophylactic therapy.
Which one of the following prophylactic agents is least appropriate?
A. Amitriptyline.
B. Cyproheptadine.
C. Pizotifen.
D. Propranolol.
E. Sodium valproate.
136. An asymptomatic infant undergoing a routine discharge check on day two is noted to have

a continuous murmur. His arterial oxygen saturation is 88% in room air.

Which one of the following is the most likely diagnosis?


A. Coronary artery fistula.
B. Persistent ductus arteriosus.
C. Pulmonary atresia and ventricular septal defect.
D. Ruptured sinus of Valsalva.
E. Transposition of the great arteries.
137. A four-month-old boy is referred for investigation of severe failure to thrive. His weight
has progressively fallen below the 3rd percentile (weight 3.2 kg). He drinks approximately 600
mL of cows’ milk-based formula per day. He has only mild regurgitation after feeds and no
diarrhoea.
On examination he appears generally well in himself and very alert. He has lost most of his
subcutaneous tissue. His abdomen is non-tender without palpable masses or organomegaly.
The most likely diagnosis is:
A. cows’ milk intolerance.
B. cystic fibrosis.
C. diencephalic syndrome.
D. hyperthyroidism.
E. neuroblastoma.
138. A five-year-old girl presents with constant wetting (day and night). She voids normally but

her underpants are always damp. Physical examination is normal. Urine culture is clear.

Which one of the following is the most likely diagnosis?

A. Depression.

B. Detrusor instability.

C. Diabetes insipidus.

D. Ectopic ureter.

E. Maturational delay.
139. A 10-year-old obese boy presents with high fevers, shortness of breath, and he appears

quite unwell. A chest X-ray reveals bilateral pleural effusions and a large heart. Ultrasound

confirms a pericardial effusion. There is no pulsus paradoxus. He is started on intravenous

cefotaxime. He becomes more short of breath and remains oxygen dependent after 24 hours,

with his chest X-ray showing increased pleural effusions. There are no changes in blood

pressure.

The priority in his management is to:


A. add intravenous flucloxacillin.

B. insert large pleural chest drains.


C. observe in intensive care unit.

D. perform a thoracotomy to create a pericardial window.

E. perform pleural and pericardial paracentesis.


140. The parents of a 15-year-old boy telephone about his changing behaviour over the
previous two weeks. His teachers have described poor concentration and disruptive behaviour
at school and he has been truant for most of the past week. He has been very difficult to live
with, arguing with his older brother, watching television all night and going out during the day.
The parents of his girlfriend are also concerned as they believe the boy and their daughter have
started having unprotected sex and he has encouraged her to leave school. The boy refuses to
see you.
Which one of the following diagnoses best explains these symptoms?
A. Attention deficit/hyperactivity disorder.
B. Bipolar affective disorder.
C. Conduct disorder.
D. Marijuana abuse.
E. Schizophreniform disorder.
141. Which one of the following is the least reliable predictor of neurodevelopmental outcome
in term infants with hypoxic-ischaemic encephalopathy?
A. Age at onset of seizures of less than 12 hours.
B. An abnormal electroencephalogram (EEG) at seven days of age.
C. Anuria or oliguria for more than 24 hours.
D. Apgar score at 10 minutes.
E. Neurological examination at 10 days of age.
142. A 13-month-old boy presented with a three-week history of daily fevers and persistent
mouth ulcers, and two days of vomiting and diarrhoea. On examination he was miserable,
febrile to 38.9ºC and drooling. The gingiva were red and friable with multiple shallow grey-
coloured ulcers which did not involve the lips, palate or pharynx. He had multiple small cervical
lymph nodes.
Initial investigations revealed:
haemoglobin 89 g/L [95-140]
9
white cell count 11.3 x 10 /L [5.0-17.0]
9
neutrophils 0.2 x 10 /L [1.0-8.0]
9
lymphocytes 9 x 10 /L [2-13]
9
platelet count 614 x 10 /L [150-500]
erythrocyte sedimentation rate (ESR) 7 mm/hr [2-20]
urine culture negative
blood culture negative
The ulcers were swabbed for viral culture, but no virus was isolated.
The fever, vomiting and diarrhoea settled within three days, the mouth ulcers had healed after
six days and the neutrophil count returned to normal after 10 days.
A diagnosis of probable herpes simplex virus infection associated with viral-induced
neutropenia was made.
He re-presents two weeks later with a fever and erythema and swelling of the right eye,
secondary to preseptal cellulitis.
Serum immunoglobulin levels are:
IgG <0.33 g/L [3.55-12.10]
IgA <0.07 g/L [0.15-1.24]
IgM 0.92 g/L [0.38-1.66]
Antibodies to tetanus, diphtheria and Haemophilus influenzae type b are not detected despite
his immunisations being up-to-date. T and B cell numbers are normal and T cell proliferation
in response to concanavalin A is normal.
This pattern of immunodeficiency fits best with:
A. chronic granulomatous disease.
B. hyper-IgM syndrome.
C. severe-combined immunodeficiency.
D. transient hypogammaglobulinaemia.
E. X-linked hypogammaglobulinaemia.
143. Stevens-Johnson syndrome is most likely to occur with which one of the following
anticonvulsant drugs?
A. Carbamazepine.
B. Lamotrigine.
C. Phenytoin.
D. Sodium valproate.
E. Vigabatrin.
144. An 18-month-old girl develops fever and diarrhoea. Two days later she begins to vomit,
develops a faint, generalised maculopapular rash, increasing tachypnoea and becomes
obtunded. She is febrile 39.2°C, pale and poorly perfused. Her capillary refill time is 4 seconds.
Her pulse is 140/minute and thready. Blood pressure is 90/60 mmHg. Her apex beat is difficult
to feel but is in the mid-axillary line. She has a pansystolic murmur radiating to the axilla. Her
respiratory rate is 52/minute and chest clear. Her liver is palpable 5 cm below the costal
margin. She is drowsy and has moderate neck stiffness. Her generalised rash is truncal and is
fine, maculopapular and blanches on pressure.
Investigations show:
haemoglobin 108 g/L [110-140]
9
white cell count 11.4 x 10 /L [4.0-11.0]
9
neutrophils 5.2 x 10 /L [1.0-4.0]
9
platelet count 201 x 10 /L [150-400]
liver function tests normal
cerebrospinal fluid (CSF)
6
white cell count 420 x 10 /L (300 polymorphs, 120 monocytes)
6
red cell count 20 x 10 /L
sugar 2.2 mmol/L (blood sugar 4.2 mmol/L)
protein 0.5 mmol/L [0.2-0.6]
CSF Gram stain no organisms seen
chest X-ray cardiomegaly with pulmonary congestion
Which one of the following is the most likely causative organism?
A. Coxsackie B virus.
B. Enterovirus 71.
C. Herpes simplex virus.
D. Mycoplasma pneumoniae.
E. Neisseria meningitidis.
145. Abuse of which one of the following recreational drugs during pregnancy presents the
greatest risk of birth defects to the foetus?
A. Amphetamines.
B. Cocaine.
C. Heroin.
D. THC (tetrahydrocannibol).
E. Tobacco.
146. Following an emergency caesarean section, a term male infant, birth weight 2800 g,
required extensive resuscitation with endotracheal intubation, ventilation and cardiac
massage. His Apgar scores were recorded as 0 at 1 minute, 3 at 5 minutes and 5 at 10 minutes.
At 15 minutes of age he is noted to be bleeding from intravenous puncture sites and his
umbilicus, and to develop widespread ecchymoses.
Investigation of the cause of his bleeding reveals:
prothrombin time (PT) 65 seconds [10-24]
activated partial thromboplastin time (APTT) 150 seconds [31-55]
fibrin degradation products 15 μg/mL [<10]
9
platelet count 60 x 10 /L [150-400]
Which one of the following is the most appropriate treatment of the bleeding diathesis?
A. Exchange transfusion.
B. Intravenous cryoprecipitate.
C. Intravenous fresh frozen plasma.
D. Intravenous heparin.
E. Intravenous platelets.
147. A 12-year-old girl presents to the emergency department with swollen eyes and ankles
for one week. She has been lethargic and complained of back and right flank pain for
approximately 10 days. Examination reveals a well grown adolescent with blood pressure
120/84 mmHg. She has pitting oedema up to her thighs and her liver is palpable 2 cm below
the right costal margin and is tender. Urine microscopy shows 400 red cells, red and white cell
casts and 4+ protein.
Blood tests show:
haemoglobin 85 g/L [115-150]
9
white cell count 4.62 x 10 /L [4.40-12.50]
9
segmented neutrophils 3.33 x 10 /L [1.50-7.40]
9
lymphocytes 1.02 x 10 /L [1.50-4.70]
9
platelet count 110 x 10 /L [150-400]
plasma creatinine 0.10 mmol/L [0.04-0.08]
Which one of the following is the most likely diagnosis?
A. Focal segmental glomerulosclerosis.
B. IgA nephropathy.
C. Lupus nephritis.
D. Membranoproliferative glomerulonephritis.
E. Post-streptococcal glomerulonephritis.
148. A four-month-old infant known to have tetralogy of Fallot presents to the emergency

department with cyanotic episodes associated with crying.

Which one of the following is the most useful indicator of whether these episodes are
representative of hypercyanotic (tetralogy) spells?
A. Baseline arterial saturation.
B. Echocardiographic gradient across the right ventricular outflow tract at rest.
C. Echocardiographic gradient across the right ventricular outflow tract during a cyanotic
episode.
D. Intensity of the murmur during a cyanotic episode.
E. Oxygen responsiveness during a cyanotic episode.
149. A two-month-old girl, born at 30 weeks gestation, is referred by her family doctor as she
had been noticed to stop breathing during sleep over the previous three nights. Two days
previously she had developed an upper respiratory tract infection. She also has a history of
vomiting which is being treated with ranitidine. Oximetry shows oxygen desaturation to the
high 80’s.
Which one of the following is the most likely diagnosis?
A. Aspiration.
B. Chlamydial pneumonia.
C. Obstructive sleep apnoea.
D. Respiratory syncytial virus infection.
E. Seizure disorder.
150. Which one of the following conditions is not associated with Wilms tumour?
A. Aniridia.
B. Beckwith-Wiedemann syndrome.
C. Cryptorchidism.
D. Hemihypertrophy.
E. Fanconi anaemia.
151. Which one of the following is least likely to be associated with Munchausen by proxy
syndrome?
A. Father frequently absent.
B. Maternal intravenous drug abuse.
C. Older sibling died from sudden infant death syndrome aged six months.
D. Recurrent polymicrobial bacteraemia.
E. Recurrent unwitnessed seizures.
151. A one-year-old child arrives in the emergency department intubated and ventilated with
high flow oxygen, following a motor vehicle accident. Observation reveals a child who is
unconscious, cyanosed, pale and with limited chest expansion. Neck veins are slightly
distended.
Further examination demonstrates poor peripheral perfusion, a weak pulse and tachycardia at
160/minute. The child’s trachea is deviated to the right. Air entry is decreased bilaterally but
with air entry on the left reduced compared with the right-hand side. There is hyperresonance
over the left chest.
The most appropriate next step is:
A. chest drain insertion to the left chest.
B. needle thoracocentesis of the left chest.
C. repositioning of the endotracheal tube.
D. suction of the endotracheal tube.
E. urgent chest X-ray.
152. Which one of the following sleep behaviours has the highest prevalence in primary school
aged children?
A. Bruxism.
B. Enuresis.
C. Night terrors.
D. Sleep talking.
E. Sleep walking.
153. A 12-year-old male bone marrow recipient develops acute upper abdominal pain on day

10 after transplantation from a related donor. Over the following three days he develops

increasing abdominal distension and jaundice. He has developed tender hepatomegaly and

gained 4 kg in weight. His bowel sounds are sparse. His liver function tests reveal:

alanine aminotransferase (ALT) 1850 U/L [5-45]


gamma glutamyltransferase (GGT) 683 U/L [5-24]
conjugated bilirubin 125.0 μmol/L [0-3.4]
unconjugated bilirubin 12 μmol/L [0-17]
albumin 30 g/L [40-53]
total protein 55 g/L [66-82]
His plain abdominal film shows marked ascites.
The most likely diagnosis is
A. acute cholecystitis.
B. graft-versus-host disease of the liver.
C. hepatic veno-occlusive disease.
D. primary peritonitis.
E. transfusion-related acute hepatitis.
154. A 14-year-old girl with cystic fibrosis is reviewed in clinic. It is two months since her last
review and in that time she has lost 2 kg in weight, and has had a 10% reduction in forced
expiratory volume in one second (FEV1) but has had no recent wet cough. She has had normal
stools. On examination her chest is clear.
Which one of the following is the most likely diagnosis?
A. Diabetes mellitus.
B. Gastro-oesophageal reflux.
C. Inadequate pancreatic supplementation.
D. Inadequate salt intake.
E. Recurrent active bronchitis.
155. A four-year-old girl is diagnosed with Duchenne muscular dystrophy (DMD). She is of
normal intelligence and is not dysmorphic. Her height is on the 3rd percentile. There is no
family history of any similar disorder, but her mother has a high serum creatine kinase (CK)
level.
Which one of the following is the most likely mechanism for the expression of this X-linked
recessive disorder in this girl?
A. Father is mildly affected and mother is a carrier of the X-linked mutation.
B. Mother is a carrier and a new mutation has occurred on the paternally derived allele.
C. She has inherited an apparently balanced X/autosome translocation from her mother.
D. She has inherited two copies of the X-linked mutation due to uniparental disomy.
E. She has Turner syndrome as well as having inherited an X-linked mutation.
156. A term infant was born by breech delivery with forceps assisted delivery of the head.
Foetal heart rate decelerations (type 1) were noted towards the end of the second stage. The
Apgar scores were 5 at 1 minute and 8 at 5 minutes. Over the next 18 hours the infant was
drowsy but able to rouse for feeds and was afebrile. At 18 hours of age the infant develops
apnoea and abnormal movements consistent with seizures.
Which one of the following is the most likely diagnosis?
A. Bacterial meningitis.
B. Congenital malformation of the brain.
C. Hypoxic-ischaemic encephalopathy.
D. Intraventricular haemorrhage.
E. Sub-arachnoid haemorrhage.
157. A mother presents to you with her first child, aged two months. He has been crying most
days for the last three weeks, up to two hours each day. He vomits after every second feed. He
has 4-6 loose yellow bowel motions each day. He is currently on S26 formula and is gaining
weight. Physical examination is normal.
Which one of the following is the most appropriate next step?
A. Change feeds to soy-based formula.
B. Discuss normal crying patterns.
C. Take stool sample for reducing substances.
D. Trial of ranitidine.
E. Urine culture.
158. Anorexia nervosa has a highly variable outcome. Which one of the following factors is
least predictive of a poor prognosis?
A. Degree of weight loss at initial presentation.
B. Frequent hospitalisations.
C. Highly disturbed relationships with family members.
D. Prepubertal onset.
E. Poor premorbid functioning.
159.
A 14-year-old Pre-lunch Pre-dinner Pre-bedtime
Lebanese boy with
type 1 (insulin-
dependent)
diabetes mellitus
of two years
duration has a
deterioration in
his metabolic
control. His
current insulin
dose is 0.8
U/kg/day. He
measures his
blood glucose at
home using a
glucometer. His
blood glucose
record book over
the past week
(typical of the past
three months’
readings) is shown
below (all values
are in mmol/L).
Pre-breakfast
Monday 4.5 6.3 5.7 8.0
Tuesday 5.6 10.3 6.2 7.0
Wednesday 6.3 5.6 10.3 9.6
Thursday 7.4 8.4 6.9 7.6
Friday 5.8 5.9 8.5 6.8
Saturday 7.1 12.6 5.9 11.5
Sunday 10.5 8.1 6.8 9.4
160. A male infant is born at term following an uncomplicated pregnancy. He has Apgar scores
of 8 at 1 and 5 minutes. Severe hypotonia is present from birth. Feeding is difficult to establish
and tube feeding is required. However, no respiratory support is needed. On examination at
six weeks of age, he remains very hypotonic and is still being tube fed. His reflexes are normal
and despite his hypotonia, he is observed to move all limbs and has a normal cry.
The investigation which is most likely to make a diagnosis in this infant is:
A. acetylcholine receptor studies.
B. deletion studies for spinal muscular atrophy.
C. dystrophin deletion studies.
D. fluorescent in-situ hybridisation (FISH) for deletion on chromosome 15p.
E. mutation testing for myotonic dystrophy.
161. A five-year-old boy has been rescued from a house fire. He is not cyanosed but has mild

inspiratory stridor, a cough, burns to the face and carbon deposits around the mouth and nose.

Cervical spine injury is excluded. An intravenous cannula has been inserted and he is being

given oxygen by face mask.

The most urgent next step in his management is to:

A. administer adequate analgesia.


B. administer nebulised adrenaline.
C. administer nebulised steroids.
D. administer volume replacement rapidly.
E. prepare for endotracheal intubation
162. A six-day-old breastfed female infant is taken to the emergency department by her
mother because of bright red rectal bleeding, mild diarrhoea with mucus and persistent crying.
On examination the baby is generally well and haemodynamically stable. The perianal region
appears normal.
Which one of the following is the most likely diagnosis?
A. Allergic colitis.
B. Colonic duplication.
C. Juvenile polyp.
D. Meckel diverticulum.
E. Swallowed blood.
163. Which one of the following is the most common among adolescent (13-17 years) girls in
the community?
A. Anorexia nervosa.
B. Anxiety disorders.
C. Attention deficit/hyperactivity disorder.
D. Conduct disorder.
E. Depressive disorders.
164. A 16-year-old boy presents with a history of three episodes of low back pain over six
months, each lasting for 10 days. The pain is worse later in the day, eased by rest, and is not
associated with morning stiffness in the back.
Investigations show:
full blood examination within normal limits
erythrocyte sedimentation rate (ESR) 19 mm/hr [2-20]
HLA-B27 positive
X-ray of lumbosacral spine and sacro-iliac joints normal
The most likely diagnosis is:
A. ankylosing spondylitis.
B. discitis.
C. non-specific low back pain.
D. Scheuermann’s osteochondritis.
E. spondylolysis.
165. Which one of the following drug associations is least likely to occur?
A. Chloramphenicol and grey baby syndrome.
B. Fusidic acid and jaundice.
C. Rifabutin and red/orange skin discolouration.
D. Rifampicin and orange tears.
E. Teicoplanin and red man syndrome.
166. A 12-month-old boy presents for assessment of poor weight gain. His birth weight was

3800 g. He was breastfed for four months then weaned to formula and commenced on solids.

His weight has decreased from the 50th percentile at age three months to the 10th percentile

at age 12 months. His length remains on the 50th percentile. His mother reports that he is a

fussy eater.

Past history includes mild bronchiolitis at eight months of age, upper respiratory tract
infections at 10 and 11 months of age, and intermittent purulent nasal discharge. He has not
had diarrhoea or vomiting.
On examination, the child is thin. He has bilateral purulent nasal discharge and bilateral otitis
media with effusion. Physical and developmental examinations are otherwise normal.
Which one of the following is the most likely cause of his failure to thrive?
A. Coeliac disease.
B. Cows’ milk protein intolerance.
C. Cystic fibrosis.
D. Immune deficiency.
E. Inadequate nutrition.
167. A three and a half-year-old boy presents to the emergency department with a decreased

level of consciousness. He has had vomiting and diarrhoea for 24 hours. His past history is

unremarkable and he has had normal development. His height and weight are on the 10th

percentile and his physical examination is normal. His blood glucose is 1.9 mmol/L [>3.5] and

urine is positive for ketones.

Which one of the following is the most likely diagnosis?


A. Growth hormone deficiency.
B. Hyperinsulinism.
C. Ketotic hypoglycaemia.
D. Medium-chain acyl-CoA dehydrogenase deficiency.
E. Type IV glycogen storage disease.
168. A term baby is noted to be going blue during feeds on the second day of life. He seems to
behave normally between feeds except for frequent small vomits. No abnormality is detected
on physical examination. His chest X-ray shows patchy changes in the right upper lobe
consistent with consolidation.
Which one of the following is the most likely diagnosis?
A. Choanal atresia.
B. Congenital heart disease.
C. Congenital laryngeal cleft.
D. Gastro-oesophageal reflux.
E. Tracheo-oesophageal fistula.
169. For which one of the following conditions is there the best evidence of benefit from
systemic corticosteriods?
A. Acute disseminated encephalomyelitis.
B. Cerebral malaria.
C. Guillain-Barré syndrome.
D. Meningococcal meningitis.
E. Tuberculous meningitis.
170. A four-month-old infant presents with biphasic stridor since day one of life and an unusual
cry, which has been described as ‘quiet’. The infant sucks and swallows normally. Growth and
development are also normal.
The most likely cause of the stridor is:
A. infantile larynx (laryngomalacia).
B. lingual cyst.
C. subglottic haemangioma.
D. vascular ring.
E. vocal cord lesion.
171. A three-year-old boy presents with a two-week history of profuse, watery diarrhoea. He
has a history of otitis media for which he was treated with amoxycillin. Laboratory tests reveal
the following findings:
serum sodium 138 mmol/L [135-145]
serum potassium 4.3 mmol/L [3.5-5.5]
faecal sodium 98 mmol/L
faecal potassium 32 mmol/L
stool reducing sugars 0.25%
stool pH 6.5
These findings are most consistent with:
A. coeliac disease.
B. cryptosporidiosis.
C. lactose intolerance.
D. laxative abuse.
E. sucrase-isomaltase deficiency.
172. A 27-month-old boy is referred for assessment. His parents have observed approximately
10 episodes in the past three months where he has seemed distressed and unstable on his
legs. The episodes have been brief (15-20 seconds) and recent episodes have seen him
suddenly stop, close his eyes and grasp at a nearby wall or door for the duration. He may
become pale during an episode, but no other signs have been observed by his parents and he
quickly resumes his activities.
Detailed neurological examination is normal although fundoscopy is not possible. Family
history includes a maternal uncle with migraine but no family history of epilepsy. His past
medical history reveals a normal pregnancy and delivery. He had a three-minute febrile
convulsion at 15 months of age in association with otitis media. Neurodevelopmental progress
is normal.
Which one of the following is the most likely cause/diagnosis?
A. Benign paroxysmal vertigo.
B. Breath-holding episodes.
C. Cardiac arrhythmia.
D. Epilepsy.
E. Migraine.
173. Which one of the following vaccines is contraindicated in a child with anaphylactic egg
allergy?
A. Bacille Calmette-Guérin (BCG).
B. Influenza.
C. Measles-mumps-rubella (MMR).
D. Oral polio.
E. Whole cell pertussis.
174. A four-year-old child, on no medications, presents with fever and is generally unwell.
Meningococcal sepsis is least likely if there is:
A. a macular, blanching rash.
B. a maculopapular rash.
C. a mixed maculopapular and petechial rash.
D. an urticarial rash.
E. no rash.
175. A 13-year-old boy with a history of cystic fibrosis and complex partial seizures is currently
on carbamazepine. He develops a bout of sinusitis and is prescribed an antibiotic. Two days
later he presents to the Emergency Department with a depressed conscious state and
recurrent seizures. He is found to have markedly elevated carbamazepine levels.
Which one of the following antibiotics is he most likely to have been prescribed?
A. Amoxycillin-clavulanic acid.
B. Cefaclor.
C. Ciprofloxacin.
D. Doxycycline.
E. Erythromycin.
176. Successful active management of a child with nocturnal enuresis is most dependent on
the child’s:
A. age.
B. bladder size.
C. family history.
D. fluid intake.
E. motivation.
177. A seven-month-old girl presents with a four-week history of chronic diarrhoea and failure
to thrive. She was entirely bottle-fed with cow’s milk formula until five months of age. A range
of solids, including tinned fruit and vegetables, was then introduced into her diet. There is no
history of vomiting or fevers. The diarrhoea is watery and has caused significant perianal
excoriation.
Faecal biochemical tests are likely to show which one of the following?
pH Reducing Total
sugars sugars
A. 7.2 0.25% 2%
B. 7.2 2% 2%
C. 7.2 0.25% 0.25%
D. 7.6 0.25% 0.25%
E. 7.6 0.25% 2%
178. A 10-year-old boy presents with a 13-month history of recurrent eye-blinking and rapid
movements of his eyes, with recurrent throat clearing. The events tend to wax and wane but
are most prominent at school or when he is under stress. His paediatrician sees him and a
diagnosis of Tourette syndrome is made.
Which one of the following is most likely to be identified on family history?
A. Epilepsy.
B. Essential tremor.
C. Huntington disease.
D. Obsessive-compulsive disorder.
E. Wilson disease.
179. In the presence of meconium-stained liquor, meconium aspiration syndrome is best
prevented by:
A. inspection of the vocal cords, followed by intubation and suction if meconium is visualised.
B. intubation and endotracheal suctioning before the first breath.
C. intubation and endotracheal suctioning in the presence of thick meconium-stained liquor.
D. intubation, endotracheal suction and the administration of artificial surfactant.
E. thorough suctioning of the oropharynx before the first breath.
180. A child presents to the Emergency Department in diabetic ketoacidosis.
Following institution of therapy, the most likely cause of death is:
A. acidosis.
B. acute renal failure.
C. cerebral oedema.
D. hypoglycaemia.
E. hypokalaemia.
181. A four-year-old girl presents with a history of normal development up to nine months of
age, followed by a progressive loss of acquired skills and the onset of repetitive wringing of her
hands.
In order to make a clinical diagnosis of Rett syndrome the child should also have:
A. characteristic speech, with clear words but poor cognitive content.
B. characteristic unilateral skin pigmentation.
C. congenital microcephaly.
D. poor head growth starting after six months.
E. typical retinal changes.
182. Which one of the following immunomodulatory therapies is least effective in the
management of atopic dermatitis?
A. Allergen immunotherapy.
B. Azathioprine.
C. Cyclosporin.
D. Interferon gamma.
E. Psoralen and ultraviolet light photochemotherapy.
183. A 10-day-old boy, born at term, is brought to the Emergency Department with a three-
day history of lethargy, poor feeding and vomiting. He is breastfed, but commenced
supplemental formula feeds the day prior to the presentation. His birth weight was 3005 g,
head circumference was 34 cm and length was 49 cm.
On examination he is afebrile and mildly dehydrated. His weight is 2650 g. He has episodes of
hypertonicity with opisthotonic posturing and cycling movements of his limbs. Abdominal,
cardiovascular and respiratory examinations are normal.
Investigations show:
haemoglobin 156 g/L [135-205]
white cell count 9.5 x 109/L [5.0-15.0]
platelet count 299 x 109/L [250-450]
sodium 145 mmol/L [135-145]
chloride 105 mmol/L [95-110]
potassium 3.5 mmol/L [3.5-5.5]
bicarbonate 19 mmol/L [22-26]
urea 6.5 mmol/L [1.3-6.6]
creatinine 0.045 mmol/L [0.020-0.050]
lactate 2.2 mmol/L [0.7-2.0]
ammonia 75 μmol/L [<60]
calcium 2.20 mmol/L [2.10-2.75]
glucose 1.9 mmol/L [3.0-5.5]
pH 7.39 [7.35-7.45]
PCO2 35 mmHg
PO2 100 mmHg
cerebrospinal fluid (CSF) glucose 3.4 mmol/L [>3.0]
CSF protein 0.9 g/L [<1.2]
urinalysis ++++ ketones
Cerebral ultrasound demonstrated small ventricles suggestive of cerebral oedema.
Which one of the following is the most likely diagnosis?
A. Isovaleric acidaemia.
B. Maple syrup urine disease.
C. Medium-chain acyl-CoA dehydrogenase deficiency.
D. Ornithine transcarbamylase deficiency.
E. Phenylketonuria.
184. An eight-year-old girl presents with a six-month history of central chest pain. She
describes the pain as squeezing in nature. The pain occurs most days and can occur both at
rest and with exercise. She is a competitive swimmer and has not noted the pain whilst racing.
As an infant she was diagnosed on clinical grounds with a small muscular ventricular septal
defect but is now clinically normal.
Echocardiography is most likely to show which one of the following?
A. Aberrant origin of the left anterior descending coronary artery from the right coronary
artery.
B. Anomalous left coronary artery.
C. L-transposition of the great arteries.
D. Normal heart.
E. Single coronary artery with a branch crossing between the great arteries.
185. An 18-month-old boy is referred by his general practitioner to a paediatrician for advice
about febrile seizures. The child has had three episodes of febrile convulsions, the first at 13
months of age. On each occasion, two seizures have occurred in a 24-hour period. Each seizure
was brief (less than five minutes) in the setting of a high fever (greater than 39°C). The child is
developing normally and his neurological examination is unremarkable. The child’s father has
a history of febrile seizures.
The risk of epilepsy in this child is approximately:
A. 0.5%.
B. 2%.
C. 5%.
D. 10%.
E. 15%.
186. A 15-year-old girl with known common variable immunodeficiency, who is receiving
regular monthly infusions of intravenous immunoglobulin, presents with a one-month history
of weight loss, loose motions without blood and intermittent epigastric discomfort which has
no clear relationship to meals.
In this clinical setting, the most likely diagnosis is:
A. coeliac disease.
B. cryptosporidiosis.
C. giardiasis.
D. intestinal lymphoma.
E. peptic ulcer disease.
187. A 15-month-old girl has not yet had measles-mumps-rubella (MMR) vaccine, but is up-to-
date with her other vaccinations. The mother and local doctor are concerned about the safety
of giving MMR vaccine because of possible egg allergy. On several occasions after eating foods
containing egg the child has developed generalised urticaria without systemic symptoms.
There is a family history of atopy and febrile convulsions in siblings.
Which one of the following is the most appropriate advice?
A. Delay MMR vaccination until the child is two years of age.
B. Vaccinate with MMR.
C. Vaccinate with MMR under cover of dexamethasone and an antihistamine.
D. Withhold MMR vaccination if the child is skin-prick positive to egg.
E. Withhold MMR vaccination indefinitely as the risk of anaphylaxis is too high.
188. A pregnant woman develops an infectious mononucleosis-like illness at 12 weeks
gestation, with fever, sore throat, malaise and lymphadenopathy. She is proven serologically
to have acute cytomegalovirus (CMV) infection.
What is the most likely outcome for the baby?
A. Hepatitis.
B. Microcephaly.
C. Normal baby.
D. Pneumonitis.
E. Sensorineural deafness.
189. An adolescent presents after an episode of deliberate self-harm.
Which one of the following factors is least predictive of the adolescent going on to complete a
suicide?
A. Clear suicidal intent.
B. Gender.
C. History of high-risk impulsive behaviours.
D. Lethality of method.
E. Socio-economic status.
190. In an asymptomatic person with human immunodeficiency virus (HIV) infection, which
one of the following is the best predictor of the future rate of decline of immune function?
A. CD4+ lymphocyte count.
B. CD8+ lymphocyte count.
C. p24 antigenaemia.
D. Plasma HIV RNA concentration.
E. Serum β2 microglobulin concentration.
191. An 18-month-old child presents with a confirmed urinary tract infection.
The risk of long-term renal hypertension is best assessed by abnormalities on:
A. DMSA (dimercapto succinic acid) scan.
B. DTPA (diethylenetriamine penta-acetic acid) scan.
C. intravenous pyelogram.
D. micturating cysto-urethrogram (MCU).
E. renal ultrasound.
192. A four-month-old infant presents with biphasic stridor since day one of life and an unusual
cry, which has been described as ‘quiet’. The infant sucks and swallows normally. Growth and
development are also normal.
The most likely cause of the stridor is:
A. infantile larynx (laryngomalacia).
B. lingual cyst.
C. subglottic haemangioma.
D. vascular ring.
E. vocal cord lesion.
193. Which one of the following conditions is not associated with Wilms tumour?
A. Aniridia.
B. Beckwith-Wiedemann syndrome.
C. Cryptorchidism.
D. Hemihypertrophy.
E. Fanconi anaemia.
194. An 18-month-old boy has recurrent blue breath-holding episodes, followed on one
occasion by a 15-second generalised seizure.
The investigation most likely to be useful in directing therapy is:
A. electrocardiogram (ECG).
B. electroencephalogram (EEG).
C. iron studies.
D. plasma calcium.
E. plasma glucose.
195. A child weighing 24 kg with a known, small, perimembranous ventricular septal defect is
to undergo routine isolated dental ‘cleaning and scaling’.
Assuming the child is not allergic to penicillin, which one of the following is recommended in
relation to bacterial endocarditis prophylaxis?
A. Amoxycillin 1.2 g, per oral, administered one hour before the procedure only.
B. Amoxycillin 600 mg, intravenously, administered one hour before the procedure and
repeated six hours after the procedure.
C. Amoxycillin 600 mg, q8h, per oral, for 48 hours prior to the procedure.
D. Antibiotic prophylaxis is not recommended for this procedure.
E. Phenoxymethylpenicillin 1.2 g, per oral, administered one hour before the procedure.
196. Which feature distinguishes hyper-IgE syndrome from severe atopic dermatitis?
A. Decreased number of peripheral blood CD8+ lymphocytes.
B. Distribution of the eczematous rash.
C. Extremely high serum IgE levels.
D. Negative delayed skin tests to Candida.
E. Staphylococcal abscesses in axillary lymph nodes.
197. Stevens-Johnson syndrome is most likely to occur with which one of the following
anticonvulsant drugs?
A. Carbamazepine.
B. Lamotrigine.
C. Phenytoin.
D. Sodium valproate.
E. Vigabatrin.
197. A 15-month-old boy is referred for persistent collapse of the right middle lobe, which has
been present for eight weeks following confirmed adenoviral pneumonia. There is no
improvement after six weeks of bronchodilators, oral steroids and physiotherapy. Fine crackles
are heard throughout the right lung field. Investigations reveal a mild iron deficiency anaemia,
normal white cell count, normal immunological profile, normal sweat chloride, negative
Mantoux test and a bronchoscopy does not reveal any foreign body or obvious mucosal lesions.
Which one of the following is the most likely diagnosis?
A. Asthma.
B. Chronic bronchiolitis of infancy.
C. Necrotising pneumonia.
D. Obliterative bronchiolitis.
E. Organising pneumonia.
198. Persistent reduction of serum complement component 3 (C3) is most strongly associated
with which one of the following forms of glomerulonephritis?
A. Diffuse proliferative systemic lupus erythematosus (SLE).
B. Idiopathic membranous.
C. Mesangial IgA disease.
D. Mesangiocapillary.
E. Post-streptococcal.
199. A 13-month-old boy presented with a three-week history of daily fevers and persistent
mouth ulcers, and two days of vomiting and diarrhoea. On examination he was miserable,
febrile to 38.9ºC and drooling. The gingiva were red and friable with multiple shallow grey-
coloured ulcers which did not involve the lips, palate or pharynx. He had multiple small cervical
lymph nodes.
Initial investigations revealed:
haemoglobin 89 g/L [95-140]
white cell count 11.3 x 109/L [5.0-17.0]
neutrophils 0.2 x 109/L [1.0-8.0]
lymphocytes 9 x 109/L [2-13]
platelet count 614 x 109/L [150-500]
ESR 7 mm/hr [2-20]
urine culture negative
blood culture negative
The ulcers were swabbed for viral culture, but no virus was isolated.
The fever, vomiting and diarrhoea settled within three days, the mouth ulcers had healed after
six days and the neutrophil count returned to normal after 10 days.
A diagnosis of probable herpes simplex virus infection associated with viral-induced
neutropenia was made.
He re-presents two weeks later with a fever and erythema and swelling of the right eye,
secondary to preseptal cellulitis.
Serum immunoglobulin levels are:
IgG <0.33 g/L [3.55-12.10]
IgA <0.07 g/L [0.15-1.24]
IgM 0.92 g/L [0.38-1.66]
Antibodies to tetanus, diphtheria and Haemophilus influenzae type b are not detected despite
his immunisations being up-to-date. T and B cell numbers are normal and T cell proliferation
in response to concanavalin A is normal.
This pattern of immunodeficiency fits best with:
A. chronic granulomatous disease.
B. hyper-IgM syndrome.
C. severe-combined immunodeficiency.
D. transient hypogammaglobulinaemia.
E. X-linked hypogammaglobulinaemia.
200. A 15-year-old girl lives in a rural town where you consult once per month. She presents
with a 12-month history of anxiety-based symptoms, which have prevented her from attending
school.
She describes a sense of dread if she is away from home, associated with palpitations, sweating
and a heavy feeling in her chest. She finds it difficult to fall asleep and has numerous
nightmares. She worries about germs and frequently washes her hands. She also describes
magical thoughts in that she dreads something bad will happen if she tapes over her old videos.
She believes that these thoughts are silly.
Prior to the onset of her symptoms, she functioned well at school and had many friends. She
was not aware of any pressures at the time.
She lives with her father and a 20-year-old sister, her mother having died from a cerebral
haemorrhage eight years ago. There is no family history of psychiatric illness. She refuses to
see a psychiatrist but is willing to continue to see you.
In addition to further counselling, which one of the following medications is most appropriate?
A. Amitriptyline.
B. Clomipramine.
C. Oxazepam.
D. Paroxetine.
E. Thioridazine.
Key test 11

1¶ B
2¶ A
3¶ E
4¶ C
5¶ D
6¶ C
7¶ D
8¶ D
9¶ C
10¶D
11¶C
12¶A
13¶C
14¶B
15¶B
16¶C
17¶C
18¶B
19¶A
20¶A
21¶D
22¶D
23¶D
24¶A
25¶A
26¶E
27¶B
28¶C
29¶C
30¶E
31¶A
32¶C
33¶A
34¶A
35¶C
36¶D
37¶D
38¶E
39¶B
40¶B
41¶D
42¶B
43¶E
44¶C
45¶B
46¶A
47¶D
48¶C
49¶B
50¶D
51¶B
52¶D
53¶B
54¶D
55¶E
56¶C
57¶C
58¶E
59¶B
60¶C
61¶D
62¶A
63¶A
64¶C
65¶C
66¶D
67¶A
68¶E
69¶E
70¶D
71¶D
72¶A
73¶C
74¶A repeat 74 D
75¶E
76¶E
77¶C
78¶D
79¶C
80¶C
81¶E
82¶B
83¶A
84¶D
85¶E
86¶D
87¶B
88¶C
89¶A
90¶E
91¶B
92¶C
93¶A
94¶B
95¶B
96¶C
97¶E
98¶E
99¶D
100¶ B
101¶ C
102¶ D
103¶ C
104¶ B
105¶ D
106¶ d
107¶ c
108¶ c
109¶ e
110¶ e
111¶ b
112¶ a
113¶ a
114¶ a
115¶ c
116¶ b
117¶ c
118¶ c
119¶ d
120¶ c
121¶ c
122¶ c
123¶ b
124¶ a
125¶ c
126¶ b
127¶ d
128¶ b
129¶ d
130¶ c
131¶ d
132¶ a
133¶ a
134¶ c
135¶ d
136¶ c
137¶ c
138¶ d
139¶ b
140¶ b
141¶ d
142¶ b
143¶ b
144¶ a
145¶ b
146¶ c
147¶ c
148¶ d
149¶ d
150¶ e
151¶ b then 151 b
152¶ d
153¶ c
154¶ a
155¶ e
156¶ e
157¶ b
158¶ d
159¶ ---
160¶ d
161¶ e
162¶ a
163¶ b
164¶ c
165¶ e
166¶ e
167¶ c
168¶ e
169¶ e
170¶ e
171¶ b
172¶ a
173¶ b
174¶ d
175¶ e
176¶ e
177¶ a
178¶ d
179¶ e
180¶ c
181¶ d
182¶ a
183¶ b
184¶ d
185¶ b
186¶ c
187¶ b
188¶ c
189¶ e
190¶ d
191¶ a
192¶ e
193¶ e
194¶ c
195¶ a
196¶ e
197¶ b then 197 d
198¶ d
199¶ b
200¶ d

You might also like